Download as pdf or txt
Download as pdf or txt
You are on page 1of 306

Obstetrics and Gynecology

SMLE Questions and Answers

Ver.1
Table of Content
Chapter 1: Obstetrics ................................................................................................................................................... 3
Contraindications to Pregnancy (Medications and Vaccines) ............................................................................................................................................................................................. 4

Cervical Incompetence ........................................................................................................................................................................................................................................................ 9

Antenatal care .................................................................................................................................................................................................................................................................... 15

Fetal Medicine ................................................................................................................................................................................................................................................................... 20

Hypertension in Pregnancy and Preeclampsia .................................................................................................................................................................................................................. 28

Diabetes Mellitus and Gestational Diabetes Mellitus ....................................................................................................................................................................................................... 52

Pregnancy Related Medical and Surgical Conditions ....................................................................................................................................................................................................... 61

Antepartum Hemorrhage (APH) ....................................................................................................................................................................................................................................... 65

Labor and CTG Monitoring .............................................................................................................................................................................................................................................. 80

Preterm labor, Preterm Rupture of Membrane (PROM) and Premature Preterm Rupture of Membrane (PPROM) ..................................................................................................... 119

Postpartum Hemorrhage (PPH) ....................................................................................................................................................................................................................................... 134

Postpartum ....................................................................................................................................................................................................................................................................... 152

Chapter 2: Gynecology............................................................................................................................................. 154

Puberty Disorders, Pediatrics, Adolescent and Young Gynecology ............................................................................................................................................................................... 155

Menstrual Cycle Abnormalities and Abnormal Uterine Bleeding (AUB) ...................................................................................................................................................................... 157

Vaginal Infections ........................................................................................................................................................................................................................................................... 169

Pelvic Inflammatory Disease (PID) ................................................................................................................................................................................................................................ 179

Contraception and Hormonal Replacement Therapy (HRT) .......................................................................................................................................................................................... 186

Abortion, Pregnancy Loss, and Intrauterine Fetal Demise (IUFD) ................................................................................................................................................................................ 193

Ectopic Pregnancy ........................................................................................................................................................................................................................................................... 207

Gestational Trophoblastic Disease (Molar pregnancy and Choriocarcinoma) ............................................................................................................................................................... 229

Adnexal masses ............................................................................................................................................................................................................................................................... 238

Leiomyoma (Uterine Fibroids) and Leiomyosarcoma .................................................................................................................................................................................................... 239

Endometriosis .................................................................................................................................................................................................................................................................. 248

Adenomyosis ................................................................................................................................................................................................................................................................... 256

Asherman’s Syndrome .................................................................................................................................................................................................................................................... 261

Cervical Cancer and Screening ....................................................................................................................................................................................................................................... 263

Endometrial Polyp, Hyperplasia and Carcinoma ............................................................................................................................................................................................................ 282

Urogynecology ................................................................................................................................................................................................................................................................ 297


Chapter 1: Obstetrics

3|Page
Obstetrics and Gynecology: SMLE Questions and Answers Chapter 1: Obstetrics

Contraindications to Pregnancy (Medications and Vaccines)

A 28-year-old female with history of recurrent pregnancy loss. She comes now want to
improve her immunity before trying to conceive. What you will give her?

A. Influenza vaccine
B. Rubella vaccine
C. Hepatitis
D. Immunoglobulin

Correct Answer: B
Explanation: (According to UpToDate)
Congenital rubella syndrome — Rubella infection can have catastrophic effects on the developing
fetus, resulting in spontaneous abortion, fetal infection, stillbirth, or intrauterine growth restriction

Female her previous pregnancy is stillbirth and now she want to pregnant and ask the doctor
about all the vaccines that she is need before conception and reduce the stillbirth?

A. Rubella
B. Varicella
C. Influenza

Correct Answer: A
Explanation: (According to UpToDate)
Congenital rubella syndrome — Rubella infection can have catastrophic effects on the developing
fetus, resulting in spontaneous abortion, fetal infection, stillbirth, or intrauterine growth restriction

4|Page
Obstetrics and Gynecology: SMLE Questions and Answers Chapter 1: Obstetrics

Married women came in winter to OB /gyn clinic she want to conceive later what vaccine you
should give her before conceive?

A. Rubella
B. Influenza
C. Varicella
D. Tdap

Correct Answer: B
Explanation: (According to Williams Obstetrics and UpToDate)
PRECONCEPTION IMMUNIZATION

- Influenza
o Vaccinate all women who will be pregnant during flu season. Vaccinate high-risk
women prior to flu season.
o Vaccination against influenza throughout the influenza season, but optimally in
October or November, is recommended by the Centers for Disease Control and
Prevention (CDC) and the American College of Obstetricians and Gynecologists for
all women who will be pregnant during the influenza season.

- Measles, mumps, rubella (MMR)


o Screen for rubella immunity. If nonimmune, vaccinate and counsel on the need for
effective contraception during the subsequent month.
- Varicella
o Screen for varicella immunity. If nonimmune, vaccinate and counsel on the need for
effective contraception during the subsequent month.
- Tdap (tetanus, diphtheria, pertussis)
o Update vaccination in all reproductive-aged women.

5|Page
Obstetrics and Gynecology: SMLE Questions and Answers Chapter 1: Obstetrics

Pregnant in 2nd trimester which vaccine to give her?

A. H. influenza
B. DTAP

Correct Answer: A
Explanation: (According to UpToDate)
- Tdap
• Pregnant women should receive Tdap, ideally during the early part of the 27 to 36
week gestational age range (third trimester)
- Inactivated Influenza Vaccine
• All women who are pregnant or might be pregnant during the influenza season
should receive the inactivated influenza vaccine as soon as it becomes available and
before onset of influenza activity in the community, regardless of their stage of
pregnancy

Pregnant lady, what vaccine should she get at first visit?

A. Influenza
B. Tdap

Correct Answer: A
Explanation: (According to UpToDate)
- Tdap
• Pregnant women should receive Tdap, ideally during the early part of the 27 to 36
week gestational age range (third trimester)
- Inactivated Influenza Vaccine
• All women who are pregnant or might be pregnant during the influenza season
should receive the inactivated influenza vaccine as soon as it becomes available and
before onset of influenza activity in the community, regardless of their stage of
pregnancy

Pregnant nullipara what vaccine should you give her?

A. Influenza
B. DTaP
C. Rh immunoglobulin

Correct Answer: Depends on the gestational age


Explanation:
- If 1st or second trimester à I would go with influenza
- If third trimester à I would go with Tdap

6|Page
Obstetrics and Gynecology: SMLE Questions and Answers Chapter 1: Obstetrics

Female have regular cycle every 30 days her last period before 36, she received rubella vaccine
before 3 weeks. She was asked not to get pregnant until 2 months of receiving rubella , But she
got pregnant, what’s the most likely pregnancy outcome?

A. Not affected
B. Associated with congenital malformations.

Correct Answer: A
Explanation: (According to Williams Obstetrics and UpToDate)
- Inadvertent administration of measles, mumps, rubella (MMR) or varicella vaccines during
pregnancy should not generally be considered indications for pregnancy termination.
- ACIP and American College of Obstetricians and Gynecologists (ACOG)
recommendations to avoid pregnancy for one month following each dose of a live vaccine.
Nevertheless, adverse outcomes in women who became pregnant soon after receiving these
vaccines have not been established

Pregnant women doesnt have rubella vaccine what should do?

A. Take in second trimester.


B. Postpartum

Correct Answer: B
Explanation: (According to UpToDate)
POSTPARTUM IMMUNIZATION
- MMR and varicella – The following vaccines should be given before discharge to protect
a nonimmune mother and newborn:
o MMR – The measles, mumps, rubella (MMR) vaccine should be administered to
women nonimmune to rubella or measles
o Varicella vaccination is recommended for women without evidence of immunity.
The first dose is given while the patient is in the hospital and the second dose is given four to eight
weeks later, which typically coincides with the routine postpartum visit. Breastfeeding is not a
contraindication to administration.

7|Page
Obstetrics and Gynecology: SMLE Questions and Answers Chapter 1: Obstetrics

Pregnant her child school had an outbreak and she’s afraid to get to her child which vaccine
she should get ?

A. DtaP
B. Influenza
C. Rubella
D. Varicella

Correct Answer: B
Explanation: (According to UpToDate)
Inactivated Influenza Vaccine
- All women who are pregnant or might be pregnant during the influenza season should
receive the inactivated influenza vaccine as soon as it becomes available and before onset
of influenza activity in the community, regardless of their stage of pregnancy

A woman was taking highly androgenic progesterone without knowing she is pregnant. What
complication will her daughter face?

A. Nothing will change


B. Hirsutism
C. Masculinization
D. Feminization

Correct Answer: C
Explanation: (According to UpToDate and Williams Obstetrics)
Gestational hyperandrogenism — Virilization in an XX individual with normal female internal
anatomy can result from exposure to maternal androgen or synthetic progestational agents. Because
the placenta produces the aromatase enzyme, which converts androgens to estrogens, only very
high levels of maternal androgens can overcome placental aromatase to cause virilization of the
fetus. Causes include maternal luteoma or theca lutein cysts. These disorders are suggested by a
history of maternal virilization during pregnancy and/or exogenous progestin or androgen
exposure

8|Page
Obstetrics and Gynecology: SMLE Questions and Answers Chapter 1: Obstetrics

Cervical Incompetence

G2P0 20 weeks of gestation, with cervical incompetence (cervix length 30mm), what’s the
most appropriate management?

A. Cervical cerclage
B. Strict bed rest
C. Progesterone supplementation

Correct Answer: C
Explanation: (According to ACOG and Uptodate)
Indications for Cervical Cerclage in Women With Singleton Pregnancies
- History Indicated Cerclage:
A. History of one or more second-trimester pregnancy losses related to painless cervical
dilation and in the absence of labor or abruptio placentae
B. Prior cerclage due to painless cervical dilation in the second trimester
C. Placed at approximately 13–14 weeks of gestation.
- Physical Examination Indicated Cerclage:
• Painless cervical dilation in the second trimester (known as emergency or rescue
cerclage)
- Ultrasonographic Finding With a History of Prior Preterm Birth Indicated Cerclage:
• Current singleton pregnancy, prior spontaneous preterm birth at less than 34 weeks of
gestation, and short cervical length (less than 25 mm)
• Before 24 weeks of gestation

Candidates for progesterone supplementation:


- Patients with singleton pregnancy and a short cervix (≤25 mm)
- Twin pregnany with a short cervix (<25mm)

Let’s Exclude!
- Cervical cerclage à doesn’t meet the criteria
- Strict bed rest à not used anymore.
- Progesterone supplementation à the only reasonable option.

9|Page
Obstetrics and Gynecology: SMLE Questions and Answers Chapter 1: Obstetrics
Female with previous 2 preterm labor, now she is at 20 weeks of gestation and her cervix
opened 30 mm, what you will do?

A. Immediate cervical cerclage


B. Give tocolytic & wait
C. Strict bed rest
D. Progesterone supplement

Correct Answer: D
Explanation: (According to ACOG and Uptodate)
Indications for Cervical Cerclage in Women With Singleton Pregnancies
- History Indicated Cerclage:
D. History of one or more second-trimester pregnancy losses related to painless cervical
dilation and in the absence of labor or abruptio placentae
E. Prior cerclage due to painless cervical dilation in the second trimester
F. Placed at approximately 13–14 weeks of gestation.
- Physical Examination Indicated Cerclage:
• Painless cervical dilation in the second trimester (known as emergency or rescue
cerclage)
- Ultrasonographic Finding With a History of Prior Preterm Birth Indicated Cerclage:
• Current singleton pregnancy, prior spontaneous preterm birth at less than 34 weeks of
gestation, and short cervical length (less than 25 mm)
• Before 24 weeks of gestation

Candidates for progesterone supplementation:


- Patients with singleton pregnancy and a short cervix (≤25 mm)
- Twin pregnany with a short cervix (<25mm)

Let’s Exclude!
- Cervical cerclage à (History indicated cerclage placed at 13-16 weeks, she is late) (US
indicated cerclage we need cervical length <25mm)
- Give tocolytic à she doesn’t have contractions
- Strict bed rest à not used anymore.
- Progesterone supplementation à the only reasonable option.

10 | P a g e
Obstetrics and Gynecology: SMLE Questions and Answers Chapter 1: Obstetrics
Which of the following is the indication of cervical cercalge?

A. Cervical cerclage length less than 30


B. Cervical cerclage length less than 35
C. Cervical cerclage length less than 25
D. Cervical cerclage length less than 20

Correct Answer: C
Explanation: (According to ACOG)
Indications for Cervical Cerclage in Women With Singleton Pregnancies
- History Indicated Cerclage:
G. History of one or more second-trimester pregnancy losses related to painless cervical
dilation and in the absence of labor or abruptio placentae
H. Prior cerclage due to painless cervical dilation in the second trimester
I. Placed at approximately 13–14 weeks of gestation.
- Physical Examination Indicated Cerclage:
• Painless cervical dilation in the second trimester (known as emergency or rescue
cerclage)
- Ultrasonographic Finding With a History of Prior Preterm Birth Indicated Cerclage:
• Current singleton pregnancy, prior spontaneous preterm birth at less than 34 weeks of
gestation, and short cervical length (less than 25 mm)
• Before 24 weeks of gestation

Pregnant at 18 weeks of gestation with cervical incompetence, history of previous fetal


passage at 28 weeks. What is the management?

A. Cervical cerclage
B. OCPs
C. Follow up by serial us visits

Correct Answer: C
Explanation: (According to Uptodate)
For women with a singleton pregnancy and a history of prior spontaneous preterm birth, we
begin TVUS cervical length screening at 14 to 16 weeks of gestation, and if her Cervical
length is:
- >25mm-> we perform serial examinations.
- <25mm-> Cervical cerclage placement before 24 weeks of gestation

Note:
I would go for Cervical cerclage if:

11 | P a g e
Obstetrics and Gynecology: SMLE Questions and Answers Chapter 1: Obstetrics
- Her cervix now is <25mm, and now she is at <24 weeks of gestation (US indicated
cerclage)
OR
- She has a history or preterm birth, and now she is at 13 or 14 weeks of gestation (History
indicated cerclage)

Pregnant in 8 weak gastation had misscarrage in previous prgnancy in 20 weak what


should you do at this pregnancy?

A. Close antenatal follow up


B. Cervical cerclage at 16-18 weeks

Correct Answer: A
Explanation:
According to Uptodate
For women with a singleton pregnancy and a history of prior spontaneous preterm birth, we
begin TVUS cervical length screening at 14 to 16 weeks of gestation, and if her Cervical
length is:
- >25mm-> we perform serial examinations.
- <25mm-> Cervical cerclage placement before 24 weeks of gestation

According to Williams Obstetrics:


or women with an unequivocal history of second-trimester painless delivery, prophylactic
cerclage placement is an option and reinforces a weak cervix by an encircling suture.

Cervical length screening is now recommended by both the American College of Obstetricians
and Gynecologists and the Society for Maternal-Fetal Medicine for women with prior preterm
birth. Between 16 and 24 weeks' gestation, sonographic cervical measurement is completed
every 2 weeks.
- If an initial or subsequent cervical length is 25 to 29 mmà then a weekly interval is
considered.
- If the cervical length measures <25 mmà cerclage is offered to this group of women.

12 | P a g e
Obstetrics and Gynecology: SMLE Questions and Answers Chapter 1: Obstetrics
Pregnant at 13 weeks of gestation with history or spontaneous fetal loss at 20 week. What is
the most appropriate action to do?

A. Regular Follow up without specific intervention


B. Cervical cerclage now

Correct Answer: B
Explanation: (According to ACOG)
Indications for Cervical Cerclage in Women With Singleton Pregnancies
- History Indicated Cerclage:
J. History of one or more second-trimester pregnancy losses related to painless cervical
dilation and in the absence of labor or abruptio placentae
K. Prior cerclage due to painless cervical dilation in the second trimester
L. Placed at approximately 13–14 weeks of gestation.

Patient at 8 weeks of gestation diagnosed as cervical incompetence, what to do?

A. Do cervical suture now


B. Do cervical suture at 13-14 weeks
C. Start beta mimitic drug
D. Confirm with heglar dilator

Correct Answer: B
Explanation: (According to ACOG)
Indications for Cervical Cerclage in Women With Singleton Pregnancies
- History Indicated Cerclage:
M. History of one or more second-trimester pregnancy losses related to painless cervical
dilation and in the absence of labor or abruptio placentae
N. Prior cerclage due to painless cervical dilation in the second trimester
O. Placed at approximately 13–14 weeks of gestation.

Pregnant at 18 weeks of gestation, she has a history of recurrent fetal loss. Now came to ER
due to sudden fetal parts expulsion What is the diagnosis?

A. Bicornuate uterus
B. Cervical incompetence

Correct Answer: B
Explanation: (According to Uptodate)
Cervical insufficiency: Inability of the uterine cervix to retain a pregnancy in the absence of the
signs and symptoms of clinical contractions, or labor, or both in the second trimester

13 | P a g e
Obstetrics and Gynecology: SMLE Questions and Answers Chapter 1: Obstetrics
Female had history of preterm labor at 34 wks and now she is on 24 wks what is the highest
diagnostic value for her case?

A. Cervical length measurement


B. Speculum

Correct Answer: A

Pregnant woman with history of preterm labor two times, presented with vaginal spotting
What to give her?

A. Estrogen
B. Progesterone
C. Indomethacin
D. Mg sulphate

Correct Answer: B
Explanation: (According to Uptodate)
Women with a prior spontaneous preterm birth are at high risk for recurrence and are offered
progesterone supplementation (vaginal or intramuscular) to reduce this risk based on their history
of spontaneous preterm birth alone

14 | P a g e
Obstetrics and Gynecology: SMLE Questions and Answers Chapter 1: Obstetrics

Antenatal care

What is the Folic acid quantity for a healthy lady wants to conceive and with no prior
diseases or disorders?

A. 1 mg
B. 5 mg
C. 10 mg
D. 15 mg

Correct Answer: A
Explanation: (According to ACOG, Uptodate and Williams Obstetrics)
Female prepregnancy folic acid supplementation should be encouraged to reduce the risk of
NTDs.
- All women of reproductive age (15–45 years) should take folic acid supplementation. For
average-risk women, supplementation with 400 micrograms per day is adequate.
- Women at increased risk of NTDs, including women with a prior pregnancy with an NTD
or women with seizure disorders, should be counseled to take 4 mg of folic acid daily
- Standard 1mg of folate in prenatal vitamins

Placenta in implanted in the uterine wall, what is that?

A. Placenta previa
B. Placenta accrete
C. Placenta increta
D. Placenta perecreta

Correct Answer: C
Explanation:
Abnormal Placental Implantation
- Placenta Accrete: chorionic villi Attach to the myometrium
- Placenta Increta: chorionic villi Invade into the myometrium
- Placenta Percreta : chorionic villi Penetrate though the myometrium, penetrate the serosa

A 46-year-old, G3P1+1 at 34 weeks’ gestation presented to antenatal clinic for regular


check-up, she has unremarkable medical history and uncomplicated pregnancy Braxton
Hicks and non-pruritic cervical discharge. Her pre-pregnancy weight was 54.4 on
examination cervical length was 33 mm.

VS were given & I believe they were normal.

15 | P a g e
Obstetrics and Gynecology: SMLE Questions and Answers Chapter 1: Obstetrics
Current weight: 52

Rubella AB: -ve

HBsAg: -ve

Blood type: O+

Which of the following is the most appropriate next step?

A. Follow up after 2 weeks


B. OGGT test
C. Do rubella Ab test / Repeat rubella screen
D. Give anti-D Ab

Correct Answer: A
Explanation: (According to Williams Obstetrics)
PRENATAL VISITS
Traditionally scheduled at 4-week intervals until 28 weeks, then every 2 weeks until 36 weeks, and
weekly thereafter. Women with complicated pregnancies-for example, with twins or diabetes-often
require return visits at 1- to 2-week intervals

Pregnant women, her last menstrual period 7th of May, she has regular period and is sure
about it. What is the Expected date of delivery?

A. 10 February next year


B. 10 December same year
C. 25 December next year
D. 30 February next year

Correct Answer: A
Explanation:
The accurate date is 14th of Feb next year

How to calculate the Estimated Delivery Date?


- Day + 7 / Month +9 / Year +1 or 0 (depending on the month)
- E.g., Patient LMP was 18th of May 2020, calculate the EDD?
So, 18+7 / 5+9 / 2020 +1 or 0 (depending on the month)
Her EDD is 25/2/2021

16 | P a g e
Obstetrics and Gynecology: SMLE Questions and Answers Chapter 1: Obstetrics
GBS vaginal swab screening in pregnant women?

A. 15 weeks
B. 25 weeks
C. 35 weeks
D. 40 weeks

Correct Answer: C
Explanation: (According to ACOG)
Routine antepartum GBS vagina and rectal cultures on all pregnant women at 35 to 37 weeks

12 Weeks pregnant, what will her blood test show?

A. Decrease in serum creatinine


B. Increase in plasma sodium
C. Increase in plasma BUN
D. Decrease in BUN

Correct Answer: A
Explanation:

17 | P a g e
Obstetrics and Gynecology: SMLE Questions and Answers Chapter 1: Obstetrics
(1) Female pregnant, what of the following true regarding Elevated BhCG?

A. High BhCg indicator of ectopic pregnancy


B. High BhCg in second Trimester indicator of molar pregnancy
C. High BhCg in second Trimester is the most sensitive marker of Down syndrome
D. High Bhcg can cause depression of TSH

Correct Answer: D

Another recall

(2) Female pregnant, what of the following is true regarding elevated BhCG?

A. High BhCg indicator of ectopic pregnancy


B. High BhCg in second trimester indicator of molar pregnancy.
C. High BhCg in second trimester is the most sensitive marker of Down syndrome.
D. High Bhcg can cause elevation of TRH which causes hyperthyroidism

Correct Answer: C
Explanation:
- Hyperthyroidism in pregnancy is caused by direct stimulation of the maternal thyroid
gland by elevated levels of human chorionic gonadotropin (hCG), which can be associated
with a transient lowering in serum TRH and TSH levels
- Second-trimester (QUADRUPLE test) total levels of hCG, dimeric inhibin A (DIA), AFP,
unconjugated estriol (uE3) are the most sensitive test (its QUADRUPLE test not BhCG
alone)

So, for our CASES:


( In the first recall) (1)
- D is the correct answer because high B-HCG causes reciprocal suppression of TSH and
TRH.
- Why not C? it’s true that high BHCG is a sensitive marker for down syndrome (but not
ALONE!! It’s a quadruple test)

(In the second recall) (2)


- D is wrong! High bhCG never causes elevation of TRH levels!! (it causes suppression of
TRH and TSH due to the direct stimulation of the thyroid gland” hyperthyroidism”)
- So, in that case C is the only correct answer so I would go with C

18 | P a g e
Obstetrics and Gynecology: SMLE Questions and Answers Chapter 1: Obstetrics
What would you expect in pregnancy?

A. Hematocrit decrease by 20-25%


B. Hematocrit decrease by 40-45%
C. Blood volume increase by 20-25%
D. Blood volume increase by 40-45%

Correct Answer: D
Explanation:

What tine breastfeeding counseling is best done?

A. Before conceiving
B. 1st trimester or prenatal
C. Postpartum

Correct Answer: B
Explanation:

19 | P a g e
Obstetrics and Gynecology: SMLE Questions and Answers Chapter 1: Obstetrics

Fetal Medicine

Patient with absent fetal heartbeat on ultrasound, which of the following is best to use for
chromosomal analysis?
A. Umbilical cord sampling
B. Amniotic fluid sampling
C. Fetal cord blood
D. Placental tissue

Correct Answer: B
Explanation: (According to Berghella)
Genetic evaluation via karyotype or preferably micro-array if available should be performed on
all stillbirths. Ideally, an amniocentesis should be performed prior to delivery.

Mother came for antenatal care and US shows week 32 reversed end diastolic blood flow,
what is the most appropriate management?

A. Follow up 2 week and reassess


B. Immediate delivery now
C. Administer steroids 1 week and delivery
D. NST

Correct Answer: B
Explanation: (According to UpToDate)
The presence of REDV at any gestational age beyond 32 weeks should prompt consideration for
immediate delivery. This is supported by Society for Maternal-Fetal Medicine guidelines, which
recommend intense fetal surveillance of these fetuses and continuing expectant management until
32 weeks as long as fetal surveillance remains reassuring

20 | P a g e
Obstetrics and Gynecology: SMLE Questions and Answers Chapter 1: Obstetrics
A 35-year-old mother with GA 33 weeks, she has an ultrasound which showed reversed end
diastolic flow in umbilical artery. CTG was normal. what is your appropriate
management?

A. Immediate delivery by CS
B. Follow up after 2 weeks
C. give corticosteroids and deliver within 1 week
D. deliver at 37 week

Correct Answer: A
Explanation: (According to UpToDate)
The presence of REDV at any gestational age beyond 32 weeks should prompt consideration for
immediate delivery. This is supported by Society for Maternal-Fetal Medicine guidelines, which
recommend intense fetal surveillance of these fetuses and continuing expectant management until
32 weeks as long as fetal surveillance remains reassuring

Pregnant at 33 weeks gestation has reversed flow of doppler artery of umbilical, what will
you do?

A. Emergent CS
B. Give steroids and wait for 1 week
C. Wait till 37 weeks

Correct Answer: A
Explanation: (According to UpToDate)
The presence of REDV at any gestational age beyond 32 weeks should prompt consideration for
immediate delivery. This is supported by Society for Maternal-Fetal Medicine guidelines, which
recommend intense fetal surveillance of these fetuses and continuing expectant management until
32 weeks as long as fetal surveillance remains reassuring

21 | P a g e
Obstetrics and Gynecology: SMLE Questions and Answers Chapter 1: Obstetrics

Pregnant lady at 30 weeks on antenatal care on U/S: finding fetus size decrease than before
with oligohyromnios, doppler of umbilical artery find reversed diastolic flow mother
denied any loss of fetus movement, what is the appropriate next step?

A. Non-stress test
B. Serial us after one week
C. Serial doppler for umbilical artery after two weeks
D. Kick fetal chart

Correct Answer: A
Explanation: (According to UpToDate)

A 28 year-old female, pregnant at 35 weeks of gestation presenting with decreased fetal


movement, CTG was reassuring with fetal HR 130, then 1 hr later CTG showing good
variability. What is the best management for her?

A. Observe for 24 hr.

22 | P a g e
Obstetrics and Gynecology: SMLE Questions and Answers Chapter 1: Obstetrics
B. Induction of labor
C. C/S
D. Discharge her with fetal kick chart

Correct Answer: D
Explanation: (According to ACOG)
- For a pregnant individual reporting decreased fetal movement after viability, one-time
antenatal fetal surveillance at the time the decreased movement is reported may be
considered. These include fetal movement assessment, nonstress test, contraction stress
test, fetal biophysical profile, modified biophysical profile and umbilical artery Doppler
velocimetry. (NOT OBSERVATION)
- If the NST is reactive, we believe that ultrasound examination is a valuable additional tool
for assessment of pregnancies complicated by persistent DFM, and is reassuring for
mothers.

Note:
- If there’s a biophysical profile or US examination in the choices, I would go with it. But
with these options? I would go with D

Pregnant lady 32 weeks GA is worried that her baby stopped moving. What is the next
more appropriate step for this case?

A. Non-stress test
B. Biophysical profile
C. Pelvic examination
D. Pelvic US

Correct Answer: A
Explanation:
Management of reduced fetal movement:
- Nonstress test (First)
- Biophysical profile (US) (Second)

Patient at 29 weeks, didn't feel fetal movement for 1 day, CTG was reactive, Biophysical
profile was 8. What to do next?

A. Steroid and repeat Biophysical profile after 24 hours


B. Repeat Biophysical profile at 1 week
C. IOL
D. Urgent CS

Correct Answer: B

23 | P a g e
Obstetrics and Gynecology: SMLE Questions and Answers Chapter 1: Obstetrics
Explanation:
Her CTG is normal and Biophysical profile 8 or 10 is normal.

- For women <37 weeks of gestation with persistent DFM and normal fetal evaluation à
nonstress testing and ultrasound examination twice weekly is recommended

Pregnant around 30 weeks who is a case of Rh alloimmunization, fetus was found to have
anemia, Management?

A. Deliver immediately
B. Duplex
C. Fetal blood transfusion

Correct Answer: C
Explanation: (According to ACOG)
- Delivery of the infant of an alloimmunized patient is a controversial subject, and literature
on the subject is limited. Standard treatment is to prolong the pregnancy until the fetus
reaches a gestational age necessary for survival. Intrauterine transfusion up to 36 weeks of
gestation when intravascular transfusion is feasible in order to limit neonatal morbidity.
Delivery can then be accomplished between 37 and 38 weeks of gestation.

- Douplexà why? He already gave me the diagnosis (anemia) the next step (after MCA
doppler) would be obtaining fetal blood by cordocentesis for hemoglobin determination.

Mother who is Rh negative delivered a baby who is Rh + she was given Anti-D Ig 300
microg what does it cover ?

A. 15 ml of the whole fetal blood


B. 30 ml of the whole fetal blood
C. 10 ml of the whole fetal blood
D. 30 ml of Rh(D) positive fetal RBCs

Correct Answer: B
Explanation:
According to ACOG
A prophylactic dose of 300 micrograms of anti-D immune globulin can prevent Rh D
alloimmunization after exposure to up to
- 30 mL of Rh D-positive fetal whole blood or 15 mL of fetal red blood cells

According to Williams Obstetrics

24 | P a g e
Obstetrics and Gynecology: SMLE Questions and Answers Chapter 1: Obstetrics
The dosage of anti-D immune globulin is calculated from the estimated volume of the fetal-to-
maternal hemorrhage
- One 300-μg dose is given for each 15 mL of fetal red cells or 30 mL of fetal whole blood to
be neutralized.

Pregnant of twins, one has increased nuchal translucency in Ultrasound. What will he
have?

A. Congenital cardiac malformation


B. Turner syndrome
C. Neural tube defect

Correct Answer: A
Explanation:
All chromosomal syndromes have cardiac disease

How to know fetal weight intrapartum at 37 weeks of gestation?

A. Femur length
B. Head circumflex
C. Biparietal diameter
D. Abdominal circumference

Correct Answer: D

Women delivered a baby with Down syndrome, and she wants to know about future
pregnancy. Which of the following is BEST choice of her?

A. Karyotype of infant
B. Karyotype of infant and mother.
C. U/S in next pregnancy
D. Amniocentesis in next pregnancy

Correct Answer: D
Explanation:
Down syndrome prenatal screening:
- Combined test “US determination of nuchal translucency NT + determination of
biochemical markers associated with aneuploidy
Down syndrome definitive prenatal diagnosis:
- Chorionic villi sampling (CVS) or Amniocentesis

25 | P a g e
Obstetrics and Gynecology: SMLE Questions and Answers Chapter 1: Obstetrics
Pregnant who has a child with down syndrome. She’s concerned about having another
child with down syndrome. What is the best test to rule out down syndrome in the second
trimester?

A. Amniotic fluid sample


B. Chorionic villous sample
C. Triple test

Correct Answer: A
Explanation:
- Chorionic Villi Sampling (CVS): is the procedure of choice for first trimester testing
(between 10-13 weeks.)
- Amniocentesis: is the procedure of choice for second trimester testing optimally performed
(ACOG: between 15-20 weeks.), (Uptodate: between 15-17+6)

Best indicator of chromosomal abnormalities in which week?

A. 16 to 18wk
B. 18 to 22wk
C. 13 to 16wk
D. 24 to 28wk

Correct Answer: A
Explanation:
What does the Question mean? “The correct timing of the DIAGNOSTIC test for chromosomal
abnormalities”

So, what are the diagnostic tests? Chorionic Villi Sampling (CVS) and Amniocentesis!
- Chorionic Villi Sampling (CVS): is the procedure of choice for first trimester testing
(between 10-13 weeks.)
- Amniocentesis: is the procedure of choice for second trimester testing optimally performed
(ACOG: between 15-20 weeks.), (Uptodate: between 15-17+6)

26 | P a g e
Obstetrics and Gynecology: SMLE Questions and Answers Chapter 1: Obstetrics
What is the time interval between ovulation and cleavage in dichorionic diamniotic twins?

A. 0-3 days
B. 4-8 days
C. 9-12 days
D. >12 days

Correct Answer: A
Explanation:

27 | P a g e
Obstetrics and Gynecology: SMLE Questions and Answers Chapter 1: Obstetrics

Hypertension in Pregnancy and Preeclampsia

Pregnant lady at 39 weeks of gestation, her routine BP throughout the pregnancy was
120/80, then suddenly became 150/90 what is the diagnosis?

A. Eclampsia
B. Gestational hypertension
C. Chronic hypertension
D. Superimposed hypertension

Correct Answer: B
Explanation:
- Preeclampsia à new-onset gestational hypertension with proteinuria or end-organ
dysfunction
- Eclampsia à severe form of preeclampsia with convulsive seizures
- Gestational hypertension à onset after 20 weeks' gestation without proteinuria or end-
organ dysfunction
- Chronic hypertensionà < 20 weeks' gestation or before pregnancy
- Superimposed hypertension à chronic hypertension with superimposed preeclampsia

Pregnant present at 38 weeks in labor her BP 150/90 and elevated proteins /creatinine
ratio. What is the diagnosis?

A. Preeclampsia
B. Chronic hypertension
C. Gestational hypertension
D. Superimposed hypertension

Correct Answer: A
Explanation:
Preeclampsia
New onset of hypertension with proteinuria ( ≥0.3 g or protein/creatinine ratio ≥0.3
(mg/mg)) or end-organ dysfunction after 20 weeks of gestation

Female 36 year, at 15 weeks of gestation, came with hypertension 180/110 no proteinuria,


what is the diagnosis?

A. Primary HTN.
B. Pregnancy induced HTN.
C. White coat syndrome.
D. Eclampsia

28 | P a g e
Obstetrics and Gynecology: SMLE Questions and Answers Chapter 1: Obstetrics

Correct Answer: A
Explanation:
Chronic hypertensionà < 20 weeks' gestation or before pregnancy

Female 36 year, at 15 weeks of gestation, complaining of headache, blurred vision since 2


weeks with hypertension, what is the diagnosis?

A. Primary HTN.
B. Pregnancy induced HTN.
C. White coat syndrome.
D. Eclampsia

Correct Answer: A
Explanation:
Chronic hypertensionà < 20 weeks' gestation or before pregnancy

First line treatment of hypertension in pregnancy is?

A. Methyldopa
B. Labetolol
C. Hydralazine
D. Nifdepine

Correct Answer: B
Explanation: (According to ACOG)
- For chronic maintenance treatment, oral labetalol (first line) or nifedipine (second line)
are reasonable options and are recommended above all other antihypertensive drugs.
Methyldopa is generally less favored.

29 | P a g e
Obstetrics and Gynecology: SMLE Questions and Answers Chapter 1: Obstetrics
Pregnant female with Hypertension 140/90, no proteinuria, what is first line in
management?

A. Methyldopa
B. Labetalol
C. Nifedipine
D. Hydralazine

Correct Answer: B
Explanation: (According to ACOG)
- For chronic maintenance treatment, oral labetalol (first line) or nifedipine (second line)
are reasonable options and are recommended above all other antihypertensive drugs.
Methyldopa is generally less favored.

Management of chronic hypertension in pregnancy?

A. Methyldopa
B. Labetalol
C. Nifedipine
D. Hydralazine

Correct Answer: B
Explanation:
For chronic maintenance treatment, oral labetalol (first line) or nifedipine (second line) are
reasonable options and are recommended above all other antihypertensive drugs. Methyldopa is
generally less favored.

Pregnant bp 140/90 or 150/90 what to give?

A. Nifidepine
B. Hydralazine
C. Metoprolol

Correct Answer: A
Explanation: (According to ACOG)
Oral Antihypertensive Agents in Pregnancy for chronic HTN , gestational HTN, or
preeclampsia without severe features
1- Labetalol (first line)
2- Nifedipine
3- Methyldopa
Antihypertensive Agents Used for Urgent Blood Pressure Control in Pregnancy
1- Labetalol (first line)
2- Hydralazine

30 | P a g e
Obstetrics and Gynecology: SMLE Questions and Answers Chapter 1: Obstetrics
3- Nifedipine

Pregnant at 20 weeks of gestation with HTN 160/90 what is the appropriate


antihypertensive drug for her?

A. Labetalol
B. Hydralazine
C. Methyldopa
D. Nifedipine

Correct Answer: A
Explanation: (According to ACOG)
Antihypertensive agents for urgent blood pressure control in pregnancy:
- IV labetalol (first line)
- IV Hydralazine (second line)

Severe preeclampsia, acute management of HTN drug?

A. Hydralazine
B. Methyldopa
C. Nifedipine
D. Sodium nitroprusside

Correct Answer: A
Explanation: (According to ACOG)
Antihypertensive agents for urgent blood pressure control in pregnancy:
- IV labetalol (first line)
- IV Hydralazine (second line)

Pregnant lady had seizure and is unconscious, her baby is healthy, what to do?

A. MgSo4
B. Establish airway
C. Fluids
D. Urgent delivery

31 | P a g e
Obstetrics and Gynecology: SMLE Questions and Answers Chapter 1: Obstetrics

Correct Answer: B
Explanation:
Management of Eclampsia:
⁃ ABC:
• Calling for help, prevention of maternal injury, placement in lateral decubitus
position, prevention of aspiration, administration of oxygen, and monitoring vital
signs including oxygen saturation.
⁃ Anticonvulsive Therapy:
• Magnesium sulfate
- Delivery! after maternal hemodynamic stabilization

A 23-year-old primigravida presented at 32-weeks of gestation with seizure. Blood pressure


160/110 mmHg, Heart rate 78 /min, Respiratory rate 18 /min , Temperature 36.6°C

Urine :Protein +++

Which of the following is the most appropriate next step in management?

A. Steroids
B. Diuretics
C. Hydralazine
D. Magnesium sulphate

Correct Answer: D
Explanation:
Management of Eclampsia:
⁃ ABC:
• Calling for help, prevention of maternal injury, placement in lateral decubitus
position, prevention of aspiration, administration of oxygen, and monitoring vital
signs including oxygen saturation.
⁃ Anticonvulsive Therapy:
• Magnesium sulfate
- Delivery! after maternal hemodynamic stabilization

Pregnant at 34 weeks with blurred vision, headache and her BP 170/90 What to do?

A. Stabilize + MgSo and wait till 37 weeks


B. Call anaesthesia now and deliver
C. Stabilize and give MgSo and deliver

Correct Answer: C

32 | P a g e
Obstetrics and Gynecology: SMLE Questions and Answers Chapter 1: Obstetrics
Explanation:
Management of Preeclampsia without severe features
o <37 weeks of gestation:
Expectant management + oral labetalol or nifedipine
o ≥37 weeks of gestation
IOL + oral labetalol or nifedipine
o ≥34 weeks of gestation with pretem labor or PPROM
IOL + oral labetalol or nifedipine
Management of Preeclampsia with severe features
o <34 weeks of gestation:
Expectant management
Admission + Corticosteroid + Magnesium sulfate (seizure prophylaxis) + IV
Labetalol
o ≥34 weeks of gestation
IOL after stabilizing the mother + Magnesium sulfate (seizure prophylaxis) + IV
Labetalol

Pregnant at 28 weeks of gestation, presents with generalized fatigue, systolic blood pressure
is 162, urine proteins +3, what is the next step?

A. MgSo4
B. Labetalol
C. Methyldopa

Correct Answer: A
Explanation:
Management of Preeclampsia with severe features
o <34 weeks of gestation:
Expectant management
Admission + Corticosteroid + Magnesium sulfate (seizure prophylaxis) + IV
Labetalol
o ≥34 weeks of gestation
IOL after stabilizing the mother + Magnesium sulfate (seizure prophylaxis) + IV
Labetalol
Note:
First thing to do is admission, if it wasn’t in the choices, I would go with magnesium to prevent
seizure first.

33 | P a g e
Obstetrics and Gynecology: SMLE Questions and Answers Chapter 1: Obstetrics

Primigravida at 32 weeks of gestation, came with mild headache with no abdominal pain,
or visual disturbance, Blood pressure 150/90, urine analysis +3 protien, appropriate
management?

A. Close outpatient after 1 week


B. Admission and observe
C. Induction of labor
D. C-section

Correct Answer: A
Explanation: (According to ACOG)
She has preeclampsia without severe features which is managed by outpatient close monitoring at
least once weekly.

A female pregnant with hypertension and proteinuria, she has right upper quadrant pain
what is the reason?

34 | P a g e
Obstetrics and Gynecology: SMLE Questions and Answers Chapter 1: Obstetrics

A. Distended Hepatic Capsule


B. Hepatic Rupture
C. Gall Bladder Stone

Correct Answer: A
Explanation: (According to ACOG)
Pain is thought to be due to periportal and focal parenchymal necrosis, hepatic cell edema, or
Glisson’s capsule distension, or a combination.

A pregnant woman came with abdominal pain and back pain and visual disturbance. Her
blood pressure is 145/92 mmHg. Her lab shows (uric acid high, platelet 70k). What is a
severe feature of preeclampsia for this condition?

A. Her abdominal pain


B. Her blood pressure
C. Platelet count
D. Uric acid

Correct Answer: C
Explanation:
The presence of one or more of the following indicates a diagnosis of "preeclampsia with
severe feature”:
Severe blood pressure Systolic blood pressure ≥160 mmHg or diastolic blood pressure ≥110 mmHg
elevation on 2 occasions at least 4 hours apart while the patient is on bedrest
Symptoms of central New-onset cerebral or visual disturbance, such as:
nervous system § Photopsia, scotomata, cortical blindness, retinal vasospasm
dysfunction § Severe headache (ie, incapacitating, "the worst headache I've ever
had") or headache that persists and progresses despite analgesic
therapy(acetaminophen) and not accounted for by alternative
diagnoses

Hepatic abnormality Impaired liver function not accounted for by another diagnosis and
characterized by serum transaminase concentration >2 times the upper limit
of the normal range or severe persistent right upper quadrant or epigastric
pain unresponsive to medication and not accounted for by an alternative
diagnosis, or both
Thrombocytopenia <100,000 platelets/microL
Renal abnormality Renal insufficiency (serum creatinine >1.1 mg/dL [97.2 micromol/L] or a
doubling of the serum creatinine concentration in the absence of other renal
disease)
Pulmonary edema The symptom complex of dyspnea, chest pain, and/or decreased (≤93 percent)
oxygen saturation

Let’s Exclude!
- Her abdominal pain à if it was unresponsive to analgesia (indicates severe preeclampsia)
- Her blood pressure à if it was ≥160/110 (indicates severe preeclampsia)
35 | P a g e
Obstetrics and Gynecology: SMLE Questions and Answers Chapter 1: Obstetrics
- Uric acid à Not one of the criteria of severe preeclampsia

Pregnant, now her B/P 140/90, platelets 90k, history of previously severe preeclampsia.
What indication here of severe preeclampsia?

A. B/P
B. Platelets
C. Urine concentration

Correct Answer: B
Explanation:
The presence of one or more of the following indicates a diagnosis of "preeclampsia with
severe feature”:
Severe blood pressure Systolic blood pressure ≥160 mmHg or diastolic blood pressure ≥110 mmHg
elevation on 2 occasions at least 4 hours apart while the patient is on bedrest
Symptoms of central New-onset cerebral or visual disturbance, such as:
nervous system § Photopsia, scotomata, cortical blindness, retinal vasospasm
dysfunction § Severe headache (ie, incapacitating, "the worst headache I've ever
had") or headache that persists and progresses despite analgesic
therapy(acetaminophen) and not accounted for by alternative
diagnoses

Hepatic abnormality Impaired liver function not accounted for by another diagnosis and
characterized by serum transaminase concentration >2 times the upper limit
of the normal range or severe persistent right upper quadrant or epigastric
pain unresponsive to medication and not accounted for by an alternative
diagnosis, or both
Thrombocytopenia <100,000 platelets/microL
Renal abnormality Renal insufficiency (serum creatinine >1.1 mg/dL [97.2 micromol/L] or a
doubling of the serum creatinine concentration in the absence of other renal
disease)
Pulmonary edema The symptom complex of dyspnea, chest pain, and/or decreased (≤93 percent)
oxygen saturation

Pregnant at 12 weeks, complaining of mild leg edema, blood pressure mildly elevated,
Positive trace proteinuria.

A. Methyldopa
B. Labetalol
C. Losartan
D. Captopril

Correct Answer: B
Explanation:
This is chronic hypertension which is treated by labetalol as a first line.

36 | P a g e
Obstetrics and Gynecology: SMLE Questions and Answers Chapter 1: Obstetrics
(Approximately 11% of women with chronic hypertension have proteinuria (more than 300
mg/day) at baseline because of hypertension- related nephrosclerosis or, less frequently,
undiagnosed chronic kidney disease)

Pregnant at 39 weeks of gestation, came with proteinuria and B/P 140/90, what is your
action?

A. Admit and observation


B. Labor induction
C. Follow up and observe

Correct Answer: B
Explanation:
Management of Preeclampsia without severe features
o <37 weeks of gestation:
Expectant management + oral labetalol or nifedipine
o ≥37 weeks of gestation
IOL + oral labetalol or nifedipine
o ≥34 weeks of gestation with pretem labor or PPROM
IOL + oral labetalol or nifedipine
Management of Preeclampsia with severe features
o <34 weeks of gestation:
Expectant management
Admission + Corticosteroid + Magnesium sulfate (seizure prophylaxis) + IV
Labetalol
o ≥34 weeks of gestation
IOL after stabilizing the mother + Magnesium sulfate (seizure prophylaxis) + IV
Labetalol

Pregnant at 33 weeks of gestation, presented with severe headache protein urine +3, B/P
150/100, what to do?

A. Immediate C-section
B. Delay for a week and give steroid.
C. Admitted for observation

Correct Answer: C
Explanation:
Management of Preeclampsia with severe features
o <34 weeks of gestation:
Expectant management
Admission + Corticosteroid + Magnesium sulfate (seizure prophylaxis) + IV
Labetalol
o ≥34 weeks of gestation

37 | P a g e
Obstetrics and Gynecology: SMLE Questions and Answers Chapter 1: Obstetrics
IOL after stabilizing the mother + Magnesium sulfate (seizure prophylaxis) + IV
Labetalol

Pregnant at 34 weeks of gestation with preeclampsia, high BP 170 /100 what to give?

A. Stabilize, MgSo4 and deliver


B. Stabilize, corticosteroids and deliver

Correct Answer: A
Explanation:
Management of Preeclampsia with severe features
o <34 weeks of gestation:
Expectant management
Admission + Corticosteroid + Magnesium sulfate (seizure prophylaxis) + IV
Labetalol
o ≥34 weeks of gestation
IOL after stabilizing the mother + Magnesium sulfate (seizure prophylaxis) + IV
Labetalol

Pregnant at 35 weeks of gestation, hypertensive came with sever headache, abdominal pain
and feel dizzy , what to do?

A. Give Mg salfate and admission for delivery


B. Give steroid and admission for delivery
C. Give Mg salfate and wait

Correct Answer: A
Explanation:
Management of Preeclampsia with severe features
o <34 weeks of gestation:
Expectant management
Admission + Corticosteroid + Magnesium sulfate (seizure prophylaxis) + IV
Labetalol
o ≥34 weeks of gestation
IOL after stabilizing the mother + Magnesium sulfate (seizure prophylaxis) + IV
Labetalol

Pregnant at 37 week with BP 160/110 every thing else was normal Her BP during
pregnancy was 120/90

A. Eclampsia
B. Gestational HTN

38 | P a g e
Obstetrics and Gynecology: SMLE Questions and Answers Chapter 1: Obstetrics
C. Pre-eclampsia

Correct Answer: C
Explanation:
The presence of one or more of the following indicates a diagnosis of "preeclampsia with
severe feature”:
Severe blood pressure Systolic blood pressure ≥160 mmHg or diastolic blood pressure ≥110 mmHg
elevation on 2 occasions at least 4 hours apart while the patient is on bedrest
Symptoms of central New-onset cerebral or visual disturbance, such as:
nervous system § Photopsia, scotomata, cortical blindness, retinal vasospasm
dysfunction § Severe headache (ie, incapacitating, "the worst headache I've ever
had") or headache that persists and progresses despite analgesic
therapy(acetaminophen) and not accounted for by alternative
diagnoses

Hepatic abnormality Impaired liver function not accounted for by another diagnosis and
characterized by serum transaminase concentration >2 times the upper limit
of the normal range or severe persistent right upper quadrant or epigastric
pain unresponsive to medication and not accounted for by an alternative
diagnosis, or both
Thrombocytopenia <100,000 platelets/microL
Renal abnormality Renal insufficiency (serum creatinine >1.1 mg/dL [97.2 micromol/L] or a
doubling of the serum creatinine concentration in the absence of other renal
disease)
Pulmonary edema The symptom complex of dyspnea, chest pain, and/or decreased (≤93 percent)
oxygen saturation

Pregnant at 34 weeks of gestation, complaining of vaginal bleeding, open cervix 6cm, she
has hypertension 160/90 and proteinuria, CTG shows fetal bradycardia, what is the
management?

A. MgSo4 and deliver


B. Stabilize and MgSo wait until 37 week
C. Stabilize give steroid then labour

Correct Answer: A
Explanation:
Management of Preeclampsia with severe features
o <34 weeks of gestation:
Expectant management
Admission + Corticosteroid + Magnesium sulfate (seizure prophylaxis) + IV
Labetalol
o ≥34 weeks of gestation
IOL after stabilizing the mother + Magnesium sulfate (seizure prophylaxis) + IV
Labetalol

39 | P a g e
Obstetrics and Gynecology: SMLE Questions and Answers Chapter 1: Obstetrics
Pregnant at 34 weeks of gestation, has preeclamsia patient came with epigastric pain,
headache, blurred vision, non stress test reassuring, what is the most appropriate
management?

A. MgSo4 and delivery


B. MgSo4 and wait till 37 weeks
C. Call anesthesiologist for delivery

Correct Answer: A
Explanation:
Management of Preeclampsia with severe features
o <34 weeks of gestation:
Expectant management
Admission + Corticosteroid + Magnesium sulfate (seizure prophylaxis) + IV
Labetalol
o ≥34 weeks of gestation
IOL after stabilizing the mother + Magnesium sulfate (seizure prophylaxis) + IV
Labetalol

Magnisium sulfate given to hypertensive pregnant to?

A. Prevent the seizure attack


B. For fetal neuroprotection
C. Reduce her BP

Correct Answer: A
Explanation: (According to Uptodate)
Most eclamptic seizures are self-limited. Magnesium sulfate is not necessary to arrest the seizure
but to prevent recurrent convulsions.

Pregnant with preeclampsia at 36 weeks, developed seizure. After stabilization you should
give Mg Sulfate to?

A. Manage her seizure


B. Prevent the seizure attack
C. For fetal neuroprotection

Correct Answer: B
Explanation: (According to Uptodate)
Most eclamptic seizures are self-limited. Magnesium sulfate is not necessary to arrest the seizure
but to prevent recurrent convulsions.

40 | P a g e
Obstetrics and Gynecology: SMLE Questions and Answers Chapter 1: Obstetrics
Preganat complaining of severe abdominal pain and uterine cotractions she was given 6mg
Mg sulfate and the contractions become normal, then decrease the dose to 4mg, then she
complained of shortness of breath. What to do?

A. Give her oxygen


B. Change to left lateral position
C. Stop Mg sulfate and give Ca gluconate

Correct Answer: C
Explanation:
- Magnesium toxicity à Deep tendon reflexes are lost, respiratory depression, and cardiac
arrest
- When MgSo4 toxicity occurs à The infusion should be stopped emergency correction
with calcium gluconate

Pregnant with seizure given 6 mg sulphate then decrease to 4mg. On Examination there is
absent deep tendon reflex, what to do?

A. Reassurance
B. Re-increase Mg sulfate dose
C. Stop Mg sulfate and give Ca gluconate

Correct Answer: C
Explanation:
- Magnesium toxicity à Deep tendon reflexes are lost, respiratory depression, and cardiac
arrest
- When MgSo4 toxicity occurs à The infusion should be stopped emergency correction
with calcium gluconate

What is the medication for seizure in pregnancy?

A. Diazepine
B. Phenytoin
C. Magnesium sulfate
D. Lorazepam

Correct Answer: C
Explanation:
Diazepine and phenytoin are justified only in the context of antiepileptic treatment or when
magnesium sulfate is contraindicated or unavailable (myasthenia gravis, hypocalcemia, moderate-
to-severe renal failure, cardiac ischemia, heart block, or myocarditis).

Risk factor for preeclampsia?

41 | P a g e
Obstetrics and Gynecology: SMLE Questions and Answers Chapter 1: Obstetrics

A. Maternal age
B. Gestational age
C. Multiple pregnancy

Correct Answer: C
Explanation: (According to ACOG)
Risk factors for Preeclampsia
⁃ Nulliparity
⁃ Multifetal gestations
⁃ Preeclampsia in a previous pregnancy
⁃ Chronic hypertension
⁃ Pregestational diabetes
⁃ Gestational diabetes
⁃ Thrombophilia
⁃ Systemic lupus erythematosus
⁃ Prepregnancy body mass index greater than 30
⁃ Antiphospholipid antibody syndrome
⁃ Maternal age ≥35 years or <18 years
⁃ Kidney disease
⁃ Assisted reproductive technology (IVF)
⁃ Obstructive sleep apnea
⁃ Obesity (BMI≥30)
⁃ Hydatidiform mole
⁃ Family history of preeclampsia

Preeclampsia what expected to be significantly decrease ?

A. Urea
B. Creatinine
C. Plasma volume
D. Platelets

Correct Answer: D
Explanation:
Let’s Exclude!
- Uric acid à Serum uric acid increases with preeclampsia
- Creatinine à it may increase with preeclampsia not decrease
- Plasma volume à it increases in pregnancy in general
- Thrombocytopenia à may occur and may reach severe levels as part of HELLP

Primigravida at 32 weeks of gestation with BP of 150/90 mmHg. There is edema of hands


and legs. What to do?

42 | P a g e
Obstetrics and Gynecology: SMLE Questions and Answers Chapter 1: Obstetrics
A. Diuretics
B. Tabs Labetalol
C. Continued evaluation

Correct Answer: C
Explanation:
The main goals in the initial evaluation of pregnant women with newly developed
hypertension are to distinguish gestational hypertension from preeclampsia.

I would evaluate her more first with urinary protein excretion and look for features of severe
disease, then I will manage accordingly.

Patient with preeclampsia, what will you order?

A. Liver function test LFT


B. Platelet
C. Urinalysis

Correct Answer: C
Explanation:
First thing is to do urine analysis to distinguish gestational hypertension from preeclampsia

Which of these causes IUGR?

A. Oligohydramnios
B. Polyhydramnios
C. Preeclampsia

Correct Answer: C
Explanation: (According to ACOG)
Among women with preeclampsia, clinical manifestations that follow from this
uteroplacental ischemia include:
- Fetal growth restriction(IUGR)
- Oligohydramnios
- Placental abruption
- Nonreassuring fetal status
- Increased risk of spontaneous or indicated preterm delivery.

Women with HTN what do you expect the complications?

A. IUGR
B. Abnormal placental lining

43 | P a g e
Obstetrics and Gynecology: SMLE Questions and Answers Chapter 1: Obstetrics
Correct Answer: A
Explanation: (According to ACOG)
Among women with preeclampsia, clinical manifestations that follow from this
uteroplacental ischemia include:
- Fetal growth restriction(IUGR)
- Oligohydramnios
- Placental abruption
- Nonreassuring fetal status
- Increased risk of spontaneous or indicated preterm delivery.

Patient hypertensive with proteinuria on labetalol she’s 32 weeks, fundal height 28. What
will commonly occur with IUGR?

A. Oligohydramnios
B. Polyhydramnios

Correct Answer: A
Explanation: (According to ACOG)
Among women with preeclampsia, clinical manifestations that follow from this
uteroplacental ischemia include:
- Fetal growth restriction(IUGR)
- Oligohydramnios
- Placental abruption
- Nonreassuring fetal status
- Increased risk of spontaneous or indicated preterm delivery.

Typical case of preeclampsia, asking about which of the following clinical findings with this
condition?

A. Elevated mother liver enzymes


B. Oligohydramnios
C. Polyhydramnios

Correct Answer: B
Explanation: (According to ACOG)
Among women with preeclampsia, clinical manifestations that follow from this
uteroplacental ischemia include:
- Fetal growth restriction (IUGR)
- Oligohydramnios
- Placental abruption
- Nonreassuring fetal status
- Increased risk of spontaneous or indicated preterm delivery.

Note:
44 | P a g e
Obstetrics and Gynecology: SMLE Questions and Answers Chapter 1: Obstetrics
I would go with elevated liver enzymes if the question asked about pre-eclampsia with severe
features.

Patient had history of preeclampsia in her previous pregnancy. What of the following
decreases the risk of preeclampsia?

A. Antibiotic
B. Aspirin
C. MgSo4

Correct Answer: B
Explanation: (According to ACOG)
Women with any of the high-risk factors for preeclampsia (previous pregnancy with
preeclampsia, multifetal gestation, renal disease, autoimmune disease, type 1 or type 2 diabetes
mellitus, and chronic hypertension) and those with more than one of the moderate-risk factors
(first pregnancy, maternal age of 35 years or older, a body mass index BMI of more than 30, family
history of preeclampsia, sociodemographic characteristics, and personal history factors) should
receive low-dose (81 mg/day) aspirin for preeclampsia prophylaxis initiated between 12
weeks and 28 weeks of gestation (optimally before 16 weeks of gestation) and continuing until
delivery

What of the following decreases the risk of eclampsia?

A. Antibiotic
B. Aspirin
C. MgSo4

Correct Answer: C
Explanation: (According to ACOG)
Magnesium sulfate is used to prevent seizures in women with preeclampsia with severe features
and eclampsia

Note:
- Prevention of preeclampsia àAspirin
- Prevention of eclampsia à MgSo4

A women known chronic hypertension came to prenatal care counseling with expected
pregnancy, she is on hydrochlorothiazide and lisinopril?

A. Stop both
B. Stop lisinopril and start methyldopa
C. Continue both
D. Stop ACEI and continue thiazide

45 | P a g e
Obstetrics and Gynecology: SMLE Questions and Answers Chapter 1: Obstetrics
Correct Answer: A
Explanation: (According to Williams Obstetrics)
- For most women with mild to moderate hypertension, the College recommends that
treatment be withheld as long as systolic blood pressure is < 160 mm Hg and diastolic
blood pressure is < 105 mm Hg.
- It is controversial whether or not women who present early in pregnancy and who are
already taking antihypertensive drugs should continue to take these
- According to the American College of Obstetricians and Gynecologists (2013) and the
Society for Maternal-Fetal Medicine (2015), for women with mild to moderate
hypertension, it is reasonable to discontinue medications during the first trimester
and to restart them if blood pressures approach the severe range.

The role of antihypertensive in pre-eclampsia?

A. To prevent maternal complication as stroke


B. To prevent IUGR
C. To prevent fetus demise

Correct Answer: A
Explanation:
According to ACOG:
⁃ The objectives of treating severe hypertension are to prevent congestive heart failure,
myocardial ischemia, renal injury or failure, and ischemic or hemorrhagic stroke.

According to Williams Obstetrics:


⁃ The use of antihypertensive drugs to prolong pregnancy or modify perinatal outcomes in
pregnancies complicated by various hypertensive disorders has been of considerable
interest

⁃ Drug treatment for early mild preeclampsia has been disappointing. Sibai and colleagues
(1987a) reported that women given labetalol had significantly lower mean blood pressures.
However, mean pregnancy prolongation, gestational age at delivery, and birthweight did
not differ between groups. The cesarean delivery rate and the number of newborns admitted
to special-care nurseries were also similar. The frequency of growth-restricted neonates was
doubled in women given labetalol-19 versus 9percent.

According to UTD
⁃ Severe hypertension in labor should be treated promptly with intravenous labetalol (avoid
in patients with asthma) or hydralazine or, less commonly, oral nifedipine to prevent stroke

What is the rational of antihypertensive medication in preeclampsia?

A. Decrease IUGR
B. Decrease mother mortality

46 | P a g e
Obstetrics and Gynecology: SMLE Questions and Answers Chapter 1: Obstetrics
C. Decrease fetal mortality

Correct Answer: B
Explanation:
According to ACOG:
⁃ The objectives of treating severe hypertension are to prevent congestive heart failure,
myocardial ischemia, renal injury or failure, and ischemic or hemorrhagic stroke.

Patient at 34 weeks of gestation with eclampsia. She was managed and stabilized. what is
the most appropriate next step?

A. Observation
B. Wait till week 37 and deliver
C. Prepare for delivery
D. Discharge home

Correct Answer: C
Explanation: (According to ACOG and Uptodate)
Eclampsia
Severe form of preeclampsia with convulsive seizures and/or coma (in the absence of other
neurologic conditions that could account for the seizure)
Management:
⁃ ABC: calling for help, prevention of maternal injury, placement in lateral decubitus position,
prevention of aspiration, administration of oxygen, and monitoring vital signs including
oxygen saturation.
⁃ Anticonvulsive Therapy:
• Magnesium sulfate: (Most eclamptic seizures are self-limited. Magnesium sulfate
is not necessary to arrest the seizure but to prevent recurrent convulsions.)
Dose: [IV] administration of a 4–6 g loading dose over 20–30 minutes, followed by a
maintenance dose of 1–2 g/hour
Side effects: Deep tendon reflexes are lost, warmth and flushing, respiratory
depression, and cardiac arrest
Monitoring: respiration status, tendon reflexes and measuring urine output (because
magnesium sulfate is excreted almost exclusively in the urine)
When toxicity occurs? The infusion should be stopped emergency correction with
calcium gluconate 10% solution, 10 mL IV over 3 minutes, along with furosemide
intravenously to accelerate the rate of urinary excretion.
• Diazepine and phenytoin are justified only in the context of antiepileptic
treatment or when magnesium sulfate is contraindicated or unavailable
(myasthenia gravis, hypocalcemia, moderate-to-severe renal failure, cardiac
ischemia, heart block, or myocarditis).

- Delivery! after maternal hemodynamic stabilization

47 | P a g e
Obstetrics and Gynecology: SMLE Questions and Answers Chapter 1: Obstetrics

Patient at 32 weeks presented with seizure and high blood pressure, what is the next
appropriate step in management?

A. Steroid
B. Hydralazine
C. Magnesium sulfate
D. Antibiotics

Correct Answer: C
Explanation: (According to ACOG and Uptodate)
Eclampsia
Severe form of preeclampsia with convulsive seizures and/or coma (in the absence of other
neurologic conditions that could account for the seizure)
Management:
⁃ ABC: calling for help, prevention of maternal injury, placement in lateral decubitus
position, prevention of aspiration, administration of oxygen, and monitoring vital signs
including oxygen saturation.
⁃ Anticonvulsive Therapy:
• Magnesium sulfate: (Most eclamptic seizures are self-limited. Magnesium
sulfate is not necessary to arrest the seizure but to prevent recurrent
convulsions.)
Dose: [IV] administration of a 4–6 g loading dose over 20–30 minutes, followed by
a maintenance dose of 1–2 g/hour
Side effects: Deep tendon reflexes are lost, warmth and flushing, respiratory
depression, and cardiac arrest
Monitoring: respiration status, tendon reflexes and measuring urine output (because
magnesium sulfate is excreted almost exclusively in the urine)
vWhen toxicity occurs? The infusion should be stopped emergency correction with
calcium gluconate 10% solution, 10 mL IV over 3 minutes, along with furosemide
intravenously to accelerate the rate of urinary excretion.
• Diazepine and phenytoin are justified only in the context of antiepileptic
treatment or when magnesium sulfate is contraindicated or unavailable
(myasthenia gravis, hypocalcemia, moderate-to-severe renal failure, cardiac
ischemia, heart block, or myocarditis).

- Delivery! after maternal hemodynamic stabilization

Preganat at 38 weeks of gestation with preeclampsia B/P 160/90 with HELP syndrome,
elevated liver enzymes and peripheral blood smear show halment cells, what is the most
appropriate management?

48 | P a g e
Obstetrics and Gynecology: SMLE Questions and Answers Chapter 1: Obstetrics

A. Plasma exchange
B. Urgent delivery

Correct Answer: B
Explanation: (According to ACOG and Uptodate)
HELLP Syndrome
A life-threatening form of preeclampsia characterized by Hemolysis, Elevated Liver enzymes,
and Low Platelets
- To make the diagnosis: (Tennessee classification):
o Hemolysis, established by at least two of the following:
• Peripheral smear with schistocytes and burr cells
• Serum bilirubin ≥1.2 mg/dL (20.52 micromol/L)
• Low serum haptoglobin (≤25 mg/dL) or lactate dehydrogenase (LDH) ≥2 times
the upper level of normal (based on laboratory-specific reference ranges)
• Severe anemia, unrelated to blood loss

o Elevated liver enzymes:


• Aspartate aminotransferase (AST) or alanine aminotransferase (ALT) ≥2 times
the upper level of normal (based on laboratory-specific reference ranges)

o Low platelets: <100,000 cells/microL

- Clinical presentation:
o Right upper quadrant pain and generalized malaise (90%)
o Nausea and vomiting (50%)
- Management
o Women with HELLP syndrome should be delivered regardless of their gestational
age

Note:
Therapeutic plasma exchange has no benefit in patients with HELLP

Patient with pre-eclampsia, what will decrease?

A. Plasma uric acid


B. Plasma creatinine
C. Plasma volume
D. Serum urea

Correct Answer: C
Explanation: (According to Williams Obstetrics)
Pathophysiology of hypertensive disorders in pregnancy
- Blood Volume
Hemoconcentration is a hallmark of eclampsia.

49 | P a g e
Obstetrics and Gynecology: SMLE Questions and Answers Chapter 1: Obstetrics
Women of average size have a blood volume of 3000 mL, and during the last several weeks
of a normal pregnancy, this averages 4500 mL. With eclampsia, however, much or all of
the anticipated 1500 mL excess is lost. Such hemoconcentration results from generalized
vasospasm that follows endothelial activation and leakage of plasma into the interstitial
space.

Pregnant at 38 weeks of gestation, presented with headache and high BP. Labs show Hb
120, low platelets, high liver enzymes. What is the most appropriate management?
A. Immediate delivery
B. MgSo4

Correct Answer: A
Explanation:
Management of Preeclampsia with severe features
o <34 weeks of gestation:
Expectant management
Admission + Corticosteroid + Magnesium sulfate (seizure prophylaxis) + IV
Labetalol
o ≥34 weeks of gestation
IOL after stabilizing the mother + Magnesium sulfate (seizure prophylaxis) + IV
Labetalol

Note:
Most appropriate à Delivery
Next? à Administer MgSo4

Normal pregnant women at 12 weeks, Indication of severe preeclampsia?

A. Increase creatinine
B. Increase urea
C. Increase NA

Correct Answer: A
Explanation:
The presence of one or more of the following indicates a diagnosis of "preeclampsia with
severe feature”:
Severe blood pressure Systolic blood pressure ≥160 mmHg or diastolic blood pressure ≥110 mmHg
elevation on 2 occasions at least 4 hours apart while the patient is on bedrest
Symptoms of central New-onset cerebral or visual disturbance, such as:
nervous system § Photopsia, scotomata, cortical blindness, retinal vasospasm
dysfunction § Severe headache (ie, incapacitating, "the worst headache I've ever
had") or headache that persists and progresses despite analgesic

50 | P a g e
Obstetrics and Gynecology: SMLE Questions and Answers Chapter 1: Obstetrics
therapy(acetaminophen) and not accounted for by alternative
diagnoses

Hepatic abnormality Impaired liver function not accounted for by another diagnosis and
characterized by serum transaminase concentration >2 times the upper limit
of the normal range or severe persistent right upper quadrant or epigastric
pain unresponsive to medication and not accounted for by an alternative
diagnosis, or both
Thrombocytopenia <100,000 platelets/microL
Renal abnormality Renal insufficiency (serum creatinine >1.1 mg/dL [97.2 micromol/L] or a
doubling of the serum creatinine concentration in the absence of other renal
disease)
Pulmonary edema The symptom complex of dyspnea, chest pain, and/or decreased (≤93 percent)
oxygen saturation

Patient was refered from another hospital and came with headache, BP: 160/100 , high
AST and ALT. What is the appropriate management?

A. Immediate C-section
B. Induction of labor

Correct Answer: B
Explanation:
Management of Preeclampsia with severe features
o <34 weeks of gestation:
Expectant management
Admission + Corticosteroid + Magnesium sulfate (seizure prophylaxis) + IV
Labetalol
o ≥34 weeks of gestation
IOL after stabilizing the mother + Magnesium sulfate (seizure prophylaxis) + IV
Labetalol

51 | P a g e
Obstetrics and Gynecology: SMLE Questions and Answers Chapter 1: Obstetrics

Diabetes Mellitus and Gestational Diabetes Mellitus

Patient with long term history of DM type 1 in 12 weeks of gestation. HbA1C 12. Which of
the following complication is most likely to happen?

A. Pre-eclampsia
B. Polyhydramnios
C. Congenital malformation
D. IUGR

Correct Answer: C
Explanation: (According to ACOG)
- In the second and third trimesters, an HbA1C less than 6% has the lowest risk of large-for-
gestational-age infants. Importantly, because of the association of elevated glucose values
and congenital anomalies, aggressive approaches to glycemic control early in the first
trimester before or during embryogenesis may reduce the risk of fetal anomalies.

- Women with true GDM are not at increased risk of having an infant with congenital
malformations because the onset of the disorder is after organogenesis, and they do not
experience diabetes-related vasculopathy because of the short duration of the disorder.

Another recall

Patient with long term history of DM type 1 in 12 weeks of gestation. HbA1C 12. Which of
the following complication is most likely to happen?

A. Pre-eclampsia
B. Polyhydramnios
C. Oligohydroamnios
D. IUGR

Correct Answer: A
Explanation: (According to Williams Obstetrics)

52 | P a g e
Obstetrics and Gynecology: SMLE Questions and Answers Chapter 1: Obstetrics

Gestational diabetes most associated with?

A. Preeclampsia
B. Polyhydramnios
C. IUGR

Correct Answer: A
Explanation:
- Women with GDM have a higher risk of developing preeclampsia
- GDM complications most commonly is macrosomia à then HTN and preeclampsia

Another recall

What is the most common complication of GDM?

A. Congenital anomalies
B. Macrosomia
C. Shoulder dystocia
D. Hypoglycemia

Correct Answer: B
Explanation:
- GDM complications most commonly is macrosomia

Patient diabetic and pregnant what's the most useful test that will determine prognosis for
the baby?

A. OGTT
B. Fasting blood glucose
C. Glycosylated hemoglobin
D. US nuchal translucency

Correct Answer: C

53 | P a g e
Obstetrics and Gynecology: SMLE Questions and Answers Chapter 1: Obstetrics
Explanation: (According to ACOG)
Glycosylated hemoglobin levels correlate directly with the frequency of anomalies.

Diabetic mother came at 10 weeks what test will tell you the risk of chromosomal
anomalies?

A. Glycosylated hemoglobin
B. US nuchal translucency

Correct Answer: B
Explanation:
Nuchal translucency scan is a prenatal screening scan to detect chromosomal abnormalities in a
fetus. Done by ultrasound between 10+3 and 13+6 gestational weeks.

Patient 36 years old, diabetic on metformin, delivered a baby with congenital anomalies,
she wants to conceive now, what is the most appropriate action?

A. Switch to insulin
B. Genetic counseling
C. Folic acid 400 mcg

Correct Answer: A
Explanation:
According to UTD:
Genetic counseling referral is indicated if the personal or family history includes a confirmed
clinical diagnosis with a known genetic etiology such as hemophilia, neurofibromatosis, or Marfan
syndrome

According to ACOG:
At least 400 micrograms of folic acid should be prescribed to all women contemplating pregnancy.
This is particularly important in women with diabetes given their increased risk of neural tube
defects. Higher doses of folic acid may be beneficial in high-risk patients and doses of 800
micrograms or 1 mg have been prescribed, especially in the presence of other risk factors for neural
tube defects.

Despite increasing data on metformin’s use in pregnancy, there is limited long-term safety
information on the infants of these women. The use of all oral hypoglycemic agents for control of
pregestational type 2 diabetes mellitus during pregnancy should be limited and individualized until
data regarding the safety and efficacy of these drugs become available. Thus, insulin is the
preferred treatment for pregestational diabetes in pregnancy not controlled by diet and exercise.
Women with type 2 diabetes who are stable on oral agents before pregnancy and become pregnant
should be counseled that insulin is the preferred therapy in pregnancy and that oral antidiabetic
medications are not approved by the U.S. FDA for treatment of diabetes during pregnancy because
they cross the placenta and lack long-term neonatal safety data.
54 | P a g e
Obstetrics and Gynecology: SMLE Questions and Answers Chapter 1: Obstetrics
For those women with type 2 diabetes who decline insulin, those who their obstetricians or
obstetric care providers believe will be unable to safely administer insulin, or those who cannot
afford insulin, metformin (and rarely glyburide) is a reasonable alternative choice in the context of
discussing with the patient the limitations of the safety data and a high rate of treatment failure,
which requires insulin supplementation

According to UTD:
Folic acid supplementation in pregnancy

Congenital malformations other than NTDs


We suggest a higher dose (1 mg per day) of periconceptional/first-trimester folic
acid supplementation for women with a history of one of the following malformations in
themselves or their partner, a prior offspring, or a first- or second-degree relative:
●Cleft lip/palate
●Congenital heart defects
●Limb reduction defects
●Urinary tract defects
●Congenital hydrocephalus

According To American Diabetes Association (ADA)


Recommendations for preconceptional care for diabetic women
- Oral medications for treatment of type 2 diabetes should be stopped and insulin started and
titrated to achieve acceptable glucose control before conception.

Note:
In conclusion she has a previous history of congenital anomalies, her folic acid dose should be at
least 800 micrograms.
So the correct answer would be switch to insulin

Female hypertensive and diabetic, on ACEI, insulin, Metformin, she decided to get
pregnant soon. Labs: heavy proteinuria, HbA1c: 8, What’s your most appropriate advice
for her regarding diabetic control and fetal congenital malformations?

A. Stop snacks and start 3 heavy meals a day.


B. Switch ACEI to ARB’s

55 | P a g e
Obstetrics and Gynecology: SMLE Questions and Answers Chapter 1: Obstetrics
C. Try to control her hga1c to normal or near normal as possible before pregnancy.
D. Increase dose of ACEI

Correct Answer: C
Explanation: (According to ACOG)
Prepregnancy counseling should focus on the importance of euglycemic control before pregnancy,
as well as the adverse obstetric and maternal outcomes that can result from poorly controlled
diabetes. The specific risk of fetal embryopathy related to glycemic control is an important
outcome to counsel patients about in order to emphasize the importance of prepregnancy glycemic
control (HbA1c less than 6%).

Pregnant and diabetes and hypoglycemic in ER and has to given glucose what the route
that will give her and don’t harm the baby?

A. Nasogastric
B. Orogastric
C. Peripheral venous
D. Central venous

Correct Answer: C
Explanation: (According to ACOG)
If patients become hypoglycemic, intravenous dextrose should be given and the insulin infusion
rate reduced.

At what gestational age we start diabetes screening in a pregnant woman?

A. 4 weeks
B. 12 weeks
C. 24 weeks
D. 34 weeks

Correct Answer: C
Explanation: (According to ACOG)
U.S. Preventive Services Task Force made a recommendation to screen all pregnant women for
GDM at or beyond 24 weeks of gestation.

Pregnant at 22 weeks gestational age oral glucose challenge test after one hour: high, after
2 hours: high, after three hours: high. What is next?

A. Repeat same test


B. HgA1c
C. Fasting blood glucose
D. Random blood glucose

56 | P a g e
Obstetrics and Gynecology: SMLE Questions and Answers Chapter 1: Obstetrics

Correct Answer: B
Explanation: (According to ACOG)
According to ACOG:
American Diabetes Association (ADA) has noted that measurement of hemoglobin A1C also can
be used for detecting pregestational diabetes or early GDM, but it may not be suitable for use alone
because of decreased sensitivity compared with OGTT approaches.

Pregnant with uncontrolled diabetes, what is the most likely complication to happen to the
baby after delivery?

A. Respiratory distress syndrome RDS


B. Hyperkalemia

Correct Answer: A
Explanation: (According to ACOG)
The neonatal consequences of poorly controlled pregestational diabetes mellitus during
pregnancy include profound hypoglycemia, a higher rate of respiratory distress syndrome,
polycythemia, organomegaly, electrolyte disturbances, and hyperbilirubinemia.

Pregnant lady with one abnormal GTT reading after 1 hour, what is the most appropriate
management?

A. Repeat GTT
B. Lifestyle modification
C. Measure HbA1c

Correct Answer: B
Explanation: (According to ACOG)
- A one-step approach to establishing the diagnosis of GDM using a 75-g, 2-hour OGTT
has been used and promoted by other organizations. For example, in 2010, the International
Association of Diabetes and Pregnancy Study Group (IADPSG) recommended that a
universal 75-g, 2-hour OGTT be performed during pregnancy and that the diagnosis of
GDM be established when any single threshold value was met or exceeded (fasting value,
92 mg/dL; 1-hour value, 180 mg/dL; or 2-hour value, 153 mg/dL)

- Women in whom GDM is diagnosed should receive nutrition and exercise counseling, and
when this fails to adequately control glucose levels, medication should be used for maternal
and fetal benefit.
- When pharmacologic treatment of GDM is indicated, insulin is considered the preferred
treatment for diabetes in pregnancy.
- In women who decline insulin therapy or who the obstetricians or other obstetric care
providers believe will be unable to safely administer insulin, or for women who cannot
afford insulin, metformin is a reasonable alternative choice.

57 | P a g e
Obstetrics and Gynecology: SMLE Questions and Answers Chapter 1: Obstetrics

What is the pathophysiology of GDM?

A. Decrease uteroplacental blood flow during late pregnancy


B. Human placental lactogen (human chorionic somatomammotropin)

Correct Answer: B
Explanation: (According to ACOG and Williams Obstetrics)
- The increase in insulin resistance is primarily the result of the effects of several placental
hormones, including human chorionic somatomammotropin (human placental lactogen),
progesterone, prolactin, placental growth hormone, and cortisol. Additionally, tumor
necrosis factor a, and leptin have been implicated as contributors to the insulin resistant
state of pregnancy and resultant maternal hyperglycemia

- Pregnancy-induced state of peripheral insulin resistance, which ensures a sustained


postprandial supply of glucose to the fetus. The mechanisms responsible for this reduced
insulin sensitivity include numerous endocrine and inlammatory factors. In particular,
pregnancy-related hormones such as progesterone, placentally derived growth hormone,
prolactin, and cortisol; cytokines such as tumor necrosis factor

Gestational diabetes is?

A. Normal pregnancy associated with diabetogenic condition due to estrogen


B. Normal pregnancy associated with diabetogenic condition due to FSH
C. Normal pregnancy associated with diabetogenic condition due to lactogen

Correct Answer: C
Explanation: (According to ACOG and Williams Obstetrics)
- The increase in insulin resistance is primarily the result of the effects of several placental
hormones, including human chorionic somatomammotropin (human placental lactogen),
progesterone, prolactin, placental growth hormone, and cortisol. Additionally, tumor
necrosis factor a, and leptin have been implicated as contributors to the insulin resistant
state of pregnancy and resultant maternal hyperglycemia

- Pregnancy-induced state of peripheral insulin resistance, which ensures a sustained


postprandial supply of glucose to the fetus.

The mechanisms responsible for this reduced insulin sensitivity include numerous
endocrine and inlammatory factors. In particular, pregnancy-related hormones such as
progesterone, placentally derived growth hormone, prolactin, and cortisol; cytokines such
as tumor necrosis factor

58 | P a g e
Obstetrics and Gynecology: SMLE Questions and Answers Chapter 1: Obstetrics
Pregnant patient known to have DM, what is the complication she has a similar chance of
getting it as in normal pregnancy?

A. Preeclampsia
B. Cystic fibrosis
C. IUGR
D. Polyhydramnios

Correct Answer: B
Explanation: (According to ACOG)
- Women with GDM have a higher risk of developing preeclampsia.
- Neonates delivered to women with pregestational diabetes are at increased risk of
macrosomia and, depending on concomitant risk factors, also may be at increased risk of
fetal growth restriction.
- Women with pregestational diabetes mellitus have a greater risk of a wide range of
obstetric complications. For these women, the rate of primary cesarean delivery is
increased; spontaneous preterm labor appears to be more common; and for some women—
particularly those with poor glycemic control the increased incidence of
polyhydramnios.

Newly diagnosed mother with GDM, what is the first line in management?

A. Diet
B. Metformin
C. Oral Insulin
D. SC insulin

Correct Answer: A
Explanation: (According to ACOG)
- Women in whom GDM is diagnosed should receive nutrition and exercise counseling, and
when this fails to adequately control glucose levels, medication should be used for maternal
and fetal benefit.
- When pharmacologic treatment of GDM is indicated, insulin is considered the preferred
treatment for diabetes in pregnancy.
- In women who decline insulin therapy or who the obstetricians or other obstetric care
providers believe will be unable to safely administer insulin, or for women who cannot
afford insulin, metformin is a reasonable alternative choice.

Best test to check the glycemic control during pregnancy in the routine visit in pregnant
with DM?

A. Random blood glucose


B. Fasting blood glucose
C. HbA1c

59 | P a g e
Obstetrics and Gynecology: SMLE Questions and Answers Chapter 1: Obstetrics

Correct Answer: C
Explanation: (According to Uptodate)
ASSESSING GLYCEMIC CONTROL
Assessment of glucose levels during pregnancy may involve blood glucose monitoring (BGM)
with fingersticks in combination with periodic measurement of glycated hemoglobin (also called
A1C, hemoglobin A1C, glycohemoglobin, or HbA1C).

60 | P a g e
Obstetrics and Gynecology: SMLE Questions and Answers Chapter 1: Obstetrics

Pregnancy Related Medical and Surgical Conditions

Female patient wants to get pregnant. TSH high, T4 normal. What is the most appropriate
management?

A. Follow up
B. Give thyroxine now
C. Give thyroxine during pregnancy
D. Proceed to pregnancy without any management

Correct Answer: B

Pregnant women with brown spot in her face?

A. Melasma
B. Chloasma

Correct Answer: B
Explanation:
Cholasma: is the occurrence of melasma during pregnancy

Pregnant, complaining of vomiting multiple times, dry mouth and oral thrush, decreased skin
turgor, urinalysis ordered, what finding in UA will confirm the diagnosis?

A. Urine leukocyte
B. Urine Proteins
C. Urine Ketones
D. Urine Glucose

Correct Answer: C
Explanation:
Hyperemesis gravidarum à Urine ketones

Physiological changes in pregnancy that can lead to heart failure in patient with mitral
stenosis?

A. Increase minute ventilation


B. Increase RBC mass
C. Increase renal plasma flow
61 | P a g e
Obstetrics and Gynecology: SMLE Questions and Answers Chapter 1: Obstetrics
D. Increase plasma volume

Correct Answer: D
Explanation:
According to hacker: During pregnancy, the mechanical obstruction associated with mitral
stenosis worsens as cardiac output increases.

According to UTD: In MS, the stenotic mitral valve restricts diastolic left ventricular filling,
resulting in an elevated transmitral gradient and left atrial pressure that are further increased by the
physiologic hypervolemia and increased heart rate during pregnancy, thereby increasing the risk of
pulmonary congestion or pulmonary edema

IN SUMMARY: mitral stenosis worsens with increased cardiac output! So what increases the
cardiac output?
- Increased HR
- Increased blood volume (plasma volume+ packed RBC’s)
But! Plasma volume increases way more than the packed RBC’s!! which causes physiological
anemia..
So, I would go with increased plasma volume. If plasma volume is not in the choices? Go for
Increased RBC

A pregnant woman presents with vague pain. US showing an ovarian cyst measuring 9 x 7 cm.
How will you manage this patient?

A. Reassurance
B. Laparoscopic drainage
C. Immediate laparotomy
D. Analgesia and Observation

Correct Answer: A
Explanation: (According to ACOG)
Adnexal Mass in Pregnancy
Most adnexal masses in pregnancy appear to have a low risk of malignancy or acute complications
and may be managed expectantly. 51–92% of adnexal masses will resolve during pregnancy
Surgical intervention is warranted for:
⁃ Symptomatic masses (acute abdomen) “OUR PATIENT HAS NO ACTUE ABDOMEN”
⁃ Or for suspected malignancy based on the results of radiologic imaging (US finding suggest
malignancy: cyst greater than 10cm) “OUR PATIENT IS LESS THAN THAT”, serum
marker testing, or both.

Let’s Exclude!!
⁃ Reassurance à because it will resolve during pregnancy and no need for follow up!

62 | P a g e
Obstetrics and Gynecology: SMLE Questions and Answers Chapter 1: Obstetrics
⁃ Laparoscopic drainage à if more than 10cm
⁃ Immediate laparotomy àif ACUTE ABODMEN (sudden abdominal pain and tenderness
with nausea and vomiting), although exploratory laparoscopy is preferred
⁃ Analgesia and observation à why would I observe her? What is my indication of observing
her?

Sickle cell anemia patient pregnant, antenatal complication associated with her condition?

A. Low birth weight (IUGR)


B. Chest infection
C. UTI

Correct Answer: A
Explanation:
Williams Obstetrics:

ACOG:

Pregnant Hepatitis Bs positive what will you give the baby in first 12 hrs?

A. Hep b vaccine + immunoglobulins


B. Hep b vaccine only
C. Immunoglobulins only

Correct Answer: A
Explanation: (According to ACOG)
Hepatitis B and pregnancy
- Newborn immunization:

63 | P a g e
Obstetrics and Gynecology: SMLE Questions and Answers Chapter 1: Obstetrics
• Newborns of mothers who test positive for HBsAg should receive passive-active
immunization, with the first dose of the hepatitis B vaccine series and one dose of
HBIG administered withing 12 hours of delivery.

64 | P a g e
Obstetrics and Gynecology: SMLE Questions and Answers Chapter 1: Obstetrics

Antepartum Hemorrhage (APH)

A pregnant woman at 32 weeks gestation presents with severe abdominal pain. she denies
any abdominal bleeding. She has CRL of 34 weeks. Examination reveals a tender and tense
uterus. what is the most appropriate next step?
A. Perform an US
B. Cesarean section
C. Check CTG

Correct Answer: A
Explanation: (According to Uptodate)
The first thing to do in antepartum hemorrhage is US to exclude placenta previa. Then CTG.

A pregnant lady complains of vaginal bleeding, shes a confirmed case of low lying placenta, no
active bleeding now and no pain. Next step in management?
A. Ultrasound
B. Biophysical profile
C. CTG
D. Emergency delivery

Correct Answer: C
Explanation: (According to Uptodate)
The first thing to do in antepartum hemorrhage is US to exclude placenta previa. Then CTG.

36 years old G4P3+0 , 38 weeks , a case of polyhydramnios, you did ARM followed by
vaginal bleeding and CTG showed fetal Bradycardia. What is the possible cause ?

A. Abruptio placenta
B. Placenta previa
C. Vasa previa

Correct Answer: C
Explanation:
Vasa Previa hallmark:
- Painless vaginal bleeding that occurs suddenly after ROM
- Fetal bradycardia or sinusoidal pattern

Pregnant women 32 weeks complaining with vaginal bleeding, there were also contractions
and dilation. What is the type of bleeding?

A. Early postpartum bleeding.

65 | P a g e
Obstetrics and Gynecology: SMLE Questions and Answers Chapter 1: Obstetrics
B. Late postpartum bleeding.
C. Antepartum bleeding.
D. Intrapartum bleeding

Correct Answer: D

Another recall

Patient pregnant about 31 weeks presented with bleeding and cervix dilated 7 cm ( no
mention of any contractions). What is the type of bleeding?

A. Early postpartum bleeding.


B. Late postpartum bleeding.
C. Antepartum bleeding.
D. Intrapartum bleeding

Correct Answer: D

Another recall

Pregnant women at 32 weeks complaining with vaginal bleeding, there is no history of


contractions or cervical dilation. What is the type of bleeding?

A. Early postpartum bleeding.


B. Late postpartum bleeding.
C. Antepartum bleeding.
D. Intrapartum bleeding

Correct Answer: C
Explanation: (According to Uptodate)
- Antepartum hemorrhage: Vaginal bleeding after 20 weeks of gestation that is unrelated
to labor and delivery.
- Intrapartum hemorrhage: Vaginal bleeding occurring in the course of normal labor and
delivery.

A pregnant woman presented with massive vaginal bleeding from the abruption placenta
and her Hgb: 8.6, BP 84\40, HR140. What is the best management to save her life?

A. Admit to ICU
B. Immediate Transfusion of 2 packs FFP
C. Call multidisciplinary and rapid response team (RRT)
D. Immediate Delivery

Correct Answer: C
Explanation: (According to ACOG)
- Some emergencies are truly sudden and catastrophic, such as a ruptured aneurysm, massive
pulmonary embolus, or complete abruptio placentae in a trauma setting. However, many

66 | P a g e
Obstetrics and Gynecology: SMLE Questions and Answers Chapter 1: Obstetrics
emergencies are preceded by a period of instability during which timely intervention may
help avoid disaster.
- Medical emergency teams—sometimes referred to as “Ob Team Stat” for obstetric
emergencies or a rapid response team—are designated skilled responders who are ready to
intervene during such emergencies.

Pregnant came with PROM 4 weeks ago and now she came with low BP and low hb

A. Admit ICU under the OB care


B. Rapid response team and multi-specialty

Correct Answer: B
Explanation:
Prolonged PROM is a risk factor for placenta abruption à which causes DIC à must be managed
by Rapid response team with multidisciplinary intervention

33 years old, pregnant women presented with lower abdominal pain and moderate vaginal
bleeding, known case of bronchial asthma, epilepsy and smoker. What is the highest risk
factor for her condition?

A. Age
B. Smoking
C. Bronchial asthma
D. Epilepsy

Correct Answer: B
Explanation:
- This is Placental abruption. Smoking is one of the risk factors.
- Also, maternal age >35 is a risk factor. (our patient is 33) and still even if she was >35
years of age, smoking is a stronger risk factor than the age.

Patient admitted to the labor room, she received prostaglandin, patient massively bleeding
with stop in uterine contraction. What is the cause of her condition?

A. Placenta previa
B. Uterine rapture
C. Placenta abruption
D. Prostaglandin hypersensitivity

Correct Answer: B
Explanation:

67 | P a g e
Obstetrics and Gynecology: SMLE Questions and Answers Chapter 1: Obstetrics
Typical scenario of uterine ruptureà Patient on oxytocin or prostaglandin then sudden pause of
uterine contractions!
- Managed by immediate laparotomy and emergency C-section.

Pregnant in third trimester, with vaginal bleeding, abdominal exam shows a length less
than the gestational age, CTG shows late decelerations, diagnosis?

A. Placenta previa
B. Vasa previa
C. Placenta abrubtion

Correct Answer: A
Explanation:
- Painless vaginal bleeding à Placenta previa
- Painless vaginal bleeding upon rupture of membrane à Vasa previa
- Painful vaginal bleeding à Placenta abruption.

Another recall

Pregnant in third trimester, with abdominal pain and vaginal bleeding, abdominal exam
shows a length less than the gestational age, CTG shows late decelerations, diagnosis?

A. Placenta previa
B. Vasa previa
C. Placenta abrubtion

Correct Answer: C
Explanation:
- Painless vaginal bleeding à Placenta previa
- Painless vaginal bleeding upon rupture of membrane à Vasa previa
- Painful vaginal bleeding à Placenta abruption.

Pregnant at 27 weeks of gestation, came with minimal vaginal bleeding, us showed placenta
totalis. What is the most important step in management?

A. Antibiotics
B. Tocolytics
C. Steroids

Correct Answer: C
Explanation: (According to Uptodate)

68 | P a g e
Obstetrics and Gynecology: SMLE Questions and Answers Chapter 1: Obstetrics
In patients with placenta previa A course of antenatal corticosteroid therapy is administered to
patients who experience bleeding.

Pregnant at 38 weeks of gestation, had polyhydramnios and PROM recently. Presented


with painful vaginal bleeding and uterine tenderness, CTG finding shows persistent fetal
bradycardia, what would be the cause?

A. Cord prolapse
B. Abruptio placenta
C. Vasa previa
D. Placenta previa

Correct Answer: B
Explanation:
Let’s Exclude!
- Cord prolapse à will not present with bleeding and tenderness
- Abruptio placenta à Usually presents with painful vaginal bleeding, abdominal tenderness.
Also, polyhydramnios especially after rapid decompression is a risk factor for placenta
abruption.
- Vasa previa à Painless vaginal bleeding upon rupture of the membranes
- Placenta previa à Painless vaginal bleeding.

Painless mild painless vaginal bleeding at 34 weeks, next step?

A. Admission
B. C-section
C. Steroid
D. Antibiotic

Correct Answer: A
Explanation:
Management of Placenta previa:

69 | P a g e
Obstetrics and Gynecology: SMLE Questions and Answers Chapter 1: Obstetrics
Placenta previa when to do C-section?

A. 36-37 weeks
B. 38 weeks
C. 39 weeks
D. 40 weeks

Correct Answer: A
Explanation:
Management of Placenta previa:

A 30 years-old woman G2 P1 at 34+2 weeks gestation presents to emergency room


reporting painless vaginal bleeding. Immediately transvaginal ultrasound shows placenta
completely overlying the cervical os. A fetus in cephalic presentation, and an amniotic fluid
index of 14. The cervical appears long and closed on speculum examination. She has slow,
continuous vaginal bleeding. Fetal heart is monitored (image missing)

BP 110/78

HR 106

RR 14

Temperature 36.9C

Which of the following is the most appropriate in management?

A. Hospitalization
B. Betamethasone
C. Cesarean section
D. Magnesium sulfate

Correct Answer: A
Explanation:

70 | P a g e
Obstetrics and Gynecology: SMLE Questions and Answers Chapter 1: Obstetrics
Management of Placenta previa:

Patient at 34 weeks of gestation, came after history of fall at home with abdominal
tenderness and noticed reduced fetal movement, 4cm cervix and 80% effacement. Fetal
heart rate 150, and moderate utrine contractions every 3 to 4 minutes. Whats diagnosis?

A. Vasa previa
B. Placenta abruption
C. Placenta previa
D. Latent phase of labor

Correct Answer: B
Explanation: (According to Uptodate)
Concealed abruptio placentae: In ∼ 20% of cases, the hemorrhage is mainly retroplacental;
vaginal bleeding does not occur and presents only with history of one of the risk factors for
placenta abruption (e.g. HTN, trauma, smoking or cocaine use) and abdominal tenderness

A primgravida at 28 weeks of gestation and a heavy smoker presented with severe vaginal
bleeding and abdominal pain. What is the most likely cause?

A. Rupture of fetal artery


B. Uterine rupture
C. Vasa previa
D. Placental abruption

Correct Answer: D
Explanation:
Smoking is a risk factor for abruptio placenta.
Patient with abruptio will present with abdominal pain or tenderness with vaginal bleeding

Pregnant unbooked present with painless vaginal bleeding, fundal high 34 weeks. She lives
far away and has difficulty in trasport. What is the most appropriate thing to do?

71 | P a g e
Obstetrics and Gynecology: SMLE Questions and Answers Chapter 1: Obstetrics
A. Corticosteroid induction
B. Deliver by C-section
C. US
D. Admit to ward

Correct Answer: D
Explanation: (According to Uptodate)
Any woman with placenta previa can’t be discharged from the hospital unless she is able to
return to the hospital within 20 minutes

Pregnant with history of placental abruption 2 times before came in 3rd preg with same
condition and severe bleeding she's on 37 week, when to admit patient?

A. Admit now
B. Wait till next bleeding
C. Wait until determination of labor day
D. Discharge and reassure.

Correct Answer: A
Explanation:
We deliver all pregnancies with suspected acute abruption at ≥36+0 weeks of gestation

Most condition cause pregnancy DIC?

A. Placenta abruption
B. Retained product of conception
C. Placenta previa
D. Congenital hemorrhagic disorders

Correct Answer: A
Explanation: (According to Uptodate)
The type and frequency of pregnancy-related conditions that triggered DIC
- Placental abruption (37%)
- Postpartum hemorrhage (PPH) (29%)
- Preeclampsia/eclampsia/HELLP syndrome (14%)
- Acute fatty liver (8%)
- Amniotic fluid embolism(6%)
Pregnancy-related sepsis (6%)

Pregnant female at 30 weeks of gestation, present with abdominal pain and bleeding.
What’s the next step?

A. Send for US

72 | P a g e
Obstetrics and Gynecology: SMLE Questions and Answers Chapter 1: Obstetrics
B. IV fluids

Correct Answer: B
Explanation:
ABC!

38 week pregnant came in labor, cervix was 10 cm dilated baby is in -3 station, CTG:
bradycardia, poor variability with recurrent late deceleration, the mother developed
abdominal pain and blood, what is the most appropriate next step?

A. Immediate laparotomy
B. Observation
C. Send to home and give follow up appointment

Correct Answer: A
Explanation:
Painful uterine bleeding is most probably due to placenta abruption or uterine rupture!
- The most common fetal distress in uterine rupture is fetal bradycardia.
- Management of uterine ruptureà Immediate laparotomy with emergency C-section

Pregnant at 34 weeks of gestation, presenting with heavy bleeding and contraction.


Ultrasound showing fundal posterior placental with retroplacental fluid and baby in
transverse lie, 3cm cervical dilatation. What to do ?

A. C-section
B. Observation
C. Tocolytics
D. ECV

Correct Answer: A
Explanation: (According to Uptodate)
Placenta abruption at 34 to 36 weeks of gestation — We deliver most patients with acute
abruption at 34+0 to 36+6 weeks of gestation, since these patients remain at risk of maternal or
fetal compromise.

Pregnant with placenta previa what’s the common complication?

A. Coagulation abnormalities
B. Postcoital bleeding

Correct Answer: B
Explanation: (According to Uptodate)

73 | P a g e
Obstetrics and Gynecology: SMLE Questions and Answers Chapter 1: Obstetrics
- The most common symptom of placenta previa is relatively painless vaginal bleeding

- PATHOPHYSIOLOGY OF BLEEDING:
Placental bleeding is the major adverse sequelae of placenta previa. It is thought to occur
when uterine contractions or gradual changes in the cervix and lower uterine segment apply
shearing forces to the inelastic placental attachment site, resulting in partial detachment.
Vaginal examination or coitus can also disrupt this site and cause bleeding. Bleeding is
primarily maternal blood from the intervillous space, but fetal bleeding can occur if fetal
vessels in the terminal villi are disrupted.

Patient known case of low laying placenta previa and stable on follow up, came to ER today
on week 32 with minimal bleeding and stopped at home. What is the initial step?

A. CTG
B. US
C. Bio physical profile
D. Emergency delivery

Correct Answer: A
Explanation:
The first thing to do in antepartum hemorrhage is US to exclude placenta previa. Then CTG.
But she is already diagnosed with placenta previa, so I would go with CTG.

Patient presented with painless vaginal bleeding and placenta covering the internal os ,
Which of the following is considerd a risk factor for her presentation?

A. Hypertension
B. Multiple gestation
C. Smoking
D. DM

Correct Answer: B
Explanation: (According to Uptodate)
Risk factors for Placenta previa:
- Major risk factors
• Previous placenta previa
• Previous cesarean birth
• Multiple gestation
- Other risk factors
• Previous uterine surgical procedure
• Increasing parity

74 | P a g e
Obstetrics and Gynecology: SMLE Questions and Answers Chapter 1: Obstetrics
• Increasing maternal age
• Infertility treatment
• Maternal smoking
• Maternal cocaine use
• Male fetus
• Prior uterine artery embolization
• Endometriosis
• Abortion, either spontaneous or induced

Female patient pregnant with history of previous x4 C-sections presented with mild
bleeding and hypotension, what is the most likely cause of her presentation?

A. Cord prolapse
B. Uterine rupture
C. Abruptio placentae
D. Fetal vessel rupture

Correct Answer: B
Explanation: (According to Williams Obstetrics and Uptodate)
Uterine rupture
- Risk factors
Surgery involving the myometrium:
• Cesarean delivery or hysterotomy
• Previously repaired uterine rupture
• Myomectomy incision through or to the endometrium
• Deep cornual resection of interstitial fallopian tube Metroplasty
Coincidental uterine trauma:
• Abortion with instrumentation-sharp or suction curette, sounds
• Sharp or blunt trauma-assaults, vehicular accidents, bullets, knives
• Silent rupture in previous pregnancy
Congenital
• Pregnancy in undeveloped uterine horn
• Defective connective tissue-Marfan or Ehlers-Danlos syndrome
- Clinical presentation
• Abnormal fetal heart rate (FHR) –category II or a category III FHR pattern
• Abdominal pain
• Vaginal bleeding –may occur but is not a cardinal symptom as it may be modest or
even absent despite major intraabdominal hemorrhage.
• Loss of station of the fetal presenting part
• Hematuria, if the rupture extends into the bladder.
• Hemodynamic instability – Intraabdominal hemorrhage from the site of rupture
can lead to rapid maternal hemodynamic deterioration (hypotension and
tachycardia).

75 | P a g e
Obstetrics and Gynecology: SMLE Questions and Answers Chapter 1: Obstetrics
•Changes in contraction patterns – Both increased uterine contractility and loss of
uterine tone
- Management
• Stabilize the patient and immediate laparotomy

Pregnant with low risk pregnancy at 32 weeks came with vaginal bleeding, diagnosed as
placenta abruption and managed conservatively , most appropriate next step?

A. Remain in the hospital


B. Reclasify as high risk and f/u with US for fetal growth
C. Apply as low risk with f/u outpatient clinic
D. Apply as low risk and f/u with US for vaginal bleeding

Correct Answer: A
Explanation: (According to Uptodate and Williams Obstetrics)
Management of Placental Abruption in stable mother and reassuring fetus <34 weeks:
- Hospitalization – There are no compelling data to guide the length of a hospital stay for
these patients. A reasonable approach is to monitor the patient in the hospital until the
bleeding has subsided for at least 48 hours, fetal heart rate tracings and ultrasound
examinations are reassuring, and the patient is asymptomatic. At that point, discharge may
be considered.

A 29 years old female pregnant at 29 week GA came to the ER with lower abdominal pain,
during Ultrasound you detected 100ml of blood behind the closed cervix. What would you
give?
A. Dexamethasone
B. Indomethacin
C. Mg sulphate
D. Nifedipine

Correct Answer: A
Explanation: (According to Uptodate)
Placental abruption
- Presentation:
• Classic presentation – Patients with an acute abruption classically present with the
abrupt onset of vaginal bleeding, mild to moderate abdominal pain, and uterine
contractions.
• Other presentations – In 10 to 20 percent of abruptions, patients present for care
only because of uterine contractions/irritability with no or minimal vaginal bleeding
and no abdominal pain. In these cases, termed concealed abruption, all or most of
the blood is trapped between the fetal membranes and decidua rather than escaping
through the cervix and vagina.
• In a few cases, a small, concealed abruption may be asymptomatic and only
recognized as an incidental finding on an ultrasound examination.
- Management:

76 | P a g e
Obstetrics and Gynecology: SMLE Questions and Answers Chapter 1: Obstetrics
• Stable mother and reassuring fetal status — If the fetal heart rate pattern or
biophysical profile score is reassuring, then the decision to deliver versus manage
expectantly depends on gestational age and whether the abruption has led to onset of
spontaneous labor.
§ Less than 34 weeks of gestation — When the fetus and mother are both
stable and there is no evidence of ongoing major blood loss or coagulopathy,
conservative management with the aim of delivering a more mature fetus is
the main goal before 34+0 weeks of gestation
o For patients not in labor, à Administer antenatal
corticosteroids – Given the increased risk of need for preterm birth,
antenatal corticosteroids to promote fetal lung maturation and reduce
complications of preterm birth

Patient at 37 weeks of gestation with placenta totalis present mild vaginal bleeding, when to
admit her for delivery?
A. Immediately
B. After planning C-section
C. No need to admit
D. After another episode of vaginal bleeding

Correct Answer: A
Explanation: (According to ACOG and Uptodate)
Placenta previa
- Timing of delivery — We perform a cesarean birth at 36+0 to 37+6 weeks of gestation in
pregnancies with uncomplicated placenta previa

Female, 30 year old pregnant at 30 weeks of gestation came with severe antepartum
hemorrhage, CTG with deceleration, the mother BP 90/60, what is the appropriate next
step?
A. Steroids
B. Observe
C. C-section

Correct Answer: C
Explanation:
Antepartum hemorrhage with unstable mother à emergency cesarean section

Pregnant at 27 weeks of gestation, presented with scanty vaginal bleeding. US showed


placenta previa totals. She's hemodynamically stable. CTG normal. What is the most
appropriate management?

A. Oxygen
B. Steroid
C. Tocolytic

77 | P a g e
Obstetrics and Gynecology: SMLE Questions and Answers Chapter 1: Obstetrics

Correct Answer: B
Explanation:
Management of Placenta previa:

Pregnant came to the ER today with moderate bleeding , what is the next step?

A. Transfer to US
B. Insert 2 large IV cannula
C. Emergency delivery
D. Pelvic examination

Correct Answer: B
Explanation:
Initial management in case of Antepartum hemorrhage:
- Initiate continuous fetal heart rate monitoring
- Secure intravenous access.
- Administer crystalloid

Most important risk factor for placental abruption?

A. Short umbilical cord


B. Hypertension
C. Fibroid

Correct Answer: B
Explanation: (According to Williams Obstetrics
Placental abruption risk factors:

78 | P a g e
Obstetrics and Gynecology: SMLE Questions and Answers Chapter 1: Obstetrics

Placental abruption and fetal distress and ph 6 wt improve mortality now?


A. Mild hypothermia
B. IV Fluid
C. Electrolytes

Correct Answer: A
Explanation:
Hypothermia treatment, also known as brain cooling, is a relatively new treatment option that
lowers a newborn’s body temperatures in order to reduce neurological injury. Traumatic births
restrict the flow of oxygen to a baby’s brain and a brain injury called hypoxic ischemic
encephalopathy (HIE) often results. Research indicates that the initiation of hypothermia treatment
within six hours of an oxygen-depriving insult significantly reduces chances of death and
neurological injuries like cerebral palsy.

79 | P a g e
Obstetrics and Gynecology: SMLE Questions and Answers Chapter 1: Obstetrics

Labor and CTG Monitoring

Best way to deliver intrauterine growth restriction fetus with reassuring CTG?
A. Spontaneous vaginal
B. C-section
C. Ventouse
D. Foreceps
Correct Answer: A
Explanation: (According to Uptodate)
Fetal growth restriction, Route of delivery — Cesarean birth is performed for standard obstetric
indications; otherwise a trial of labor is acceptable. An unfavorable cervix is not a reason to avoid
induction

Primigravida, during labor, cervix fully dilated, CTG show variable deceleration, patient
have strong contraction and head is engaged (station 0), cephalic presenting part, what is
the most appropriate management?
A. Ventouse
B. Foreceps
C. C-section
D. Wait 2 hours
Correct Answer: D
Explanation:
Let’s exclude!!
- Ventouse and Foreceps à We can’t use instrumental delivery (Ventose and forceps unless the
CERVIX IS FULLY DILATED AND AT STATION +2 and BEYOND!) never ever choose
ventose and forceps in stations that are less than +2.
- C-section à wrong answer, because it’s the last resort and should be chosen if the CTG is
category3
- The CORRECT answer is in utero resuscitation measure e.g. lateral positioning of the mother,
O2, IV fluid, stop oxytocin, administer toocolytic drugs. If not in the choices? I would go for
D

Pregnant at term 38 weeks, with irregular uterine contractions, CTG with no acceleration or
deceleration but normal variability, heart rate 140, cervix is 2 cm dilated, what to do next?
A. Oxytocin
B. Prostaglandin
C. C-section
D. Observation or send home
Correct Answer: D
Explanation:
Her CTG is Category 1 and reassuring. She is still 2 cm dilated (latent phase) no need for
intervention.

80 | P a g e
Obstetrics and Gynecology: SMLE Questions and Answers Chapter 1: Obstetrics
CTG Categories:
- Category 1
The fetal heart tracing shows All of the following
• Baseline FHR 110-160 BPM
• Moderate variability
• +- Accelerations
• +- Early decelerations

Management:
Routine surveillance

- Category 2
• Include all FHR tracing that are not included in. category1 or 3

Management:
In utero resuscitation measure e.g. lateral positioning of the mother, O2, IV fluid, stop
oxytocin, administer toocolytic drugs, and surveillance

- Category 3
At least one of the following
• Absent variability with recurrent late decelerations
• Absent variability with recurrent variable decelerations
• Absent variability with bradycardia
• Sinusoidal pattern for at least 20 minutes

Management:
In utero fetal resuscitation measures and prepare of delivery

Pregnant on epidural and oxytocin suddenly contractions stopped CTG Picture as shown
above, what would you do?
A. Stop oxytocin and immediate delivery
B. Stop epidural
C. Increase oxytocin
D. Change to lateral position and recheck after 20 minutes
Correct Answer: A
Explanation:

81 | P a g e
Obstetrics and Gynecology: SMLE Questions and Answers Chapter 1: Obstetrics
Her CTG picture showing: Prolonged decelerations and stoppage of uterine contraction.
(According to the recall, in the exam the CTG picture was showing: variable deceleration

Oxytocin side effects: uterine tachysystole and Category II or III FHR tracings are the most
common side effects. Uterine tachysystole may result in abruptio placentae or uterine rupture.

- The patient had sudden contraction stoppage! Which indicates uterine rupture (unlike
abruptio placenta causes hypertonic contractions).
- The common clinical manifestation of uterine rupture is abnormal fetal heart rate pattern
(bradycardia, variable, or late decelerations)
- Uterine rupture is managed by immediate delivery by CS

Primigravid come with labor for 4h Dilated 5 cm, effaced 80%, station +1 after 5h there is no
change in cervix, and contraction occur every 3 min. and stay for 60 sec. What to do?
A. Instrument use
B. C-section
C. IV oxytocin
D. Wait for 2h
Correct Answer: D
Explanation:
- She is still 5 cm dilated (latent phase) no need for intervention she can stay in the latent
phase up to 20 hours.
- When to choose IV oxytocin? -> if rupture of membrane is mentioned in the question. To
avoid infections, otherwise? NO INTERVENTION IN THE LATENT PHASE AT ALL.

A primigravida patient presented in labor. O/E: the cervix is 5cm dilated and the fetus is in a
station O with cephalic presenting part and this state for 4 hours even the oxytocin had been
taken. CTG Picture as shown above. what is the management for this patient?
A. Stop oxytocin
B. Immediate C-section
C. Wait for 2 hours
D. Instrumental delivery
Correct Answer: A
Explanation:

82 | P a g e
Obstetrics and Gynecology: SMLE Questions and Answers Chapter 1: Obstetrics
Her CTG is category 2 and is managed by in utero resuscitation measure e.g. lateral positioning of
the mother, O2, IV fluid, stop oxytocin, administer toocolytic drugs.

Let’s Exclude!!
- Immediate CS -> if her CTG is category3
- Wait for 2 hours -> if her CTG is category1
- Instrumental Delivery -> If her cervix is fully dilated and the head is engaged +2 and
beyond.

CTG Categories:
- Category 1
The fetal heart tracing shows All of the following
• Baseline FHR 110-160 BPM
• Moderate variability
• +- Accelerations
• +- Early decelerations

Management:
Routine surveillance

- Category 2
• Include all FHR tracing that are not included in. category1 or 3

Management:
In utero resuscitation measure e.g. lateral positioning of the mother, O2, IV fluid, stop
oxytocin, administer toocolytic drugs, and surveillance

- Category 3
At least one of the following
• Absent variability with recurrent late decelerations
• Absent variability with recurrent variable decelerations
• Absent variability with bradycardia
• Sinusoidal pattern for at least 20 minutes

Management:
In utero fetal resuscitation measures and prepare of delivery

83 | P a g e
Obstetrics and Gynecology: SMLE Questions and Answers Chapter 1: Obstetrics
Pregnant in labor was induced by oxytocin, CTG showing late deceleration (pic). what to
do to reverse condition?
A. Give epidural anesthesia.
B. Give morphine.
C. Let mother sleep supine.
D. Stop oxytocin
Correct Answer: D
Explanation:
Our patient has CTG category2, which is managed by in utero resuscitation measure e.g. lateral
positioning of the mother, O2, IV fluid, stop oxytocin, administer toocolytic drugs.
*Never choose supine position, the correct position is lateral, it was a trick question

A 39-week pregnant patient, history of caesarean section, due to breech presentations, now
she is in labor, with regular contractions 4 every 5 min, cervix fully dilated, full effacement,
station +3, What is your management?
A. Ventouse delivery
B. Cesarean section
C. Examine her after 2 hours.
Correct Answer: A
Explanation:
According to ACOG:
Similar standards should be used to evaluate the labor progress of women undergoing Vaginal
delivery after cesarean and those who have not had a prior cesarean delivery.

Let’s Exclude!!
- Ventose Delivery -> because her cervix is fully dilated, and the head is engaged +2 and
beyond. Although there is no strong indication for ventouse. Ventouse delivery indication
examples are: CTG category 2, maternal exhaustion or prolonged 2nd stage of labor.
But we answered this question by exclusion.
- Immediate CS -> if her CTG is category3
- Examine her after 2 hours -> if she wasn’t fully dilated and station +3

Pregnant patient in labor. O/E: the cervix is fully dilated for 2 hrs, head in a station - 2, but
the patient is got exhausted from pushing. What is your next step?
A. Cesarean delivery
B. Wait for another 2 hours
C. Ventouse delivery
D. Forceps delivery
Correct Answer: A
Explanation:
Prolonged second stage of labor:
- Definition:
• Greater than 3 hours in nulliparous women and greater than 2 hours in multiparous
women (in patients who received an epidural add an extra hour)
- Managed by:

84 | P a g e
Obstetrics and Gynecology: SMLE Questions and Answers Chapter 1: Obstetrics
• Instrumental Delivery (if the head is engaged at +2 station and beyond) (OUR
PATIENT DOES NOT FULL FILL THE CRITERIA)
• Cesarean Section (if the instrumental delivery is contraindicated) (LIKE OUR
PATIENT)

41 Weeks pregnant with non-reassuring CTG and she has fibroid, what is the most
appropriate step in the management?
A. Induce labor
B. C-section
C. CTG Daily

Correct Answer: B
Explanation:
- Non-reassuring CTG means CTG Category 2 or 3. But, as long as he didn’t specify the
category, then anticipate the worst and manage accordingly.
Let’s Exclude!!
- Induce labor à we cant induce labor (oxytocin) if her CTG is non-reassuring, as the
management of non-reassuring CTG in to stop oxytocin
- C-section à answered by exclusion and it’s the difintive management of CTG category3
- CTG daily à You cannot manage a non-reassuring CTG with CTG daily

Pregnant 42 weeks, in labor 7 cm dilated, meconium staining liquor, regular and strong
contractions, CTG showing fetal heart rate of 100?
A. C-section
B. Augment labor

Correct Answer: A
Explanation: (According to Uptodate)
Management of Meconium stained amniotic fluid:
- Induction of labor and continuous fetal monitoring. (expectant management in case of
reassuring CTG is acceptable also)
- Evaluation and interventions are implemented in cases with abnormal tracings indicative of
fetal stress to reduce the likelihood of perinatal asphyxia. We agree that FHR monitoring
identifies signs of hypoxemia and allows the caregivers to initiate prompt interventions in
order to reduce the risk of MAS.
Let’s Exclude!!
- C-section à By exclusion, as our patient has CTG category2 (bradycardia), which is
managed by in utero resuscitation measure e.g. lateral positioning of the mother, O2, IV
fluid, stop oxytocin, administer toocolytic drugs.
If in utero resuscitation measures are not in the choices, i would go with C-section
- Augment labor à Management of CTG category 2 is to stop oxytocin as mentioned above.
Also, she has regular and strong contractions no need to augment labor

85 | P a g e
Obstetrics and Gynecology: SMLE Questions and Answers Chapter 1: Obstetrics

Pregnant, primigravida in labor for couple of hours, 6cm dilated, 80% effacement, 0
station, was managed with oxytocin and ruptured membrane for 20 hours. CTG showed
late deceleration (see above pic) what is the appropriate mx?
A. C-section
B. Stop oxytocin
C. Amnioinfusion
D. Ampicillin

Correct Answer: A
Explanation:
Prolonged Active Phase:
- Definition:
≥ 6 cm cervical dilation and one of the following:
• No change in cervical dilation after 6 hours of inadequate contractions
• No change in cervical dilation after 4 hours of adequate contractions
- Managed By:
• Augmentation with Oxytocin for hypotonic contractions (with cervical ripening for
unfavorable cervix 6cm or less)
• Amniotomy (Rupture of membrane)
Arrested Active Phase:
- Definition:
≥ 6 cm cervical dilation with ruptured membranes and no cervical change after one of the
following:
• ≥ 4 hours of adequate contractions
• > 6 hours of inadequate contractions despite oxytocin administration
- Managed By:
• Cesarean Section!

Let’s Exclude!!
- C-section à The patient had prolonged active phase of labor-> she was managed by
amniotomy and oxytoci. Now she is having arrested active phase, which is managed by
C-section!
- Stop oxytocin à I would do it as a next step (not the most appropriate) initiating in utero
resuscitative measures while I’m waiting for the C-section (in utero resuscitative measures
includes: change of maternal position is a reasonable first treatment option, followed by
O2, IV fluid, stop oxytocin, administer toocolytic drugs.)

86 | P a g e
Obstetrics and Gynecology: SMLE Questions and Answers Chapter 1: Obstetrics
- Amnioinfusion à is the second line option after in utero resuscitative measures (so, i will
not choose it as a next step or most appropriate)
- Ampicillin à If he says next not most appropriate.

20-years-old primigravida, 42 weeks with closed cervix. Induction of labor with


prostaglandins gel was started. Her CTG was “fetal HR 140-160” after 1 h fetal HR 80 and
uterine contraction last 2 minutes, most important step in management?
A. C-section
B. Oxygen mask
C. SC terbutaline
D. Check for cord prolapse

Correct Answer: C
Explanation:
Uterine Tachysystole
- Definition
• >5 contractions in 10 minutes, averaged over a 30-minute window. Or uterine
hypersystole/hypertonus (a contraction lasting at least 2 minutes) .
• Uterine tachysystole is one of the causes of fetal bradycardia.
- Managed by:
• Discontinue oxytocin or cervical ripening agents + administer tocolytics (e.g.
terbutaline)

Note:
**Check cord prolapse-> if the membrane was ruptured (hint) was in the question

Pregnant lady during labor, CTG show fetal persistent bradycardia, what is the cause of
her condition?
A. Placental insufficiency
B. Congenital heart disease

Correct Answer: A
Explanation: (According to ACOG)
- Rarely, bradycardia occur in fetuses with congenital heart abnormalities or myocardial
conduction defects, such as those associated with maternal collagen vascular disease.

87 | P a g e
Obstetrics and Gynecology: SMLE Questions and Answers Chapter 1: Obstetrics
- Most often the onset of bradycardia associated with congenital heart block occurs in the
second trimester; it is extremely unlikely that new onset intrapartum bradycardia would be
due to this condition.

Pregnant lady, 41 weeks GA in labor on epidural analgesia, mg sulfate for pre-eclampsia


and oxytocin, CTG showed prolonged deceleration and the mother was hypotensive, most
likely cause of the CTG finding:
A. Mg Sulfate
B. Oxytocin
C. Epidural analgesia

Correct Answer: C
Explanation: (According to ACOG)
Medications that can affect fetal heart rate:
- Mg Sulfate: causes minimal or reduced variability
- Epidural analgesia: causes maternal hypotension à uteroplacental insufficiency à late or
prolonged decelerations
- Oxytocin: late or prolonged decelerations + uterine hyperstimulation

Pregnant 38 weeks. Diagnosed with preeclampsia and managed with magnesium sulfate.
Shes in labor and epidural anesthesia was started. Oxytocin infusion is started as well.

88 | P a g e
Obstetrics and Gynecology: SMLE Questions and Answers Chapter 1: Obstetrics
Normal regular contraction. CTG picture: as shown above, Whats the cause of this CTG
finding?
A. Oxytocin infusion
B. Magnesium sulfate infusion
C. Epidural analgesia
D. Head position of the baby

Correct Answer: B
Explanation:
CTG picture is showing, reduced variability.

Medications that can affect fetal heart rate:


- Mg Sulfate: causes minimal or reduced variability
- Epidural analgesia: causes maternal hypotension à uteroplacental insufficiency à late or
prolonged decelerations
- Oxytocin: late or prolonged decelerations + uterine hyperstimulation

Reduced variability is caused by?


A. Magnesium sulfate
B. Epidural analgesia

Correct Answer: A
Explanation:
Medications that can affect fetal heart rate:
- Mg Sulfate: causes minimal or reduced variability
- Epidural analgesia: causes maternal hypotension à uteroplacental insufficiency à late or
prolonged decelerations
- Oxytocin: late or prolonged decelerations + uterine hyperstimulation

Woman had C-section. What is the best way to prevent adhesions?


A. Perform the C-section before onset of labor
B. Add adhesion barrier consisting of oxidized regenerated cellulose before closing the
wound
C. Closure of the peritoneum
D. Add a layer the incision site

Correct Answer: B

Normal vaginal delivery, Baby weight 4.2kg, Laceration reaching rectal mucosa, which
degree:
A. First
B. Second

89 | P a g e
Obstetrics and Gynecology: SMLE Questions and Answers Chapter 1: Obstetrics
C. Third
D. Fourth

Correct Answer: D
Explanation:
Degrees of perineal tears
- First degree: Skin
- Second degree: Muscle
- Third degree: Sphincter
- Fourth degree: Rectal mucosa

35-Year-old pregnant lady with fetal death and DIC, her cervix is 4cm dilated, (her vitals
are normal), what is the management?
A. Induction of labor
B. Urgent C-section

Correct Answer: A

Another recall

35-Year-old pregnant lady with fetal death and DIC, her cervix is 4cm dilated, (her vitals
shows hypotension), what is the management?
A. Induction of labor
B. Urgent C-section

Correct Answer: B
Explanation: (According to Uptodate)
DIC in Pregnancy
- In Hemodynamically unstable mother OR fetal distress OR contraindication to vaginal
delivery
• Cesarean delivery is indicated
Why? if the mother is hemodynamically unstable, Vaginal delivery is not the safest
maternal option if hemodynamic instability from ongoing brisk uterine bleeding persists
despite vigorous transfusion of blood and blood products. In these cases, cesarean
delivery is indicated to save the mother's life
- In Hemodynamically stable mother with dead or nonviable fetus
• Induction of Labor is indicated
Why? avoiding cesarean delivery because of the risk of uncontrollable hemorrhage
from surgical incisions and lacerations.
Delivery is initiated, as removal of the products of conception removes the trigger for
DIC

- In Hemodynamically stable mother with a viable fetus and reassuring fetal status
• In those rare cases in which the mother with acute disseminated intravascular
coagulation (DIC) is responding appropriately to resuscitation with blood products and

90 | P a g e
Obstetrics and Gynecology: SMLE Questions and Answers Chapter 1: Obstetrics
the fetal heart rate (FHR) tracing is normal, there is no compelling need for immediate
delivery (induction of labor, cesarean birth). Instead, the clinical focus is on reversing or
eliminating the cause of DIC and reducing the risk of bleeding and thrombosis.
• After this has been achieved, decision-making regarding expectant management or
delivery depends on the gestational age and expected course of the underlying cause of
DIC.

Pregnant with DIC and CTG normal no deceleration and good contract what to do?
A. Augment labor
B. C-section
C. Reassure

Correct Answer: C
Explanation: (According to Uptodate)
DIC in Pregnancy
- In Hemodynamically stable mother with a viable fetus and reassuring fetal status
• In those rare cases in which the mother with acute disseminated intravascular
coagulation (DIC) is responding appropriately to resuscitation with blood products and
the fetal heart rate (FHR) tracing is normal, there is no compelling need for immediate
delivery (induction of labor, cesarean birth). Instead, the clinical focus is on reversing or
eliminating the cause of DIC and reducing the risk of bleeding and thrombosis.
• After this has been achieved, decision-making regarding expectant management or
delivery depends on the gestational age and expected course of the underlying cause of
DIC.

Pregnant lady, diabetic, 38 weeks in active labor and having DKA profile and fetus in
distress (CTG that is suggested of bradycardia) what to do?
A. Change the mother’s position of labor
B. Stop and do C-section

Correct Answer: A
Explanation: (According to Uptodate and ACOG)
Diabetic KetoAcidosis in Pregnancy DKA
- Clinical presentation
• Includes abdominal pain, nausea, vomiting, and altered sensorium.
- Laboratory findings:
• Includes hyperglycemia (usually >250 mg/dL [13.9 mmol/L]), acidemia (arterial pH
<7.30), an elevated anion gap (>12 mEq/L), ketonemia, low serum bicarbonate (<15
mEq/L), elevated base deficit (>4 mEq/L), and renal dysfunction
- Continuous fetal heart rate monitoring
• Minimal or absent variability and absent accelerations, as well as repetitive
decelerations. These abnormalities usually resolve with resolution of DKA
- Management:
• Intravenous insulin, appropriate volume replacement, correction of electrolyte
abnormalities (including potassium, phosphate, and magnesium), monitoring acidosis,
and a search for precipitating causes.
91 | P a g e
Obstetrics and Gynecology: SMLE Questions and Answers Chapter 1: Obstetrics
• Emergency delivery before maternal stabilization should be avoided because it
increases the risk of maternal morbidity and mortality and may result in delivery of a
hypoxic, acidotic preterm infant.

Note:
- If IV insulin and hydration in the choices I would choose it.
- If not? in utero resuscitation measure is also correct.

Urine Dipstick pic with: +2 protein, very high glucose, +ve ketones in pregnant lady 39
weeks with effacement 90% and cervix dilation 2 cm, what is your most appropriate
action?
A. Induction of labor (IOL)
B. C-section
C. Expectant management

Correct Answer: A
Explanation: (According to ACOG )
When to deliver in case of Diabetes Mellitus?
- At 39+0 to 39+6 weeks if well-controlled glucose levels and no vascular disease;
- At 36+0 to 38+6 weeks if poorly controlled glucose levels or vascular disease (even earlier
if severity of complications warrants earlier delivery)
- Expectant management beyond 40+0 weeks is not recommended.

Note:
- Delivery before maternal stabilization should be avoided!
- For this patient the correct answer is to manage the DKA (by insulin and hydration) and
stabilize the mother! à Then induction of labor after correction of her status.
- If IV insulin and hydration in the choices I would choose it. If not? IOL

While the obstetrician closes the caesarean incision, patient developed bleeding. What is the
cause?
A. Liver haemangioma
B. Spleen aneurysm
C. Perforated peptic ulcer
D. Mesenteric ischemia

Correct Answer: B
Explanation:
Splenic artery aneurysms
- Are the third most common true aneurysm occurring in the abdomen after aortic and iliac
artery aneurysms. Splenic artery aneurysms are more common in women (female:male =
4:1) and are commonly associated with conditions of increased flow, such as pregnancy
(particularly multiparity, because the risk increases with increasing parity)

92 | P a g e
Obstetrics and Gynecology: SMLE Questions and Answers Chapter 1: Obstetrics
- Approximately, 95% of SAA rupture occurs during pregnancy, most commonly during the
third trimester.
- If a woman has an existing SAA, the risk of rupture during pregnancy is 20–50%.
- Though the rupture of a SAA during pregnancy is a rare event, it carries a high risk of
maternal and fetal mortality. The mortality in the general population when a SAA ruptures
is 25%. In pregnant women, this rate increases to a 75% maternal mortality rate and a 95%
fetal mortality rate
- Obstetricians and other emergency providers should consider a ruptured SAA in any
pregnant woman who presents with an acute surgical abdomen.
- In the rare minority of women of childbearing age who are discovered to have an
asymptomatic SAA prior to rupture, a proactive approach to management should be
undertaken due to the high risk of rupture in pregnancy.

Liver hemangioma
- May increase in size during pregnancy or with estrogen therapy. But, risk of lesion rupture
is similar for pregnant and nonpregnant women

Elective Cesarean section which week?


A. 36
B. 37
C. 38
D. 39

Correct Answer: D

Another recall

Elective Cesarean section which week?


A. 36-37
B. 38
C. 39
D. Full term

Correct Answer: D
Explanation: (According to Uptodate)
- When a primary cesarean delivery is indicated for maternal or fetal reasons, but preterm
birth is not indicated, there is consensus that planned term cesarean delivery should be
scheduled in the 39th or 40th week of gestation
- Full term = between 39 and 40 weeks

Female pregnant, polyhydramnios had ruptured membrane, on CTG persistent fetal


bradycardia?
A. Rapid fetal descend
B. Cord prolapse
C. Anomaly

93 | P a g e
Obstetrics and Gynecology: SMLE Questions and Answers Chapter 1: Obstetrics

Correct Answer: B
Explanation: (According to Uptodate)
Risk factors for cord prolapse
- Malpresentation (breech, transverse, oblique, or unstable lie)
- Preterm gestational age
- Low birth weight
- Second twin
- Low lying placentation
- Pelvic deformities
- Uterine malformations/tumors
- External fetal anomalies
- Multiparity
- Polyhydramnios
- Long umbilical cord
- Unengaged presenting part
- Prolonged labor
- Atypical placental cord insertions (velamentous and marginal)
Obstetric interventions account for approximately 50 percent of cases of cord prolapse, include:
- Iatrogenic rupture of membranes, especially with an unengaged presenting part
- Cervical ripening with a balloon catheter
- Induction of labor
- Application of an internal scalp electrode
- Insertion of an intrauterine pressure catheter
- Manual rotation of the fetal head
- Amnioinfusion
- External cephalic version
- Internal podalic version
- Application of forceps or vacuum

Note:
Polyhydramnios + Ruptured membrane + Bradycardia = Cord prolapse!

Signs of fetal distress in CTG:


A. Less contractions
B. Early deceleration
C. Late deceleration

Correct Answer: C
Explanation:
CTG Categories:
- Category 1
The fetal heart tracing shows All of the following
• Baseline FHR 110-160 BPM
• Moderate variability
• +- Accelerations
94 | P a g e
Obstetrics and Gynecology: SMLE Questions and Answers Chapter 1: Obstetrics
• +- Early decelerations

Management:
Routine surveillance

- Category 2
• Include all FHR tracing that are not included in. category1 or 3

Management:
In utero resuscitation measure e.g. lateral positioning of the mother, O2, IV fluid, stop
oxytocin, administer toocolytic drugs, and surveillance

- Category 3
At least one of the following
• Absent variability with recurrent late decelerations
• Absent variability with recurrent variable decelerations
• Absent variability with bradycardia
• Sinusoidal pattern for at least 20 minutes

Management:
In utero fetal resuscitation measures and prepare of delivery

Note
- Fetal distress = non-reassuring CTG = CTG category 2 or 3

What's the best way to deliver placenta in C-section?


A. Spontaneous separation
B. Fundus pressure
C. Manual removal

Correct Answer: C
Explanation: (According to NICE guidelines and ACOG)
Method of placental removal
Remove the placenta in caesarean birth using controlled cord traction and not manual removal to
reduce the risk of endometritis.

Two methods are frequently used to deliver the placenta at C-section:


- Cord traction
- Manual removal
The best method to deliver the placenta during C/section is by cord traction followed by fundal
message and pressure (active management of the 3rd stage to prevent PPH)

Note:
95 | P a g e
Obstetrics and Gynecology: SMLE Questions and Answers Chapter 1: Obstetrics
If cord traction in the choices it’s the correct answer, if not I would go with manual removal

Pregnant G3P2, 37 weeks with a history of C-section due to non-reassuring CTG. She is in
labor with a 4 cm dilation. The presentation is breech. What is the absolute
contraindication for ECV?
A. History of C-section
B. Active labor
C. Variable decelerations

Correct Answer: C
Explanation:
External Cephalic Version: Should be offered in all cases ≥ 37 weeks who would like to attempt
a vaginal delivery unless there are contraindications.
Contraindications include:
- Absolute:
• Prior classical cesarean delivery
• Prior uterine surgery that entered the endometrial cavity, such as myomectomy
• Placenta previa
• Non-reassuring fetal heart rate
• Unexplanied APH
• Multiple pregnancy
• Suspected macrosomia (typically 5000 grams in women without diabetes, 4500
grams in women with diabetes)
• Mechanical obstruction to vaginal birth (eg, large fibroid, severely displaced pelvic
fracture, severe fetal hydrocephalus)
• Uterine rupture
- Relative:
• Early labor
• Oligohydramnios or rupture of membranes
• Known nuchal cord
• Structural uterine abnormalities
• Fetal growth restriction IUGR
• Prior abruption or its risks e.g. preeclampsia

Pregnant at 34 weeks, in labor, examination showed transverse presentation she want to


know if she can do ECV Ultrasound shows bicornuate uterus with normal fetus What is the
contraindication for ECV?
A. Bicornuate uterus
B. Breech presentation
C. History of previous cesarean section

Correct Answer: A
Explanation:

96 | P a g e
Obstetrics and Gynecology: SMLE Questions and Answers Chapter 1: Obstetrics
External Cephalic Version: Should be offered in all cases ≥ 37 weeks who would like to attempt
a vaginal delivery unless there are contraindications.
Contraindications include:
- Absolute:
• Prior classical cesarean delivery
• Prior uterine surgery that entered the endometrial cavity, such as myomectomy
• Placenta previa
• Non-reassuring fetal heart rate
• Unexplanied APH
• Multiple pregnancy
• Suspected macrosomia (typically 5000 grams in women without diabetes, 4500
grams in women with diabetes)
• Mechanical obstruction to vaginal birth (eg, large fibroid, severely displaced pelvic
fracture, severe fetal hydrocephalus)
• Uterine rupture
- Relative:
• Early labor
• Oligohydramnios or rupture of membranes
• Known nuchal cord
• Structural uterine abnormalities
• Fetal growth restriction IUGR
• Prior abruption or its risks e.g. preeclampsia

Note:
- History of previous C-section is not a contraindication unless it’s a classical C-section

37 weeks pregnant came with breech presentation what is your next step?
A. Cesarean section
B. External cephalic version

Correct Answer: B
Explanation:
External Cephalic Version: Should be offered in all cases ≥ 37 weeks who would like to attempt
a vaginal delivery unless there are contraindications.

Lady at 34 weeks of gestation, had previous one C-section, on pelvic exam only the cervix
was 3 cm dilated on US placenta was anterior and laying low. Why is ECV contraindicated
in this case?
A. Gestational age
B. Vaginal exam findings
C. US findings
D. The previous history of C-section

Correct Answer: C
Explanation:

97 | P a g e
Obstetrics and Gynecology: SMLE Questions and Answers Chapter 1: Obstetrics
External Cephalic Version: Should be offered in all cases ≥ 37 weeks who would like to attempt
a vaginal delivery unless there are contraindications.
Contraindications include:
- Absolute:
• Prior classical cesarean delivery
• Prior uterine surgery that entered the endometrial cavity, such as myomectomy
• Placenta previa
• Non-reassuring fetal heart rate
• Unexplanied APH
• Multiple pregnancy
• Suspected macrosomia (typically 5000 grams in women without diabetes, 4500
grams in women with diabetes)
• Mechanical obstruction to vaginal birth (eg, large fibroid, severely displaced pelvic
fracture, severe fetal hydrocephalus)
• Uterine rupture
- Relative:
• Early labor
• Oligohydramnios or rupture of membranes
• Known nuchal cord
• Structural uterine abnormalities
• Fetal growth restriction IUGR
• Prior abruption or its risks e.g. preeclampsia

Pregnant, twins one cephalic and another is breech presentation, how to deliver?
A. Cesarean section
B. Normal delivery

Correct Answer: B
Explanation: (According to ACOG)
Perinatal outcomes for twin gestations in which the first twin is in cephalic presentation are not
improved by cesarean delivery. Thus, women with either cephalic/cephalic-presenting twins or
cephalic/noncephalic-presenting twins should be counseled to attempt vaginal delivery.

Pregnant, twins both cephalic presentation with re-assuring CTG, how to deliver?
A. Cesarean section
B. Normal delivery
C. Ventose
D. Forceps

Correct Answer: B
Explanation: (According to ACOG)

98 | P a g e
Obstetrics and Gynecology: SMLE Questions and Answers Chapter 1: Obstetrics
Perinatal outcomes for twin gestations in which the first twin is in cephalic presentation are not
improved by cesarean delivery. Thus, women with either cephalic/cephalic-presenting twins or
cephalic/noncephalic-presenting twins should be counseled to attempt vaginal delivery.

A pregnant woman with twins, first twin is breech, second twin is cephalic, what is the
management?
A. Spontaneous vaginal delivery
B. ECV then vaginal delivery
C. C-section

Correct Answer: C
Explanation: (According to Uptodate)
We suggest cesarean birth when the first twin is in a noncephalic presentation, which occurs in
approximately 20 percent of twin gestations.

Unbooked female came to the ED with labor, after investigation she has 100,000 colony
bacteria of streptococcus and she has asthma with using salbutamol, what do you wanna
give her now and after deliver?
A. Ampicillin
B. Oxytocin

Correct Answer: A
Explanation: (According to ACOG)
- If GBS bacteriuria at any colony count is detected during pregnancy, the woman is at
increased risk of GBS colonization during labor. A notation should be made in her medical
record, she should be made aware of her GBS status, and antibiotic prophylaxis should be
administered empirically during labor based on the risk factor of antepartum GBS
bacteriuria
- Intravenous penicillin remains the agent of choice for intrapartum prophylaxis, with
intravenous ampicillin as an acceptable alternative.

Pregnant at 39 weeks, now in labor during the delivery you noticed the amniotic fluid is
mixed with dark black-green what is the cause of this color?
A. Meconium aspiration syndrome
B. Fetal distress
C. Placenta abruptio
D. Preterm labor

Correct Answer: B
Explanation: (According to Uptodate)

99 | P a g e
Obstetrics and Gynecology: SMLE Questions and Answers Chapter 1: Obstetrics
- Fetal stress may result in meconium passage, due to increased peristalsis and relaxation of
the anal sphincter from increased vagal outflow associated with umbilical cord compression
or increased sympathetic inflow during hypoxia
- Fetal distress à causes meconium-stained amniotic fluid à which leads to meconium
aspiration syndrome

Female known to have DM, otherwise all normal, pregnant full term, during labor fetus
had tachycardia, how to prevent this?

Question
A. Oxytocin
B. Change mother position
C. Mg gluconate

Correct Answer: B
Explanation:
- Prompt relief of the compromising event, such as correction of maternal hypotension, can
result in fetal recovery.
CTG Categories:
- Category 1
The fetal heart tracing shows All of the following
• Baseline FHR 110-160 BPM
• Moderate variability
• +- Accelerations
• +- Early decelerations

Management:
Routine surveillance

- Category 2
• Include all FHR tracing that are not included in. category1 or 3

Management:
In utero resuscitation measure e.g. lateral positioning of the mother, O2, IV fluid, stop
oxytocin, administer toocolytic drugs, and surveillance

- Category 3
At least one of the following
• Absent variability with recurrent late decelerations
• Absent variability with recurrent variable decelerations
• Absent variability with bradycardia
• Sinusoidal pattern for at least 20 minutes

100 | P a g e
Obstetrics and Gynecology: SMLE Questions and Answers Chapter 1: Obstetrics
Management:
In utero fetal resuscitation measures and prepare of delivery

Epidural anesthesia, what it’s sparring?


A. Perineal
B. Rectum

Correct Answer: B
Explanation:
Epidural analgesia covers the pudendal nerve, which innervates:
• Sensory: The external genitalia of both sexes and the skin around the anus, anal canal and
perineum
• Motor: Pelvic muscles, the external urethral sphincter and the external anal sphincter.

After delivery of the placenta by manual extraction contracting, Retroverted uterus


happened but was back in place. Where was the placenta in the uterus?
A. Anterior
B. Posterior
C. Lateral
D. Fundus

Correct Answer: D
Explanation:
Retroverted uterus means the uterus is tipped backwards (fundus is aimed toward the rectum)
For example, if there’s fibroids in the fundus of the uterus it will cause retroversion of the uterus.

Pregnant women during vaginal delivery, what can make her has fourth degree perineal
tear?
A. Unrestrained legs and squatting position
B. Unrestrained legs and sitting on chair
C. Restrained legs and use of forceps and other metallic instrument

Correct Answer: C
Explanation: (According to ACOG)
The strongest risk factors for OASIS (Obstetric Anal Sphincter Injuries) including forceps
delivery, vacuum- assisted delivery, midline episiotomy, and increased fetal birth weight. Midline
episiotomy combined with forceps delivery substantially increases the risk of third-degree
laceration and fourth-degree laceration. The risk of anal sphincter trauma with operative delivery
and episiotomy is increased in primigravid women and multigravid women.

101 | P a g e
Obstetrics and Gynecology: SMLE Questions and Answers Chapter 1: Obstetrics
Which of the following positions of a patient in labor would most likely result in the
development of a third or fourth-degree laceration?
A. Unrestrained legs and squatting
B. Unrestrained legs and semi setting
C. Unrestrained legs and in chair
D. Restrained legs and stirrups

Correct Answer: C
Explanation: (According to ACOG)
Upright positions (including walking, sitting, standing, and kneeling), were associated with a
possible increase in second-degree perineal tears

Pregnant in labor and signs of meconium stain how to manage baby?


A. Oropharynex suction before delivering the body
B. NICU after delivery
C. Tocolytics
D. Intratracheal suctioning

Correct Answer: B
Explanation: (According to ACOG)
- If the infant born through meconium-stained amniotic fluid presents with poor muscle tone
and inadequate breathing efforts, the initial steps of resuscitation should be completed
under the radiant warmer.
- Appropriate intervention to support ventilation and oxygenation should be initiated as
indicated for each infant.
- Infants with meconium-stained amniotic fluid should no longer routinely receive
intrapartum suctioning, whether they are vigorous or not.
- In addition, meconium-stained amniotic fluid is a condition that requires the notification
and availability of an appropriately credentialed team with full resuscitation skills,
including endotracheal intubation.

Pregegnant lady with dm in active labor ctg show abnormal fetal heart pattern. Which of
the following is the most appropriate next step?
A. Change maternal position
B. Oxygen mask
C. Stop oxytocin
D. Give tocolytic

Correct Answer: A
Explanation: (According to ACOG)
In utero resuscitative measures
- Change of maternal position is a reasonable first treatment option
- Followed by O2

102 | P a g e
Obstetrics and Gynecology: SMLE Questions and Answers Chapter 1: Obstetrics
- IV fluid
- Stop oxytocin
- Administer toocolytic drugs

35 years old pregnant female at 41 weeks gestational age. No contraction, no cervical


dilation. Fetal Ultrasound is reassuring, CTG showed acceleration with good variability
(reassuring). What is the best management?
A. Induction of labor
B. Reassess after 1 week
C. Immediate C-section
D. Labor Augmentation

Correct Answer: B

Another recall

35 years old pregnant female at 42 weeks gestational age. No contraction, no cervical


dilation. Fetal Ultrasound is reassuring, CTG showed acceleration with good variability
(reassuring). What is the best management?
A. Induction of labor
B. Reassess after 1 week
C. Immediate C-section
D. Labor Augmentation

Correct Answer: A

Explanation:
According to ACOG:
Management of Late-Term and Postterm Pregnancies
⁃ Induction of labor between 41 0/7 weeks and 42 0/7 weeks of gestation can be considered.
⁃ Induction of labor after 42 0/7 weeks and by 42 6/7 weeks of gestation is recommended
given evidence of an increase in perinatal morbidity and mortality.

103 | P a g e
Obstetrics and Gynecology: SMLE Questions and Answers Chapter 1: Obstetrics
According to Williams Obstetrics:
⁃ The American College of Obstetricians and Gynecologists (2016a) defines postterm
pregnancies as having completed 42 weeks, namely, beyond 42 °/7 weeks. There is
insufficient evidence to mandate a management strategy between 40 and 42 completed
weeks. Thus, although not considered mandatory, initiation of fetal surveillance at 41
weeks is a reasonable option. After completing 42 weeks, recommendations are for labor
induction

⁃ When gestational age is uncertain, the American College of Obstetricians and


Gynecologists (20 17b) recommends delivery at 41 weeks' gestation using the best clinical
estimate of gestational age. The College also recommends against amniocentesis for fetal
lung maturity.

⁃ At Parkland Hospital, based on results from the trials just discussed, we consider 41-week
pregnancies without other complications to be normal. Thus, no interventions are practiced
solely based on fetal age until 42 completed weeks. With complications such as
hypertension, decreased fetal movement, or oligohydramnios, labor induction is carried out.
It is our view that large, randomized trials should be performed before otherwise
uncomplicated 41-week gestations are routinely considered pathologically prolonged. In
women in whom a certain² gestational age is known, labor is induced at the completion of
42 weeks.

According to UTD:
⁃ Our approach: Induction at 41+0 weeks — We favor induction of postterm pregnancies
at or shortly after 41+0 weeks of gestation irrespective of cervical status, given that
intervention at this time reduces perinatal mortality without increasing perinatal morbidity,
and reduces cesarean delivery rate.

⁃ This approach is in general agreement with professional guidelines that suggest routine
induction between 41+0 and 42+0 weeks of gestation, with the exact timing during this
week determined by clinician and patient preferences and local circumstances. The
American College of Obstetricians and Gynecologists recommends induction of labor after
42+0 weeks and by 42+6 weeks, but considers induction at 41+0 to 42+0 weeks reasonable

Women with 42 weeks of gestation, No contractions, cervical dilation 2, -3 station, normal


CTG, what u want to do?
A. Induction of labor
B. Augmentation of labor
C. C-section

104 | P a g e
Obstetrics and Gynecology: SMLE Questions and Answers Chapter 1: Obstetrics
D. Reassurance

Correct Answer: A
Explanation:
According to ACOG:
Management of Late-Term and Postterm Pregnancies
⁃ Induction of labor between 41 0/7 weeks and 42 0/7 weeks of gestation can be considered.
⁃ Induction of labor after 42 0/7 weeks and by 42 6/7 weeks of gestation is recommended
given evidence of an increase in perinatal morbidity and mortality.

Pregnant at 42week no contraction, you started prostaglandins as IOL and the patient
started to have contractions. Now she is fully dilated cervix, station + 2, baby started to
have bradycardia his heart rate was 75 you couldn't correct it what to do?
A. C-section
B. Start pushing and consider applying vacuum
C. Oxytocin

Correct Answer: B
Explanation: (According to Uptodate)
CTG category2
- Delivery is indicated if resuscitative measures to correct the underlying cause are not
possible or fail to resolve the bradycardia
• Instrumental delivery (Ventose and forceps if the CERVIX IS FULLY DILATED AND
AT STATION +2 and BEYOND)
• C-section (if there is a contraindication to normal vaginal delivery, instrumental
delivery is not applicable “cervix is not fully dilated or station less than +2”, or failed 3
attempts of instrumental delivery)

24 Years-old, Primigravida, known to have gestational diabetes, which is controlled by diet


only, and no other medical problems. She is in the second stage of labor which last more
than 2 hours, regular uterine contractions, baby’s head comes down with each contraction
and go back when uterus is relaxed, the mother’s hip is maximally flexed, one nurse is
applying suprapubic pressure, other nurse applying fundus pressure, the doctor decided to
do episiotomy and deliver the posterior shoulder. Which of the following will cause delay in
delivery?
A. Suprapubic pressure
B. Fundus pressure
C. Hip flexion
D. Delivery of the posterior shoulder

Correct Answer: B
Explanation:

105 | P a g e
Obstetrics and Gynecology: SMLE Questions and Answers Chapter 1: Obstetrics

HELPERR for Shoulder Dystocia


H: Call for Help
E: Evaluate for Episiotomy
L: Legs – McRoberts Maneuver (Hyperflexion of the thighs onto the abdomen and abduction)
P: Suprapubic Pressure
E: Enter: rotational maneuvers
R: Remove the posterior arm
R: Roll the patient to her hands and knees

Pregnant in labor at term. OE (Head in funds and both hip and knees are flexed ) spine of
baby parallel to spine of mother. What is the lie?
A. Longitudinal
B. Transverse
C. Breech
D. Cephalic

Correct Answer: A

Pregnant known case of Bicornute uterus, in leopold manouver you found globally soft
structure in 1&2 grib. Fetal heart in the level of the umbilicus of the mother, Fetal kicking
felt in lower abdomen. How you will deliver this baby?
A. C-section
B. Vaginal delivery
C. Ventose
D. Forceps

Correct Answer: A
Explanation: (According to Uptodate and ACOG)
- Bicornuate uterus is risk factor for breech presentation.
- Leopold Maneuver: The first maneuver assesses the uterine fundus. It permits
identification of fetal lie and determination of which fetal pole occupies the fundus. The
breech gives the sensation of a large, nodular mass, whereas the head feels hard and round
and is more mobile.
- External Cephalic Version: is relatively contraindicated in case of structural uterine
abnormalities

106 | P a g e
Obstetrics and Gynecology: SMLE Questions and Answers Chapter 1: Obstetrics
Case of pregnant lady in labor, vaginal examination revealed palpable orbital edge, nose,
mouth and chin, what is the presentation?
A. Occipital
B. Brow
C. Face

Correct Answer: C
Explanation: (According to Uptodate and Williams Obstetrics)
Face presentation:
- Diagnosis:
• Made by vaginal examination. Palpation of the orbital ridge and orbits, saddle of the
nose, mouth, and chin
• Based on the position of the chin, face presentation can be further divided into mentum
anterior, posterior, or transverse.
- Method of delivery:
• Mentum anterior
Vaginal delivery.
- Labor progress should be closely monitored as arrest of descent may occur,
although not inevitably as in persistent mentum posterior position.
- Oxytocin augmentation and cesarean birth are performed for standard
obstetric indications
• Mentum posterior or transverse
Cesarean delivery
- Attempts to manually convert face presentation to vertex, manual or forceps
rotation of the persistent posterior chin to anterior are contraindicated
- Unless spontaneous rotation to mentum anterior occurs, often late in the
second stage of labor, or the fetus is very small, or the pelvis is very large. If
the fetal status is reassuring and there is normal progress in labor, mentum
posterior presentation can be managed expectantly to see if spontaneous
rotation will occur

107 | P a g e
Obstetrics and Gynecology: SMLE Questions and Answers Chapter 1: Obstetrics
Pregnant at 38 weeks of gestation came with labor. Cervix effaced 80% dilated 6 cm and
there is uterine contraction. Vaginal examination revealed palpation of nose, mouth and
lips. CTG was reactive no abnormality. Which of the following is the most appropriate next
step?
A. Order x-ray pelvimetry
B. Augmentation of labor
C. Cesarean section
D. Monitoring of partograph

Correct Answer: D
Explanation: (According to Uptodate and Williams Obstetrics)
Face presentation:
- Diagnosis:
• Made by vaginal examination. Palpation of the orbital ridge and orbits, saddle of the
nose, mouth, and chin
• Based on the position of the chin, face presentation can be further divided into mentum
anterior, posterior, or transverse.
- Method of delivery:
• Mentum anterior
Vaginal delivery.
- Labor progress should be closely monitored as arrest of descent may occur,
although not inevitably as in persistent mentum posterior position.
- Oxytocin augmentation and cesarean birth are performed for standard
obstetric indications
• Mentum posterior or transverse
Cesarean delivery
- Attempts to manually convert face presentation to vertex, manual or forceps
rotation of the persistent posterior chin to anterior are contraindicated
- Unless spontaneous rotation to mentum anterior occurs, often late in the
second stage of labor, or the fetus is very small, or the pelvis is very large. If
the fetal status is reassuring and there is normal progress in labor, mentum
posterior presentation can be managed expectantly to see if spontaneous
rotation will occur

108 | P a g e
Obstetrics and Gynecology: SMLE Questions and Answers Chapter 1: Obstetrics

Pregnant, full-term came in labor she’s 5cm dilated , 60% effaced, station -3, after 6 hrs of
regular contractions she’s 7 cm dilated, station -1 on vaginal exam, there was retraction
ring , what’s most likely diagnosis?
A. Macrosomia
B. Obstructed labor
C. Preterm labor

Correct Answer: B
Explanation: (According to Uptodate)
Bandl's ring — An hourglass constriction ring of the uterus, called Bandl's ring, has been
estimated to occur in 1 in 5000 live births and is associated with obstructed labor in the second
stage. The constriction forms between the upper contractile portion of the uterus and the lower
uterine segment.

Which of the following is not contraindicated for vaginal delivery?


A. Transverse lie
B. Classical C-section
C. Occipito-posterior

Correct Answer: C
Explanation: (According to AAFP and Uptodate)
Contraindications to vaginal delivery
- Complete placenta previa
- Herpes simplex virus with active genital lesions or prodromal symptoms
- Previous classic uterine incision or extensive transfundal uterine surgery
- Untreated human immunodeficiency virus infection
- Transverse lie

Primigravida in labor, when latent phase considered prolonged?


A. 2 hours
B. 4 hours
C. 8 hours
D. 18 hours

109 | P a g e
Obstetrics and Gynecology: SMLE Questions and Answers Chapter 1: Obstetrics
Correct Answer: D
Explanation: (According to Uptodate)
Friedman considered the latent phase prolonged in nulliparas who had not entered the active phase
by the 95th percentile for duration of the latent phase in pregnant people in spontaneous labor.
- In nulliparas à 20 hours
- In multiparas à 14 hours

Pregnant lady in labor cervical dilation 4 cm, every thing was reassuring except there was
sinusoidal in CTG for 40 minutes what to do?
A. Augmentation of labor
B. C-section
C. Re-assess after 2 hours.

Correct Answer: B
Explanation:
CTG Categories:
- Category 1
The fetal heart tracing shows All of the following
• Baseline FHR 110-160 BPM
• Moderate variability
• +- Accelerations
• +- Early decelerations

Management:
Routine surveillance

- Category 2
• Include all FHR tracing that are not included in. category1 or 3

Management:
In utero resuscitation measure e.g. lateral positioning of the mother, O2, IV fluid, stop
oxytocin, administer toocolytic drugs, and surveillance

- Category 3
At least one of the following
• Absent variability with recurrent late decelerations
• Absent variability with recurrent variable decelerations
• Absent variability with bradycardia
• Sinusoidal pattern for at least 20 minutes

Management:
In utero fetal resuscitation measures and prepare of delivery

Pregnant scheduled for labor induction, what to use for cervical ripening?
A. Vaginal prostaglandin

110 | P a g e
Obstetrics and Gynecology: SMLE Questions and Answers Chapter 1: Obstetrics
B. Vaginal progesterone
C. Methyldopa
D. Methergine

Correct Answer: A
Explanation: (According to ACOG)
Effective methods for cervical ripening include the use of mechanical cervical dilators and
administration of synthetic prostaglandin E1 (PGE1) and prostaglandin E2 (PGE2)

Fetal spine parallel to mother and head at the fundus, the knee is extended and the hip is
flexed, what is the presentation?
A. Complete breech
B. Incomplete breech
C. Frank breech

Correct Answer: C
Explanation: (According to Uptodate)
TYPES OF BREECH PRESENTATION
- Frank breech – Both hips are flexed and both knees are extended so that the feet are
adjacent to the head; accounts for 50 to 70 percent of breech fetuses at term.
- Complete breech – Both hips and both knees are flexed; accounts for 5 to 10 percent of
breech fetuses at term.
- Incomplete breech – One or both hips are not completely flexed

Nurse asked you what is true about CTG?


A. Fetal head compression – Late deceleration
B. Fetal head compression – Variable deceleration
C. Cord compression – Late deceleration
D. Cord compression – Variable deceleration

Correct Answer: D
Explanation:
- Early decelerations à fetal head compression
- Variable decelerations à umbilical cord compression
- Late decelerations à uteroplacental insufficiency.

111 | P a g e
Obstetrics and Gynecology: SMLE Questions and Answers Chapter 1: Obstetrics

What is the cause of this fetal heart tracing?


A. Uteroplacental insufficiency
B. Head compression
Correct Answer: A
Explanation:
- Early decelerations à fetal head compression
- Variable decelerations à umbilical cord compression
- Late decelerations à uteroplacental insufficiency.

Note:
CTG is showing late deceleration.

40 weeks gestation G3P2+0 presented in labour ruptured membrane 5cm cervical


dilatation for 3h 60 % effaced head station - 1. Baby non stress test reassuring mother good
vitals What is the appropriate plan of management?
A. Urgent C-section
B. Observation
C. Augmentation of labor by Oxytocin

Correct Answer: B
Explanation:
Prolonged Active Phase:
- Definition:
≥ 6 cm cervical dilation and one of the following:
• No change in cervical dilation after 6 hours of inadequate contractions
• No change in cervical dilation after 4 hours of adequate contractions
- Managed By:
• Augmentation with Oxytocin for hypotonic contractions (with cervical ripening for
unfavorable cervix 6cm or less)
• Amniotomy (Rupture of membrane)
Arrested Active Phase:
- Definition:
≥ 6 cm cervical dilation with ruptured membranes and no cervical change after one of the
following:

112 | P a g e
Obstetrics and Gynecology: SMLE Questions and Answers Chapter 1: Obstetrics
• ≥ 4 hours of adequate contractions
• > 6 hours of inadequate contractions despite oxytocin administration
- Managed By:
• Cesarean Section!

Let’s Exclude!!
- Urgent C-section à only if she has category 3 CTG or arrested active phase of labor
- Augmentation of labor à if she has prolonged active phase

Female pregnant with recurrent late deceleration and dilated 6cm what to do?
A. Emergency C-section
B. Reassurance
C. Induction of labor

Correct Answer: A
Explanation:
This is category2 CTG, the correct answer should be in utero resuscitation measure e.g. lateral
positioning of the mother, O2, IV fluid, stop oxytocin, administer tocolytic drugs, and surveillance. If not
in the choices, I would go with cesarean section.

Pregnant in 38 week gestation presented with labor, during labor and the fetal head is
engaged at 0 or +2 station you noticed cord with head in cervix which disturb labor. Which
of the following is the most appropriate management?
A. Oxytocin
B. Cesarean section
C. Ventouse
D. Foreceps

Correct Answer: B
Explanation: (According to Uptodate)
MANAGEMENT OF INTRAPARTUM CORD PROLAPSE
- Call for assistance and prepare for emergency delivery; Perform emergency delivery by the
most rapid and safe route, which is typically cesarean
- Initiate maneuvers for intrauterine resuscitation
- Monitor the fetal heart rate

38 weeks in labor with Intrauterine fetal death with regular contraction and cervix 6cm,
what is the management?
A. C-section
B. Augmentation of labor

Correct Answer: B
Explanation:

113 | P a g e
Obstetrics and Gynecology: SMLE Questions and Answers Chapter 1: Obstetrics
She has regular contractions no need for augmentation of labor, if there is Re-assess in 2 hours I
would go with it, if not, augmentation of labor is acceptable by exclusion

38 weeks, she is stable, baby head is at station +3 CTG showing decelerations?


A. C-section
B. Instrumental delivery
C. Reassurance

Correct Answer: B
Explanation:
This is category2 CTG, the correct answer should be in utero resuscitation measure e.g. lateral
positioning of the mother, O2, IV fluid, stop oxytocin, administer tocolytic drugs, and surveillance. If not
in the choices, I would go with instrumental delivery

Cord prolapse what do you see in the CTG?


A. Prolonged deceleration
B. Acceleration
C. Bradycardia

Correct Answer: C
Explanation: (According to Uptodate)
Intrapartum cord prolapse should be suspected in patients with abrupt onset of severe, prolonged
fetal bradycardia or moderate to severe variable decelerations after a previously normal tracing.

37 weeks in labor, contracting regularly, dilated 4cm intact membrane. CTG is normal
except for recurrent variable deceleration. Next step?
A. C-section
B. Rupture the membrane ROM
C. Observation
D. Give tocolytics

Correct Answer: D
Explanation:
This is CTG category 2
Managed by:
In utero resuscitation measure e.g. lateral positioning of the mother, O2, IV fluid, stop
oxytocin, administer tocolytic drugs, and surveillance

Pregnant lady did CTG and showed decelerations ( no other details) What will do initially?
A. Oxygen
B. IV fluid
C. Lateral position
D. C-section

114 | P a g e
Obstetrics and Gynecology: SMLE Questions and Answers Chapter 1: Obstetrics
Correct Answer: C
Explanation: (According to ACOG)
In utero resuscitative measures
- Change of maternal position is a reasonable first treatment option
- Followed by O2
- IV fluid
- Stop oxytocin
- Administer tocolytic drugs
-

Female with breech baby will do C-section when to give the antibiotics.
A. Pre-operative order
B. After closure of the wound
C. Before umbilical cord clamping

Correct Answer: A
Explanation: (According to NICE guidelines)
Timing of antibiotic administration
Offer women prophylactic antibiotics before skin incision for caesarean birth

CTG with prolonged deceleration. What to do?


A. Emergency C-section
B. Hydration, reposition and assess in 20 minutes.

Correct Answer: B
Explanation:
This is CTG category 2
Managed by:
In utero resuscitation measure e.g. lateral positioning of the mother, O2, IV fluid, stop
oxytocin, administer tocolytic drugs, and surveillance

41 weeks +5 days came for induction of labor. The patient had 2 previous uneventful
vaginal delivery,

Bishop score:

- Position: anterior

- Consistency: intermediate

115 | P a g e
Obstetrics and Gynecology: SMLE Questions and Answers Chapter 1: Obstetrics
- Effacement: 50%

- Dilation: 2 cm

- Station: -2

Which method is the most appropriate?


A. Artificial amniotomy
B. Oxytocin infusion
C. Prostaglandin ripening

Correct Answer: C
Explanation: (According to ACOG)
- If induction is indicated and the status of the cervix is unfavorable, agents for cervical ripening may
be used. The status of the cervix can be determined by the Bishop pelvic scoring system.
- An unfavorable cervix generally has been defined as a Bishop score of 6 or less in most randomized
trials.

Patient GA 39 weeks with active labor , fetal tachycardia 180 early declaration which one
is the dangerous?
A. Gestational age
B. Active labor
C. Fetal tachycardia
D. Early decelerations

Correct Answer: C
Explanation:
Fetal tachycardia is category II CTG (non-reassuring)

Primigravida, presented in labor. O/E the cervix is 5cm dilated, the fetus is at 0 station with
cephalic presentation, and in this state for 4 hours even the oxytocin was taken. CTG shows
one variable deceleration. What is the management for this patient?
A. Stop oxytocin.
B. Immediate C-section

116 | P a g e
Obstetrics and Gynecology: SMLE Questions and Answers Chapter 1: Obstetrics
C. Follow up.
D. Instrumental delivery

Correct Answer: A
Explanation:
- She is still 5 cm dilated (latent phase) no need for intervention she can stay in the latent
phase up to 20 hours.
- She has category II CTG
Managed by:
In utero resuscitation measure e.g. lateral positioning of the mother, O2, IV fluid, stop
oxytocin, administer tocolytic drugs, and surveillance

During labor, the midwife suggested a mediolateral episiotomy. What is the advantage of
mediolateral episiotomy?
A. Decrease the 4th degree tear.
B. Better healing
C. Easy repair
D. Less infection

Correct Answer: A
Explanation: (According to ACOG)
Midline episiotomy is a strong independent risk factor for third-degree or fourth-degree lacerations

Lady in labor, Primigravida, and fetus in distressed FHR 70 BPM, station -2, fully dilated,
good regular contractions, what will you do?
A. Ventose
B. Observe
C. Reassurance
D. Encourage to push

Correct Answer: D
Explanation:
Let’s Exclude!
- Ventose à We can’t use instrumental delivery (Ventose and forceps unless the CERVIX IS
FULLY DILATED AND AT STATION +2 and BEYOND!) never ever choose ventose and
forceps in stations that are less than +2.
- Observe or reassurance à she has category II CTG, Delivery is indicated if resuscitative
measures to correct the underlying cause are not possible or fail to resolve.
- Encourage to push à as she has Category II CTG and fully dilated

117 | P a g e
Obstetrics and Gynecology: SMLE Questions and Answers Chapter 1: Obstetrics
Pregnant, multipara in active labor, cervix is 80% effaced, and dilated 7cm, and 0 station,
she has mild irregular contractions, CTG picture showing accelerations, FHR 150 baseline
and moderate variability, what to do?
A. Augment the labor with oxytocin
B. Reposition the mother and resuscitation
C. Waiting

Correct Answer: A
Explanation: (According to Uptodate )
Discontinuation of oxytocin in the active phase
- In patients who achieve a desirable labor pattern and progress, there is no consensus about
whether oxytocin should be discontinued or continued in the active phase, and either
approach is reasonable as long as labor is progressing and the fetal heart rate pattern is
reassuring.
- If discontinued, oxytocin can be restarted if labor progress slows.
Note:
As she has irregular contractions, will augment labor with oxytocin

40 weeks gestation G3 p2 presented in labor ruptured membrane 5cm cervical dilatation


for 3h 60 % effaced head station - 1. Baby non stress test reassuring mother good vitals
What is the appropriate plan of management?

A. Urgent C-section
B. Observation
C. Augmentation of labor by Oxytocin

Correct Answer: B
Explanation:
Lets Exclude!!
- Urgent C-section à if CTG category2 or arrested active phase of labor
- Augmentation of labor à if she has prolonged active phase

118 | P a g e
Obstetrics and Gynecology: SMLE Questions and Answers Chapter 1: Obstetrics

Preterm labor, Preterm Rupture of Membrane (PROM) and Premature Preterm


Rupture of Membrane (PPROM)

38 weeks of gestation with ruptured membrane for 24hrs, no contractions and CTG Is
reassuring, what is the management?
A. C-section
B. Induction of labor

Correct Answer: B
Explanation:
Prelabor Rupture of Membrane (PROM)
- Definition:
• Rupture of membranes before the onset of labor at term (≥37 weeks)

- Investigations:
• Speculum Examination: Pooling of amniotic fluid in the posterior vaginal vault
observed is the gold standard

- Management:
• Induction of labor is recommended.
• Expectant management for up to 12–24 hours is reasonable in otherwise uncomplicated
pregnancy and in the absence of infection.
• GBS Prophylaxis is indicated in case of rupture of membrane for 18hrs and more or
fever 38C!

Woman G2 P1 at 32 weeks’ gestation presents to Emergency Department complaining of


lower abdominal and back pain, which has increased in frequency and intensity over the
last few hour Abdominal examination shows fundal height equals to 32 cm longitudinal lie
fetus and cephalic presentation, Fetal heart was positive and cardiotocography is reactive
with 2 uterine contractions per 10 minutes . Vaginal examination shows dilated cervix, 70
% effacement, -3 station and cephalic. Which of the following is the most appropriate step
in management?

A. Inform neonatologist, administer corticosteroids, and strict bed rest


B. Inform neonatologist, administer corticosteroids, and hydrate the patient
C. Inform neonatologist, administer tocolytics, and start intravenous antibiotics
D. Inform neonatologist, administer intravenous antibiotics, and strict bed rest

Correct Answer: B
Explanation:
Management of Preterm labor:
- ≥34 weeks of gestation
• Admitted for delivery!

119 | P a g e
Obstetrics and Gynecology: SMLE Questions and Answers Chapter 1: Obstetrics
- <34 weeks of gestation
• A course of betamethasone
o A single course of corticosteroids is recommended for pregnant women
between 24 weeks and 34 weeks of gestation who are at risk of delivery
within 7 days.
o A single repeat course of antenatal corticosteroids should, therefore, be
considered in women who are less than 34 weeks of gestation, who are at
risk of preterm delivery within the next 7 days, and whose prior course of
antenatal corticosteroids was administered more than 14 days previously
• Tocolytic drugs for up to 48 hours
o To delay delivery and enable administration of antenatal corticosteroid
• Magnesium sulfate for 24 to 32 weeks of gestation
o Reduces the risk and severity of neurological disorders (e.g., cerebral palsy)

According to ACOG:
Although bed rest and hydration have been recommended to women with symptoms of preterm
labor to prevent preterm delivery, these measures have not been shown to be effective for the
prevention of preterm birth and should not be routinely recommended.

Note:
Our patient is at 32 weeks, which is managed by Corticosteroids and tocolytics alone.
Bed rest is wrong as its not recommended anymore.

Woman almost full term diagnosed as active labor now. She had clear fluid discharge
before her contraction or labor starts, what is the diagnosis?
A. Preterm premature ROM
B. Premature ROM
C. Preterm ROM

Correct Answer: B
Explanation:
- Spontaneous rupture of membrane (SROM) is rupture of membrane at the of labor at term
(≥37 weeks)
- Premature rupture of membranes (PROM) is rupture of membranes before the onset of labor
at term (≥37 weeks)
- Preterm premature rupture of membranes (PPROM) is rupture of membranes before labor
that occurs before 37 weeks of gestation

38 weeks pregnant presented in labor pain with progressive cervical changes. In the labor
room, she had vaginal gush of fluid, what is the diagnosis?

A. Spontaneous rupture of membrane

120 | P a g e
Obstetrics and Gynecology: SMLE Questions and Answers Chapter 1: Obstetrics
B. PROM
C. PPROM

Correct Answer: A
Explanation:
- Spontaneous rupture of membrane (SROM) is rupture of membrane at the of labor at term
(≥37 weeks)
- Premature rupture of membranes (PROM) is rupture of membranes before the onset of
labor at term (≥37 weeks)
- Preterm premature rupture of membranes (PPROM) is rupture of membranes before labor
that occurs before 37 weeks of gestation

PROM How to confirm its amniotic fluid?

A. Sterile speculum examination


B. Antibiotics
C. Chemical testing
D. Fetal hormones

Correct Answer: A
Explanation: (According to Uptodate)
Pooling is the gold standard for diagnosis.
On sterile speculum examination, pooling of amniotic fluid in the posterior vaginal vault is
observed.

Patient 27 weeks in labor, cervix 6cm dilatation. What to give?

A. Steroid
B. Antibiotics
C. Arbostan
D. MgSo4

Correct Answer: A
Explanation: (According to ACOG)
The most beneficial intervention for improvement of neonatal outcomes among patients who give
birth preterm is the administration of antenatal corticosteroids. A single course of corticosteroids is
recommended for pregnant women between 24 weeks and 34 weeks of gestation who are at risk of
delivery within 7 days.

Note:
Both A and D are correct, but I would go with A because it’s more important.

121 | P a g e
Obstetrics and Gynecology: SMLE Questions and Answers Chapter 1: Obstetrics
Healthy lady with Preterm labor what to give her?

A. Indomethacin
B. Magnesium sulfate
C. Nifedipine
D. Terbutaune or nitroglycerin

Correct Answer: A
Explanation: (According to Uptodate)
Tocolysis choice in Preterm Labor:
• First-line:
§ Indomethacin (24-<32 weeks)
§ Nifedipine (32-34 weeks or women who have a contraindication to indomethacin)
• Second-line: if the first-line drug does not inhibit contractions, we discontinue it and begin
therapy with another agent.
§ Nifedipine (24-32 weeks)
§ Terbutaline (32-34 or for those who received nifedipine as a first-line agent at 24 to
32 weeks)

Note:
All answers are correct!! Tocolytic choice depends on the gestational age and maternal
contraindication.
So, in this case I will go with the first choice (Indomethacin)

24year-old lady with rupture of membrane at 34 weeks gestation. What to do first?

A. Tocolytics
B. Steroids
C. Antibiotics

Correct Answer: B
Explanation:
Management of PPROM 34-36 6/7 weeks of gestation
• Either expectant management or immediate delivery is a reasonable option
Expectant management:
§ A course of betamethasone (if steroids not previously given, if proceeding
with induction or delivery in <24 hours and no more than 7 days, and no
evidence of chorioamnionitis)

27 weeks lady with history of PPROM. The CTG is reassuring, normal CBC, she is
medically free, she denies any uterine contractions. Management?

A. Corticosteroids
B. Antibiotics

122 | P a g e
Obstetrics and Gynecology: SMLE Questions and Answers Chapter 1: Obstetrics
C. Tocolytics
D. Go for C-section

Correct Answer: A
Explanation:
Management of PPROM 24-33 6/7 weeks of gestation
• Expectant management
§ A course of betamethasone
§ GBS prophylaxis
§ Tocolytic drugs (Should not be administered for more than 48 hours. Or to
patients who are in advanced labor (>4 cm dilation) or who have any
findings suggestive of subclinical or overt chorioamnionitis.)
§ Magnesium sulfate (if preterm delivery <32 weeks is anticipated”at risk of
imminent delivery”)
• Prompt delivery in:
§ Patients with signs of intrauterine infection, abruptio placentae,
nonreassuring fetal testing, or a high risk of cord prolapse is present or
suspected.

Note:
Both A and B are correct, but A is more important so I will go with it

32 weeks of gestation, she has preterm labor and rupture membrane. She was given
steriods and antibiotics, what to give also?
A. Mg sulphate
B. Oxytocin
C. Nifedipine
D. Indomethacin

Correct Answer: C
Explanation:
Management of PPROM 24-33 6/7 weeks of gestation
• Expectant management
§ A course of betamethasone
§ GBS prophylaxis
§ Tocolytic drugs
(Should not be administered for more than 48 hours. Or to patients who are
in advanced labor (>4 cm dilation) or who have any findings suggestive of
subclinical or overt chorioamnionitis.)
§ Magnesium sulfate
(if preterm delivery <32 weeks is anticipated”at risk of imminent delivery”)

According to ACOG:

123 | P a g e
Obstetrics and Gynecology: SMLE Questions and Answers Chapter 1: Obstetrics
Tocolytic agents can be considered in preterm PROM for steroid benefit to the neonate, especially
at earlier gestational ages, or for maternal transport but should be used cautiously and avoided if
there is evidence of infection or abruption.
Tocolytic therapy is not recommended in the setting of preterm PROM between 34 0/7 weeks of
gestation and 36 6/7 weeks of gestation.

According to UTD:
Tocolysis choice in Preterm Labor:
• First-line:
§ Indomethacin (24-<32 weeks)
§ Nifedipine (32-34 weeks or women who have a contraindication to indomethacin)
• Second-line: if the first-line drug does not inhibit contractions, we discontinue it and begin
therapy with another agent.
§ Nifedipine (24-32 weeks)
§ Terbutaline (32-34 or for those who received nifedipine as a first-line agent at 24 to
32 weeks)

Let’s Exclude!
- MgSo4 à if <32 weeks
- Oxytocin àwhy? I just gave her steroids
- Nifedipine àfirst line tocolytic choice if at 32-34 weeks
- Indomethacin à first line if (24-<32 weeks)

A pregnant female at 31+5 weeks of gestation, presents with preterm premature rupture of
membranes. CTG shows several variable decelerations. What is the next step in
management?

A. Tocolytics And Steroids


B. MgSO4
C. Expectant Management
D. Urgent Delivery

Correct Answer: D
Explanation:
- In Preterm labor or PPROM, Tocolysis is contraindicated in case of non-reassuring fetal
status. And prompt delivery is indicated.

- Route of delivery — In the absence of contraindications to labor and vaginal birth, most
patients will deliver by spontaneous or induced vaginal delivery. Cesarean delivery is
performed for standard indications; otherwise, labor is induced.

- Prompt delivery in:


Patients with signs of intrauterine infection, abruptio placentae, nonreassuring fetal testing, or a
high risk of cord prolapse is present or suspected.

124 | P a g e
Obstetrics and Gynecology: SMLE Questions and Answers Chapter 1: Obstetrics
Emergency delivery because of maternal or fetal status should not be delayed to administer
magnesium sulfate

31 weeks came after having premature PROM 1 week ago, Picture of CTG showing
decreased variability and variable deceleration. On pelvic exam there’s fluid around the os
and 4cm dilated. What to do next?

A. High vaginal swab for culture


B. C-section
C. Betamethasone
D. MgSo4

Correct Answer: B
Explanation:
According to UTD
- Expeditious delivery of women with PPROM is indicated if intrauterine infection, placental
abruption, nonreassuring fetal testing, or a high risk of cord prolapse is present or
suspected.
- Emergency delivery because of maternal or fetal status should not be delayed to administer
magnesium sulfate

According to ACOG
- When is delivery recommended for the preterm fetus in the presence of prelabor rupture of
membranes?
Abnormal results from fetal testing, clinical intraamniotic infection, and significant abruptio
placentae are clear indications for delivery.

Gestational age 31 weeks, known to have PPROM admitted on the 5th day she developed
fever and abdominal pain, GBS 10000 how will you manage?

A. Amoxicillin and gentamycin then deliver.


B. Amoxicillin and gentamycin and observe.
C. Observe
D. Find the cause of fever and treat accordingly

Correct Answer: A
Explanation:
Management of PPROM 24-33 6/7 weeks of gestation
• Expectant management
§ A course of betamethasone
§ GBS prophylaxis

125 | P a g e
Obstetrics and Gynecology: SMLE Questions and Answers Chapter 1: Obstetrics
§ Tocolytic drugs (Should not be administered for more than 48 hours. Or to
patients who are in advanced labor (>4 cm dilation) or who have any
findings suggestive of subclinical or overt chorioamnionitis.)
§ Magnesium sulfate (if preterm delivery <32 weeks is anticipated”at risk of
imminent delivery”)
• Prompt delivery in:
§ Patients with signs of intrauterine infection, abruptio placentae,
nonreassuring fetal testing, or a high risk of cord prolapse is present or
suspected.
• She is presenting with signs of intrauterine infection which is an indication for antibiotic
treatment prompt delivery

Patient with PPROM at 27 weeks present with fever what will u do first?

A. Antibiotics
B. Steroids

Correct Answer: A
Explanation:
Management of PPROM 24-33 6/7 weeks of gestation
• Expectant management
§ A course of betamethasone
§ GBS prophylaxis
§ Tocolytic drugs (Should not be administered for more than 48 hours. Or to
patients who are in advanced labor (>4 cm dilation) or who have any
findings suggestive of subclinical or overt chorioamnionitis.)
§ Magnesium sulfate (if preterm delivery <32 weeks is anticipated”at risk of
imminent delivery”)
• Prompt delivery in:
§ Patients with signs of intrauterine infection, abruptio placentae,
nonreassuring fetal testing, or a high risk of cord prolapse is present or
suspected.
• She is presenting with signs of intrauterine infection which is an indication for antibiotic
treatment prompt delivery

Primigravida, preterm known case of DM 1 came with sever contractions and closed cervix
What to give?

A. Steroids + insulin
B. Steroids + insulin +tocolytics.
C. Steroids and tocolytics
D. Insulin and tocolytics

126 | P a g e
Obstetrics and Gynecology: SMLE Questions and Answers Chapter 1: Obstetrics
Correct Answer: A (by exclusion)
Explanation:
For Pregestational diabetes mellitus:
- ACOG recommends intravenous insulin infusion for intrapartum glycemic management.

For Preterm Contractions Without Cervical Change (ACOG):


- Especially those with a cervical dilation of less than 2 cm, generally should not be treated
with tocolytics.

Preterm 30 weeks, 80% effacement, 2 cm dilatation, stable mom, and fetus. Admitted to
labor. What to do?

Question
A. Call NICU and labor
B. Give dexamethasone, GBS swab, nifedipine and labor
C. Give dexamethasone and labor
D. This is false labor

Correct Answer: B (by exclusion)


Explanation:
Pretem labor <34 weeks of gestation is managed by:
- A course of betamethasone
- Tocolytic drugs for up to 48 hours
- Magnesium sulfate for 24 to 32 weeks of gestation

Pregnant 31+5 weeks normal CTG and normal progression of labor 4 cm dilated then 5 cm
80% effaced. What is the management?

A. Tocolytics + steroids
B. Prostaglandin
C. Reassure

Correct Answer: A
Explanation:
Pretem labor <34 weeks of gestation is managed by:
- A course of betamethasone
- Tocolytic drugs for up to 48 hours
- Magnesium sulfate for 24 to 32 weeks of gestation

A Female at 31wks gestational age presented with cervical dilatation and no uterine
contractions. What is the best thing to do?

127 | P a g e
Obstetrics and Gynecology: SMLE Questions and Answers Chapter 1: Obstetrics
A. Give Bed Rest
B. Do A Cerclage
C. Tocolytics
D. Antibiotics

Correct Answer: A (by exclusion)


Explanation:
Let’s Exclude!
- Bed rest à no longer recommended but I chose it as all the remaining answers are wrong
- Cerclage à not done after 24 weeks
- Tocolytics à she doesn’t have contractions
- Antibiotics à no indication

Patient on tocolytic beta-mimetics (terbutaline), what is the most side effect?

A. Oliguria
B. Palpitation
C. Abdominal pain
D. Vaginal bleeding

Correct Answer: B
Explanation: (According to ACOG)
Terbutaline side effects (beta-adrenergic receptor agonists)
Maternal:
- Tachycardia, hypotension, tremor, palpitation, SOB, chest discomfort, pulmonary edema,
hypokalemia, and hyperglycemia
Fetal:
- Fetal tachycardia

G3P2 with gestational age of 30 weesks. Came with rupture of membrane and has no
contractions. What you’ll give?

A. Ampicilin + oxytocin
B. Dexamethasone + nifedipine
C. Dexamethasone + magnesium sulphate
D. Dexamethasone + erythromycin + ampicilin

Correct Answer: D
Explanation:
Management of PPROM 24-33 6/7 weeks of gestation
• Expectant management
§ A course of betamethasone
§ GBS prophylaxis

128 | P a g e
Obstetrics and Gynecology: SMLE Questions and Answers Chapter 1: Obstetrics
§ Tocolytic drugs (Should not be administered for more than 48 hours. Or to
patients who are in advanced labor (>4 cm dilation) or who have any
findings suggestive of subclinical or overt chorioamnionitis.)
§ Magnesium sulfate (if preterm delivery <32 weeks is anticipated”at risk of
imminent delivery”)
• Prompt delivery in:
§ Patients with signs of intrauterine infection, abruptio placentae,
nonreassuring fetal testing, or a high risk of cord prolapse is present or
suspected.

For GBS Prophylaxis (ACOG):


The recommended antibiotic regimen during expectant management of women with preterm
PROM who are at less than 34 0/7 weeks of gestation
- A 7-day course of therapy of latency antibiotics with a combination of intravenous
ampicillin and erythromycin
- Followed by oral amoxicillin and erythromycin
** Some centers have replaced the use of erythromycin with azithromycin in situations in which
erythromycin is not available or not tolerated.

Tocolysis choice in Preterm Labor( UTD):


• First-line:
§ Indomethacin (24-32 weeks)
§ Nifedipine (32-34 weeks or women who have a contraindication to indomethacin)
Note:
- Magnesium sulfate (if preterm delivery <32 weeks is anticipated “at risk of imminent
delivery)

Preterm labor at 31 weeks, cervix closed what to give?

A. Nifidipine
B. Terbutaline
C. Indomethacin
D. Mgso4

Correct Answer: C
Explanation: (According to Uptodate)
Tocolysis choice in Preterm Labor:
• First-line:
§ Indomethacin (24-<32 weeks)
§ Nifedipine (32-34 weeks or women who have a contraindication to indomethacin)
• Second-line: if the first-line drug does not inhibit contractions, we discontinue it and begin
therapy with another agent.
§ Nifedipine (24-32 weeks)

129 | P a g e
Obstetrics and Gynecology: SMLE Questions and Answers Chapter 1: Obstetrics
§ Terbutaline (32-34 or for those who received nifedipine as a first-line agent at 24 to
32 weeks)

Note:
We give MgSo4 only in case of at risk of imminent delivery

Pregnant lady known to have epilepsy and smoker, then she got premature labor. What is
the risk factor for preterm birth?

A. Smoking
B. Epilepsy

Correct Answer: A
Explanation: (According to Uptodate)
Cigarette smoking — Cigarette smoking has a modest dose-dependent relationship with the risk
for Preterm birth. This effect may be explained by increased rates of smoking-related
complications of pregnancy, such as placental abruption, placenta previa, PROM, and fetal growth
restriction.

Pregnant at 34 week of GA came with labor pain admitted, she had PROM 4 weeks ago,
O/E cervix is 3 cm, there is clear liquor, What is the next step?

A. Emergency C-section
B. Call anesthesia and deliver patient
C. Wait until 37 weeks
D. Give her corticosteroids

Correct Answer: B
Explanation:
Management of Preterm Prelabor Rupture of Membrane (PPROM) Late preterm 34-36 6/7
weeks of gestation
• Either expectant management or immediate delivery is a reasonable option

Pregnant at 30 week with history of clear fluid discharge, CTG shows regular continues
contraction, what confirm that the patient is having preterm delivery?
A. Regular contractions on CTG
B. Digital pelvic exam dilated cervix >3cm
C. History alone is enough to confirm.
D. If there is no passage of clear fluid, then it is not preterm labor

Correct Answer: B
Explanation: (According to Uptodate)
Preterm labor:

130 | P a g e
Obstetrics and Gynecology: SMLE Questions and Answers Chapter 1: Obstetrics
- Cervical dilation >3 cm in the presence of uterine contractions at 20+0 to 36+6 weeks
supports the diagnosis of preterm labor

Pregnant preterm PROM 33 weeks, treated conservatively, then after 3 days she developed
fever and abdominal pain, what is the most appropriate management?

A. Delivery
B. Search for the cause of fever then treat it

Correct Answer: A
Explanation:
Management of PPROM 24-33 6/7 weeks of gestation
• Prompt delivery in:
§ Patients with signs of intrauterine infection, abruptio placentae,
nonreassuring fetal testing, or a high risk of cord prolapse is present or
suspected.

CTG shows fetal tachycardia and prolonged PROM Not in labor, What to do?
A. Repeat cervical examination
B. Give antipyretics and re-assess after 2 hours
C. C-section

Correct Answer: C
Explanation:
Management of PPROM 24-33 6/7 weeks of gestation
• Prompt delivery in:
§ Patients with signs of intrauterine infection, abruptio placentae,
nonreassuring fetal testing, or a high risk of cord prolapse is present or
suspected.

30 years old female at 30 weeks of gestation, 2cm cervical dilatation 50% effacement with
PPROM the doctor give her steroids and tocolytic drugs, what is the purpose of Tocolytic?
A. Delay labor until steroids work
B. Prolong pregnancy until 37 weeks

Correct Answer: A
Explanation:
According to ACOG:
Tocolytic agents can be considered in preterm PROM for steroid benefit to the neonate, especially
at earlier gestational ages, or for maternal transport but should be used cautiously and avoided if
there is evidence of infection or abruption.
Tocolytic therapy is not recommended in the setting of preterm PROM between 34 0/7 weeks of
gestation and 36 6/7 weeks of gestation.

131 | P a g e
Obstetrics and Gynecology: SMLE Questions and Answers Chapter 1: Obstetrics

Rational behind giving steroid for preterm labor?

A. Decrease risk of respiratory distress syndrome


B. Brain protection

Correct Answer: A
Explanation: (According to Uptodate)
Antenatal corticosteroid therapy (ACS) administered to patients at risk for preterm birth reduced
the incidence and severity of respiratory distress syndrome (RDS) and mortality

Female presented with premature rupture of membrane with fever and vaginal discharge.
The uterus is tender to touch. What is the possible diagnosis?
A. Chorioamnionitis
B. UTI

Correct Answer: A
Explanation: (According to Uptodate)
Intraamniotic infection IAI (clinical chorioamnionitis)
- Risk factors:
Longer length of labor and duration of ruptured membranes appears to be the most important risk
factors for IAI. Several other obstetric factors have been associated with an increased risk, including:
• Multiple intrapartum digital vaginal examinations (especially with ruptured membranes)
• Digital rather than speculum examination in patients with preterm prelabor rupture of
membranes (PPROM)
• Cervical insufficiency
• Internal fetal or uterine contraction monitoring
• Intracervical balloon catheter for cervical ripening/labor induction
• Presence of genital tract pathogens (eg, sexually transmitted infections, group B Streptococcus,
bacterial vaginosis)
• Nulliparity
• Meconium-stained amniotic fluid
• Alcohol and tobacco use
• Previous IAI
- Presentation
(IAI often occurs in pregnancies with prelabor rupture of membranes (PROM) but can occur with intact
membranes, especially in laboring patients)
• Fever (100 percent).
• Maternal leukocytosis (white blood cell count >15,000/mm3; 70 to 90 percent).
• Maternal tachycardia >100/min (50 to 80 percent).
• Fetal tachycardia >160/min (40 to 70 percent).
• Uterine tenderness (4 to 25 percent).

132 | P a g e
Obstetrics and Gynecology: SMLE Questions and Answers Chapter 1: Obstetrics
• Bacteremia (5 to 10 percent). Bacteremia is most common when IAI is associated with group
B Streptococcus or Escherichia coli infection (bacteremia in 18 and 15 percent of cases,
respectively).
• Purulent or malodorous amniotic fluid.

A 27 year old pregnant lady, GA 39 came with a gush of watery fluid. On speculum exam
there were pooling of fluids in the vaginal vault. A Nitrazine test shows a PH of 7. Which
of the following is another highly diagnostic test for this condition?
A. Indigo carmine injection into the amniotic sac
B. Methylene blue injection into the amniotic sac
C. US to assess the amount of amniotic fluids
D. Fern test

Correct Answer: D
Explanation: (According to ACOG)
The diagnosis of membrane rupture typically is confirmed by conventional clinical assessment,
which includes the visualization of amniotic fluid passing from the cervical canal and pooling in
the vagina, a simple pH test of vaginal fluid, or arborization (ferning) of dried vaginal fluid, which
is identified under microscopic evaluation. The normal pH of vaginal secretions is generally 3.8–
4.5 whereas amniotic fluid usually has a pH of 7.1–7.3.

Pregnant at 34 weeks in active labor, abdomen is contracting every 5 minutes, Station - 3, 4


cm dilated, membrane intact, CTG is reactive, everything else is normal, what is your next
step?

A. C-section
B. Ventose
C. Rupture membrane
D. Tocolytic

Correct Answer: C
Explanation: (According to ACOG)
Preterm Labor
Regular uterine contractions with cervical effacement, dilation, or both before 37 weeks gestation.
• Management:
- ≥34 weeks of gestation
Admitted for delivery!

Note:
The correct answer should be admit for delivery and rupture membrane after 6cm, I answered C by
exclusion

133 | P a g e
Obstetrics and Gynecology: SMLE Questions and Answers Chapter 1: Obstetrics

Postpartum Hemorrhage (PPH)

During C-section, PPH with failed manual compression and oxytocin what you should do
next?

A. B-lynch
B. Bakri balloon
C. Artery embolization
D. Hysterectomy

Correct Answer: A
Explanation:
- In case of cesarean section à B-Lynch, failed? à Artery Embolization (ONLY if
hemodynamically stable), Hemodynamically unstable? Laparotomy and Artery ligation,
failed? à Hysterectomy
- In case of Vaginal Delivery à Bakri balloon, failed? à Artery Embolization (ONLY if
hemodynamically stable), Hemodynamically unstable? Laparotomy and Artery ligation,
failed? à Hysterectomy

During vaginal delivery, PPH with failed manual compression and oxytocin what you
should do next?

A. B-Lynch
B. Bakri ballon
C. Uterine artery ligation
D. Hysterectomy

Correct Answer: B
Explanation:
- In case of cesarean section à B-Lynch, failed? à Artery Embolization (ONLY if
hemodynamically stable), Hemodynamically unstable? Laparotomy and Artery ligation,
failed? à Hysterectomy
- In case of Vaginal Delivery à Bakri balloon, failed? à Artery Embolization (ONLY if
hemodynamically stable), Hemodynamically unstable? Laparotomy and Artery ligation,
failed? à Hysterectomy

During vaginal delivery, PPH with failed manual compression and oxytocin, and the
patient is hemodynamically unstable (hypotensive) what you should do next?

A. B-Lynch
B. Artery embolization
C. Artery ligation

134 | P a g e
Obstetrics and Gynecology: SMLE Questions and Answers Chapter 1: Obstetrics
D. Hysterectomy

Correct Answer: C
Explanation:
- In case of cesarean section à B-Lynch, failed? à Artery Embolization (ONLY if
hemodynamically stable), Hemodynamically unstable? Laparotomy and Artery ligation,
failed? à Hysterectomy
- In case of Vaginal Delivery à Bakri balloon, failed? à Artery Embolization (ONLY if
hemodynamically stable), Hemodynamically unstable? Laparotomy and Artery ligation,
failed? à Hysterectomy

Pregnant with massive bleeding from abruptio placentae. She is hypotensive and vitally
unstable. What is the most appropriate thing to do to save her life?

A. Admitted to ICU with obstetric team


B. Admitted with different spacilized team
C. 2 peripheral IV cannula and blood transfusion
D. Rapid response team with multidisciplinary intervention

Correct Answer: D
Explanation: (According to ACOG)
- Some emergencies are truly sudden and catastrophic, such as a ruptured aneurysm, massive
pulmonary embolus, or complete abruptio placentae in a trauma setting. However, many
emergencies are preceded by a period of instability during which timely intervention may
help avoid disaster.
- Medical emergency teams—sometimes referred to as “Ob Team Stat” for obstetric
emergencies or a rapid response team—are designated skilled responders who are ready to
intervene during such emergencies.

20 years old pregnant woman presenting with lost of fetal movement followed by decrease
urinary output and difficulty breathing. Her labs are as follow: aPTT prolonged,
Fibrinogen low, Platelets low, what is the diagnosis?

A. Acute glomerulonephritis
B. DIC
C. Autoimmune thrombocytopenia purpura
D. Acute amniotic embolism

Correct Answer: D
Explanation: (According to ACOG)
Amniotic fluid embolism is a rare, unpredictable, unpreventable, and devastating obstetric
emergency signaled by a triad of
1) Hemodynamic compromise
2) Respiratory compromise

135 | P a g e
Obstetrics and Gynecology: SMLE Questions and Answers Chapter 1: Obstetrics
3) Strictly defined disseminated intravascular coagulation.

Note:
- If no respiratory symptoms à DIC
- DIC+respiratory à amniotic fluid embolism

Pregnant in labour, she takes heparin, post delivery she has heavy bleeding? What to give?

A. FFP
B. Protamine sulphate
C. Vitamin K

Correct Answer: B
Explanation:
Neutralization of systemic heparin with protamine sulfate (reverse the anticoagulant effects of
heparin.)

Percentage of maternal death worldwide due to post partum hemorrhage?

A. 10%
B. 15%
C. 20%
D. 25%

Correct Answer: C
Explanation:
Reference: Systematic analysis performed by WHO in 2014, Maternal mortality

A patient with post-partum hemorrhage who was resuscitated then they found that there is
persistent bleeding at several puncture sites what is the next important step?

A. Reversal of coagulopathy
B. Oxytocin
C. Prostaglandin

Correct Answer: A
Explanation:
She is bleeding from the puncture sites à suspecting DIC à FFP and consider cryoprecipitate.

A 28 year old primigravida patient delivered her baby and developed bleeding requiring 1L of
crystelloid, 4 units of blood transfusion and despite all medical measures patient remained
hypotensive and Hb 7.5g/dL and PLT 60. What will you administer in this case?

136 | P a g e
Obstetrics and Gynecology: SMLE Questions and Answers Chapter 1: Obstetrics

A. Platelets
B. Cryoprecipitate

Correct Answer: B
Explanation: (According to Uptodate)
Management of DIC in hemodynamically unstable patients

- Key elements –To maintain oxygenation and perfusion, administer blood products,
frequently monitor hematologic parameters and electrolytes to guide therapy, and avoid
hypothermia, acidosis, and electrolyte abnormalities (potassium and calcium), which can be
lethal.

- Blood products – For patients with heavy bleeding, we order a minimum of 6 units of
packed red blood cells (pRBCs), 6 units of fresh frozen plasma (FFP), 1 dose of, and 10
bags of cryoprecipitate (two pools of 5 units), and begin transfusion of blood products prior
to receiving initial laboratory results. We transfuse pRBCs, FFP, and platelets in a ratio of
1:1:1 in cases of severe hemorrhage. This ratio is expected to reduce the risk of dilutional
coagulopathy
• Platelet transfusions – Patients with serious bleeding or need for urgent/emergent
surgery and a platelet count <50,000/microL should be given platelet transfusions.

Woman in delivery bleeding not stop, she wants to conceive in the future, which structure
you should ligate?

A. Uterosacral ligament
B. External iliac artery
C. Internal iliac artery
D. Uterine vein

Correct Answer: C

Case of PPH on oxytocin, while you manage the case you notice more bleeding, you
examine her but the bleeding prevents you from determining the exact source of bleeding,
what to do?
A. Call for help
B. Misoprostol
C. Oxytocin again
D. Methylergonovine

Correct Answer: A
Explanation:

137 | P a g e
Obstetrics and Gynecology: SMLE Questions and Answers Chapter 1: Obstetrics
First thing to do in PPH is to call for help!

Female after SVD and after delivery of placenta she bleeds heavy amounts what you will do?

A. Collect sample and send for investigations


B. Send to OR to open and see
C. Blood transfusion with O-
D. Send for blood cross-match

Correct Answer: D
Explanation:
According to Williams Obstetrics:
In postpartum Hemorrhage, it is essential to immediately identify uterine atony, retained placental
fragments, or genital tract lacerations. At least one and preferably more large-bore intravenous
infusion systems are established promptly with rapid administration of crystalloid solutions, while
blood is made available

Patient just gave birth, started to bleed heavily, what to do next ?

A. Call for help and ensure cross matching


B. Start blood transfusion with O- blood

Correct Answer: A
Explanation:
First thing to do in PPH is to call for help!

According to Williams Obstetrics:


In postpartum Hemorrhage, it is essential to immediately identify uterine atony, retained placental
fragments, or genital tract lacerations. At least one and preferably more large-bore intravenous
infusion systems are established promptly with rapid administration of crystalloid solutions, while
blood is made available

Patient post delivery massage is done, oxytocin done. She was bleeding. On inspection you
found it is due to laceration 2 cm you tried sutures but it’s not possible due to perfuse
bleeding from above, what is the most appropriate next step?

A. Prostaglandin F2alpha
B. Oxytocin again
C. Suppurative treatment
D. Explore the uterus and examine it

138 | P a g e
Obstetrics and Gynecology: SMLE Questions and Answers Chapter 1: Obstetrics
Correct Answer: D
Explanation:
First thing to do in PPH is call for help. After that to look for the cause of bleeding; uterine atony,
tear, retained products of conception, or coagulopathy, and then manage accordingly.

Patient delivered without episiotomy, placenta was check and all parts were delivered, then
patient had gush of blood coming, what is your next step?

A. Check uterine contraction


B. Get blood for CBC
C. Get blood for coagulation profile
D. Evacuate the uterine

Correct Answer: A
Explanation:
First thing to do in PPH is call for help. After that to look for the cause of bleeding; uterine atony,
tear, retained products of conception, or coagulopathy, and then manage accordingly.

Female, had vaginal delivery, removed placenta completely, shortly after, bleeds profusely,
What is next step in management?
A. Tocolytics
B. Packing
C. Check uterine contractions

Correct Answer: C
Explanation:
First thing to do in PPH is call for help. After that to look for the cause of bleeding; uterine atony,
tear, retained products of conception, or coagulopathy, and then manage accordingly.

Primigravida woman just delivered spontaneously baby is delivered complete and intact.
Massaging of the uterine is performed along with 20 units of oxytocin in 1000 of lactated
Ringers fast drip. inspection of the genital tract, there’s second degree laceration 2-cm left
lateral vaginal wall, suturing is difficult because of bleeding from above the site of laceration. a
soft, boggy uterine fundus Blood pressure 164/92 mmHg Heart rate 130 /min Which of the
following is the best step in management?

A. Prostaglandin f2a
B. Methylergonovine
C. Manual exploration
D. Oxytocin 10 units again

139 | P a g e
Obstetrics and Gynecology: SMLE Questions and Answers Chapter 1: Obstetrics
Correct Answer: A
Explanation:
Medications used in PPH Management:
1) Oxytocin 10-40 units in 500/1000ml NS/RL or 10 units IM (1st line)
2) Methylergonovine 0.2 mg IM (2nd line, Contraindicated in preeclampsia and HTN)
3) Carboprost (hemabate) PG F2a 0.25mg IM (3rd line, Contraindicated in Asthma)
4) Misoprostol 600-1000 micrograms PO, or rectal (4th line)

In case of ongoing hemorrhage and poor uterine response à multiple uterotonic agent should
be used in rapid succession

Lady presenting with vaginal bleeding with fever 15 days after C-section. Most probable
Diagnosis?

A. Wound infection
B. Retained products of conception
C. Endometritis
D. Mastitis

Correct Answer: C
Explanation:
The most important risk factor for endometritis is Cesarean Section!

Retained product of conception will present mainly with heavy bleeding.


Endometritis will give you in the history C-section and fever

Highest rate successful drug used in Postpartum Hemorrhage?

A. Carboprost
B. Oxytocin
C. Misoprostol
D. Methylergonovine

Correct Answer: B

Most common drugs used to treat PPH after Uterine atony ?

A. Oxytocin
B. Carboprost
C. Misoprostol
D. Methylergonovine

140 | P a g e
Obstetrics and Gynecology: SMLE Questions and Answers Chapter 1: Obstetrics

Correct Answer: A

Bleeding after 4 hours of delivery what type of PPH?

A. Primary
B. Secondary

Correct Answer: A
Explanation:
Primary PPH: within 24 hours
Secondary PPH: more than 24 hours after delivery, up to 12 weeks postpartum

In labor after 10 min of delivering the fetus, placenta is still not delivered she started
bleeding?

A. Primary PPH
B. Secondary PPH
C. Tertiary PPH
D. Iatrogenic PPH

Correct Answer: A
Explanation:
Primary PPH: within 24 hours
Secondary PPH: more than 24 hours after delivery, up to 12 weeks postpartum

Patient had PPH she is known case of asthma what medication you can’t give her?

A. Oxytocin
B. Carboprost
C. Misoprostol
D. Methylergonovine

Correct Answer: B
Explanation:
Medications used in PPH Management:
1) Oxytocin 10-40 units in 500/1000ml NS/RL or 10 units IM (1st line)
2) Methylergonovine 0.2 mg IM (2nd line, Contraindicated in preeclampsia and HTN)
3) Carboprost (hemabate) PG F2a 0.25mg IM (3rd line, Contraindicated in Asthma)
4) Misoprostol 600-1000 micrograms PO, or rectal (4th line)

141 | P a g e
Obstetrics and Gynecology: SMLE Questions and Answers Chapter 1: Obstetrics
Patient had PPH she is known case of HTN, what medication you can’t give her?

A. Oxytocin
B. Carboprost
C. Misoprostol
D. Methylergonovine

Correct Answer: D
Explanation:
Medications used in PPH Management:
1) Oxytocin 10-40 units in 500/1000ml NS/RL or 10 units IM (1st line)
2) Methylergonovine 0.2 mg IM (2nd line, Contraindicated in preeclampsia and HTN)
3) Carboprost (hemabate) PG F2a 0.25mg IM (3rd line, Contraindicated in Asthma)
4) Misoprostol 600-1000 micrograms PO, or rectal (4th line)

PPH Medication?

A. Oxytocin 20 units mixed with 500ml D5 IV


B. Ergo 0.5mg IM
C. Ergo 0.2 mg IV

Correct Answer: A
Explanation:
Medications used in PPH Management:
1) Oxytocin 10-40 units in 500/1000ml NS/RL or 10 units IM (1st line) (For patients in whom
infusion of sodium chloride must be avoided, 5 % dextrose solution may be used)
2) Methylergonovine 0.2 mg IM (2nd line, Contraindicated in preeclampsia and HTN)
3) Carboprost (hemabate) PG F2a 0.25mg IM (3rd line, Contraindicated in Asthma)
4)Misoprostol 600-1000 micrograms PO, or rectal (4th line)

Post partum hemorage, which of the following is used to stop bleeding?

A. Misoprostol
B. Propofol
C. Mg sulphate

Correct Answer: A
Explanation:
Medications used in PPH Management:
1) Oxytocin 10-40 units in 500/1000ml NS/RL or 10 units IM (1st line)
2) Methylergonovine 0.2 mg IM (2nd line, Contraindicated in preeclampsia and HTN)
3) Carboprost (hemabate) PG F2a 0.25mg IM (3rd line, Contraindicated in Asthma)
4) Misoprostol 600-1000 micrograms PO, or rectal (4th line)

142 | P a g e
Obstetrics and Gynecology: SMLE Questions and Answers Chapter 1: Obstetrics
Most accurate method to make sure full placental seperation?

A. Pressure and massage uterus


B. Manual removal
C. Wait for spontaneous
D. With a sponge

Correct Answer: B
Explanation:
Ultrasonography or intrauterine manual examination is usually used to diagnose retained placental
tissue.
When a retained placenta is identified, the first step is to attempt manual removal of the tissue.

Which of the following decreases the risk of PPH in a C-section?

A. Curate uterus with sponge


B. Fundal pressure
C. Spontaneous separation of the placenta

Correct Answer: B
Explanation:
Fundal pressure is part of the active management of the 3rd stage to prevent PPH

A young female , the labor was normal but there was a difficulty delivering the placenta
which complicated with uterine inversion accompanied with vaginal bleeding 1200cc.
Which of the following is the source of her bleeding?

A. Trauma from a blood vessel


B. Muscular injury and laceration
C. Uterine muscles fail to contract

Correct Answer: C
Explanation: (According to Uptodate)
Puerperal uterine inversion
Hemorrhage may occur because the invaginated fundus may not contract normally and the inverted
endometrium is stretched, which exacerbates bleeding from any areas of placental separation.

30 years old Female G8P7 in labor for 7 hours, delivered and then had PPH, what is the
reason? in her 30's had PPH after labour for 7 hours whats the reason?

A. Prolonged labor
B. Grand parity
C. Multiple gestation

143 | P a g e
Obstetrics and Gynecology: SMLE Questions and Answers Chapter 1: Obstetrics

Correct Answer: B
Explanation:
- Risk factors for PPH:
• According to Wiliams obstetrics:

• According to ACOG:

Pregnant G2P1+1 delivered after 5 hours duration of labor, the baby weight was 4600g,
after delivering the placenta she started to bleed heavily, what is most likely cause of the
bleeding?

A. Prolonged labor
B. Multiparity
C. History of PPH
D. Macrosomia

144 | P a g e
Obstetrics and Gynecology: SMLE Questions and Answers Chapter 1: Obstetrics

Correct Answer: D
Explanation:
- Risk factors for PPH:
• According to Wiliams obstetrics:

• According to ACOG:

25 years old female, G2P1, her labor was 2 hours long, baby came out before she barely
made it to the hospital, baby weight was 3000. She started bleeding after delivery of the
placenta, what is most likely the cause?

A. Baby weight
B. Prolonged labor
C. Precipitous labor

145 | P a g e
Obstetrics and Gynecology: SMLE Questions and Answers Chapter 1: Obstetrics
D. Grand parity

Correct Answer: C
Explanation:
- Risk factors for PPH:
• According to Wiliams obstetrics:

• According to ACOG:

30 years old G4P3 all SVD without complications, complaining of labor pain, delivered
normal vaginal delivery after 16 hours of labor, baby weight 3000g, she developed PPH,
what is the cause of PPH?

A. Prolonged labor

146 | P a g e
Obstetrics and Gynecology: SMLE Questions and Answers Chapter 1: Obstetrics
B. Grand parity
C. Macrocosmic baby

Correct Answer: A
Explanation:
- Risk factors for PPH:
• According to Wiliams obstetrics:

• According to ACOG:

Mother delivered twins 2 hs ago, now having bleeding. What is the most likely cause?

A. Atony
B. Bleeding disorder
C. Trauma
D. Retained tissue

147 | P a g e
Obstetrics and Gynecology: SMLE Questions and Answers Chapter 1: Obstetrics

Correct Answer: A
Explanation:
- Risk factors for PPH:
• According to Wiliams obstetrics:

• According to ACOG:

Primigravida , delivers healthy baby with uneventful vaginal delivery. After delivery of the
placenta, she’s found to have lost 1000 cc of blood. She looks well, stable and her uterus is
firm. What is the most likely cause for bleeding?
A. Uterine atony
B. Coagulopathy
C. Retained products of conception
D. Genital tract laceration

148 | P a g e
Obstetrics and Gynecology: SMLE Questions and Answers Chapter 1: Obstetrics

Correct Answer: A
Explanation:
Let’s Exclude!
- Uterine atony à Primigravida is a risk factor for uterine atony, the uterus is firm now, but
they did not give me information about the uterus condition when she was bleeding
- Coagulopathy, retained products of conception, genital tract laceration à no risk factors
given in the question

- Risk factors for PPH:


• According to Wiliams obstetrics:

• According to ACOG:

149 | P a g e
Obstetrics and Gynecology: SMLE Questions and Answers Chapter 1: Obstetrics
Female, forceps were used in delivery results in laceration in cervix and vagaina and pph,
what’s the reason for PPH?
A. Atony
B. Retained placenta
C. Genital tract trauma

Correct Answer: C
Explanation:
- Risk factors for PPH:
• According to Wiliams obstetrics:

• According to ACOG:

150 | P a g e
Obstetrics and Gynecology: SMLE Questions and Answers Chapter 1: Obstetrics
37 years old female G8P7 at 36 weeks of gestation, had previous recurrent postpartum
hemorrhage that required blood transfusion. She is medically free. What is the most
appropriate preventable measure in delivery?

A. Deliver the patient in Operation Room


B. Elective C-section at 38 weeks of gestation
C. Start active management of 3rd stage of labor
D. Give preload crystalloid fluid in labor to overcome postpartum hypovolemia

Correct Answer: C
Explanation: (According to ACOG)
Prevention of PPH:
- Many organizations have recommended active management of the third stage of labor as a
method to reduce the incidence of postpartum hemorrhage.
The three components of active management are as follows: 1) oxytocin administration, 2) uterine
massage, and 3) umbilical cord traction

Female with placenta previa, had severe bleeding, what is the most likely outcome post-
delivery?

A. Galactorrhea
B. Diabetes
C. Absence of menstrual cycle
D. Cushing syndrome

Correct Answer: C
Explanation: (According to Uptodate)
Sheehan syndrome (ie, postpartum hypopituitarism)
- Is a rare but potentially life-threatening complication due to postpartum hemorrhage and
hypovolemic shock.
- A common presentation is a combination of failure to lactate postpartum and amenorrhea or
oligomenorrhea, but any of the manifestations of hypopituitarism (eg, hypotension,
hyponatremia, hypothyroidism) can occur any time from the immediate postpartum period
to years after birth.

151 | P a g e
Obstetrics and Gynecology: SMLE Questions and Answers Chapter 1: Obstetrics

Postpartum

Patient developed DVT, had C-section 10 days ago. What to give?

A. Heparin
B. IVC
C. Thrombolytic
D. Embolectomy

Correct Answer: A

30 years old post-partum woman admitted for right leg DVT and was started on enoxaparin
80mg BID. Then she developed sudden onset dyspnea and right pleuritic chest pain, on PE; She
was dyspneic and apprehensive, heart sound showed loud P2 and lungs were clear on
auscultation, vital signs normal

ABG: normal HCO3 and Po2, decreased PCO2 and high pH

CT showed thrombus in right lower pulmonary artery

Which of the following is most appropriate step in management?

A. Switch Enoxparin to sodium heparin


B. Thrombolytic therapy
C. Same management
D. Thromboectomy

Correct Answer: C
Explanation:
She was admitted for DVT, so she is already on the therapeutic dose, NO NEED to change the
treatment, unless if she was UNSTABLE (hypotensive) à in that case I would go for B.
Thrombolytic therapy

Normal delivery and she did episiotomy the developed perineal collection 5x3, which was bluish
and painful, what is the treatment?

A. Packing
B. Aspiration
C. Surgical evacuation
D. Observe

152 | P a g e
Obstetrics and Gynecology: SMLE Questions and Answers Chapter 1: Obstetrics
Correct Answer: C
Explanation:
Vaginal hematoma:
Surgical evacuation when (large) 5cm and more, expanding or symptomatic (painful), otherwise
Observation with (RICE) Rest, Ice, Compression and Elevation.

Patient after SVD found placenta failed to deliver and cannot extract. The patient refused
hysterectomy. It was managed by ligating the placenta and started on Methotrexate therapy,
what’s the complication of this case?

A. Bleeding
B. Infection
C. DIC

Correct Answer: A
Explanation:
- The patient was managed by conservative management for placenta accreta.
- The complications of uterine conservation with placenta left in situ is:
• Most commonly is severe vaginal bleeding (53%). Followed by sepsis “which can lead
to DIC” (6%).

153 | P a g e
Chapter 2: Gynecology

154 | P a g e
Obstetrics and Gynecology: SMLE Questions and Answers Chapter 2: Gynecology

Puberty Disorders, Pediatrics, Adolescent and Young Gynecology

A 17-Year-old female, no menstruation, high testosterone, Normal breast development, coarse


pubic hair?

A. Mayer Rokitansky Kuster Hauser syndrome (or Mullerian agenesis)


B. Complete androgen insensitivity
C. Congenital hypothyroidism

Correct Answer: B
Explanation: (According to UpTodate and ACOG)
Vaginal agenesis (Mayer-Rokitansky-Kuster-Hauser syndrome)( Müllerian agenesis )
- Clinical presentation:
• NORMAL female karyotype with normal ovaries and ovarian function, thus they
develop normal secondary sexual characteristics (eg, breast development, axillary
hair, and public hair).
• PRESENTS ONLY with primary amenorrhea
- Investigations:
• Testosterone level: NORMAL “is one way to differentiate between Mayer-
Rokitansky-Küster-Hauser (MRKH) syndrome and androgen insensitivity.”

Complete androgen insensitivity (CAIS)


- Clinical presentation:
• Primary amenorrhea
• Little or no axillary and pubic hair (coarse pubic hair).
- Investigations:
• Testosterone level: within or above the normal range for boys/men. (High
testosterone levels for a women)

A 17-Year-old female, no menstruation, normal testosterone, Normal breast development,


normal pubic hair?

A. Mayer Rokitansky Kuster Hauser syndrome (or Mullerian agenesis)


B. Complete androgen insensitivity
C. Congenital hypothyroidism

Correct Answer: A
Explanation: (According to UpTodate and ACOG)
Vaginal agenesis (Mayer-Rokitansky-Kuster-Hauser syndrome)( Müllerian agenesis )
- Clinical presentation:

155 | P a g e
Obstetrics and Gynecology: SMLE Questions and Answers Chapter 2: Gynecology
• NORMAL female karyotype with normal ovaries and ovarian function, thus they
develop normal secondary sexual characteristics (eg, breast development, axillary
hair, and public hair).
• PRESENTS ONLY with primary amenorrhea
- Investigations:
• Testosterone level: NORMAL “is one way to differentiate between Mayer-
Rokitansky-Küster-Hauser (MRKH) syndrome and androgen insensitivity.”

Complete androgen insensitivity (CAIS)


- Clinical presentation:
• Primary amenorrhea
• Little or no axillary and pubic hair (coarse pubic hair).
- Investigations:
• Testosterone level: within or above the normal range for boys/men. (High
testosterone levels for a women)

A 17-year-old female, medically free, (athlete) gymnast in her class, breasts later and never
menstruated, on developed examination she is tanner stage 5, but no menstruation, diagnosis?

A. Hypothalamic hypogonadism
B. Transverse vaginal septum
C. Gonadal agenesis
D. Testicular feminization

Correct Answer: A
Explanation: (According to ACOG)
- Menstrual cycle disturbances (e.g. amenorrhea) in young women ESPECIALLY
ATHLETES!! Can be due to underlying eating disorders and dietary restrictions à
resulting into hypogonadotropic hypogonadism!
- Please read the explanation below by ACOG:

- Transverse vaginal septum will present with other symptoms(in addition to amenorrhea),
e.g., periodic lower abdominal pain, urinary retention and bulging bluish membrane!

156 | P a g e
Obstetrics and Gynecology: SMLE Questions and Answers Chapter 2: Gynecology

Menstrual Cycle Abnormalities and Abnormal Uterine Bleeding (AUB)

Patient came with heavy bleeding after doing evacuation the physician said it’s
dysfunctional vaginal bleeding what to do next?

D. D&C
E. OCPs
F. Embolization
G. Hysterectomy

Correct Answer: B
Explanation: (According to Williams Gynecology and ACOG)
Management of Acute AUB in reproductive aged women

- At times, women with AUB may have brisk bleeding that requires acute intervention
(Hemodynamically unstable) à Start with fluid resuscitation and blood transfusion as
indicated first!! à Then Medical treatment is simultaneously administered to slow
bleeding.
- Medical Management should be the initial treatment for most patients with acute AUB
• Treatment options include:
o IV conjugated equine estrogen: Primary choice for heavy bleeding (Once
bleeding has slowed, patients can be transitioned to an oral taper using
Prernarin pills or more commonly COCs.)
o Combined oral contraceptives (OCs): Primarily for less severe bleeding
o Oral progestins: Primarily for less severe bleeding and when estrogen is
contraindicated
o Tranexamic acid (TXA): is also an option for acute heavy menstrual
bleeding (HMB).

- Therapeutic D&C should be reserved as a last resort for the rare patient who continues to
have life-threatening bleeding despite high-dose of estrogen administration

Management of Chronic AUB in reproductive aged women


⁃ Levonorgestrel-Releasing IUD (Mirena): First line treatment for heavy menstrual bleeding
(HMB).
⁃ Combined oral contraceptives (OCs)
⁃ Depot medroxyprogesterone acetate
⁃ Norethindrone acetate (progestins)
⁃ Tranexamic acid (TXA)
⁃ NSAID
• Mefenamic acid
• Naproxen
• Ibuprofen
• Flurbiprofen

157 | P a g e
Obstetrics and Gynecology: SMLE Questions and Answers Chapter 2: Gynecology
- GnRH agonists

A 28-years-old, came to ER with heavy vaginal bleeding, she is nulliparous, pregnancy test
negative, no pain, regular cycle, but always with menorrhagia, how to stop the bleeding
now?

A. Give conjugated estrogen


B. Give progesterone
C. Give GnRH
D. Insert levenogestrel IUD

Correct Answer: A
Explanation: (According to Williams Gynecology and ACOG)
Management of Acute AUB in reproductive aged women

- At times, women with AUB may have brisk bleeding that requires acute intervention
(Hemodynamically unstable) à Start with fluid resuscitation and blood transfusion as
indicated first!! à Then Medical treatment is simultaneously administered to slow
bleeding.
- Medical Management should be the initial treatment for most patients with acute AUB
• Treatment options include:
o IV conjugated equine estrogen: Primary choice for heavy bleeding (Once
bleeding has slowed, patients can be transitioned to an oral taper using
Prernarin pills or more commonly COCs.)
o Combined oral contraceptives (OCs): Primarily for less severe bleeding
o Oral progestins: Primarily for less severe bleeding and when estrogen is
contraindicated
o Tranexamic acid (TXA): is also an option for acute heavy menstrual
bleeding (HMB).

- Therapeutic D&C should be reserved as a last resort for the rare patient who continues to
have life-threatening bleeding despite high-dose of estrogen administration

Management of Chronic AUB in reproductive aged women


⁃ Levonorgestrel-Releasing IUD (Mirena): First line treatment for heavy menstrual bleeding
(HMB).
⁃ Combined oral contraceptives (OCs)
⁃ Depot medroxyprogesterone acetate
⁃ Norethindrone acetate (progestins)
⁃ Tranexamic acid (TXA)
⁃ NSAID
• Mefenamic acid
• Naproxen
• Ibuprofen
• Flurbiprofen

158 | P a g e
Obstetrics and Gynecology: SMLE Questions and Answers Chapter 2: Gynecology
- GnRH agonists

A 40-Year-old female, with heavy vaginal bleeding came to ER, what to do?

A. IUCD
B. D&C
C. Hysterectomy
D. Mefenamic acid

Correct Answer: B
Explanation: (According to Williams Gynecology and ACOG)
Management of Acute AUB in reproductive aged women

- At times, women with AUB may have brisk bleeding that requires acute intervention
(Hemodynamically unstable) à Start with fluid resuscitation and blood transfusion as
indicated first!! à Then Medical treatment is simultaneously administered to slow
bleeding.
- Medical Management should be the initial treatment for most patients with acute AUB
• Treatment options include:
o IV conjugated equine estrogen: Primary choice for heavy bleeding (Once
bleeding has slowed, patients can be transitioned to an oral taper using
Prernarin pills or more commonly COCs.)
o Combined oral contraceptives (OCs): Primarily for less severe bleeding
o Oral progestins: Primarily for less severe bleeding and when estrogen is
contraindicated
o Tranexamic acid (TXA): is also an option for acute heavy menstrual
bleeding (HMB).

- Therapeutic D&C should be reserved as a last resort for the rare patient who continues to
have life-threatening bleeding despite high-dose of estrogen administration

Management of Chronic AUB in reproductive aged women


⁃ Levonorgestrel-Releasing IUD (Mirena): First line treatment for heavy menstrual bleeding
(HMB).
⁃ Combined oral contraceptives (OCs)
⁃ Depot medroxyprogesterone acetate
⁃ Norethindrone acetate (progestins)
⁃ Tranexamic acid (TXA)
⁃ NSAID
• Mefenamic acid
• Naproxen
• Ibuprofen
• Flurbiprofen
- GnRH agonists

159 | P a g e
Obstetrics and Gynecology: SMLE Questions and Answers Chapter 2: Gynecology
Let’s Exclude!!
- IUD à first line for chronic HMB
- D&C à in case of acute bleeding
- Hysterectomy à why?
- Mefanamic acid à Also for chronic HMB but not the first line!

A female patient present with heavy PV bleeding. Her bleeding is associated with pain and
of large volume. Upon vaginal examination, you noticed pooling of blood. Pregnancy test is
negative, her BP low. Labs: RBC low, Hb low. What is your next step in management?

A. Progesterone
B. Conjugated estrogen
C. Blood transfusion

Correct Answer: C
Explanation:
- In hemodynamically unstable patients à Start with fluid resuscitation and blood
transfusion as indicated first!! à Then Medical treatment is simultaneously administered to
slow bleeding (IV conjugated equine estrogen)

Young female presented to the ER complaining of general fatigue symptoms and dizziness
and mention that she is bleeding for 15 days since her period started. No mention of
Examination or current bleeding status. Lab: Hb 7 What is the most appropriate next
step?

A. US
B. Start blood transfusion
C. Stop the bleeding and send her home

Correct Answer: B
Explanation:
- In hemodynamically unstable patients à Start with fluid resuscitation and blood
transfusion as indicated first!! à Then Medical treatment is simultaneously administered to
slow bleeding (IV conjugated equine estrogen)

Patient with uterine bleeding. What is the most appropriate investigation?

A. CBC
B. TFT
C. B-HCG
D. US

160 | P a g e
Obstetrics and Gynecology: SMLE Questions and Answers Chapter 2: Gynecology
Correct Answer: C
Explanation: (According to ACOG)
To Exclude pregnancy
Pregnancy should be excluded in all reproductive-age patients with AUB

Patient with intermenstrual bleeding. What is the most appropriate investigation?

A. CBC
B. TFT
C. B-HCG
D. US

Correct Answer: D
Explanation: (According to ACOG)
Intermenstrual Bleeding is mainly caused my uterine and cervical etiologies.

Lets Exclude!
- CBC à would be initial
- TFH à would be initial (Thyroid dysfuction causes oligo- or amenorrhea or menorrhagia
and NOT intermenstrual bleeding)
- B-HCG à its intermenstrual bleeding (between menstruations), I would go for it if its only
bleeding or AUB, but the type of bleeding is specified in the question.
- US à is the primary imaging is the primary imaging test of the uterus for the evaluation of
AUB (ACOG)

Female patient complaining of pain before menses and resolved in the third day of menses,
how do you diagnose it?

A. Hysteroscopy
B. Abdominal US
C. Clinical symptoms

Correct Answer: C
Explanation: (According to ACOG)
Primary Dysmenorrhea:
Is the cramping pain that comes before or during a period pain tends to lessen after the first few
days of a period.
In the absence of pelvic pathology.
- Evaluation include:
• History to determine whether the patient has primary dysmenorrhea or symptoms
suggestive of secondary dysmenorrhea.
• When a patient presents with symptoms only of primary dysmenorrhea, a pelvic
examination is not necessary.
- Management:
161 | P a g e
Obstetrics and Gynecology: SMLE Questions and Answers Chapter 2: Gynecology
• NSAID (first line)
• OCP (second line, if a trial of NSAIDs does not provide adequate relief of
dysmenorrhea symptoms)

Secondary dysmenorrhea:
Is caused by a disorder in the reproductive organs. The pain tends to get worse over time and it
often lasts longer than normal menstrual cramps.
- Causes include:
• Endometriosis diagnosed by histologic evaluation of a lesion biopsied during
surgery (typically laparoscopy)
• Adenomyosis diagnosed by histology after hysterectomy

A 19-Year-old female, complaining of dysmenorrhea resolve on 3rd day of menses,


symptoms associated with sever pain radiated to upper thigh, she had this for several years
and getting worse, what is the diagnosis?

A. Primary dysmenorrhea
B. Premenstrual
C. secondary dysmenorrhea
D. Endometriosis

Correct Answer: A
Explanation: (According to ACOG)
Primary Dysmenorrhea:
Is the cramping pain that comes before or during a period pain tends to lessen after the first few days
of a period.
In the absence of pelvic pathology.
- Evaluation include:
• History to determine whether the patient has primary dysmenorrhea or symptoms
suggestive of secondary dysmenorrhea.
• When a patient presents with symptoms only of primary dysmenorrhea, a pelvic
examination is not necessary.
- Management:
• NSAID (first line)
• OCP (second line, if a trial of NSAIDs does not provide adequate relief of
dysmenorrhea symptoms)

Secondary dysmenorrhea:
Is caused by a disorder in the reproductive organs. The pain tends to get worse over time and it often
lasts longer than normal menstrual cramps.
- Causes include:
• Endometriosis diagnosed by histologic evaluation of a lesion biopsied during surgery
(typically laparoscopy)
• Adenomyosis diagnosed by histology after hysterectomy

162 | P a g e
Obstetrics and Gynecology: SMLE Questions and Answers Chapter 2: Gynecology

A 35-Year-old female complaining of secondary ammenorrhea for 8 months, what to do next?

A. Pregnancy test
B. US
C. Full history taking
D. Physical exam

Correct Answer: C

Patient with severe abdominal pain with menstrual cycle (dysmenorrhea), affecting her
work, what can you give?

A. Misoprostol
B. NSAID
C. Progesterone
D. OCP

Correct Answer: B
Explanation: (According to ACOG)
Management of Primary Dysmenorrhea
- NSAID (first line)
- OCP (second line, if a trial of NSAIDs does not provide adequate relief of dysmenorrhea
symptoms)

Patient with severe abdominal pain with menstrual cycle (dysmenorrhea), affecting her
work, what can you give?

A. Misoprostol
B. Paracetamol
C. Progesterone
D. OCP

Correct Answer: D
Explanation: (According to ACOG)
Management of Primary Dysmenorrhea
- NSAID (first line)
- OCP (second line, if a trial of NSAIDs does not provide adequate relief of dysmenorrhea
symptoms)

163 | P a g e
Obstetrics and Gynecology: SMLE Questions and Answers Chapter 2: Gynecology
A 17-Years-old female came complaining of primary dysmenorrhea. How will you
manage?

A. COCPs
B. NSAIDs
C. Mefanimic acid

Correct Answer: B
Explanation: (According to ACOG)
Management of Primary Dysmenorrhea
- NSAID (first line)
- OCP (second line, if a trial of NSAIDs does not provide adequate relief of dysmenorrhea
symptoms)

A 26-year-old female came to the Clinic complaining of severe lower abdominal pain tat
started just before her menses and disappeared on the 3rd day, the pain so severe that
affect her daily activity and prevent her from going to work for many days. Her boss is not
happy. Which of the following is the most appropriate management?

A. Hysterectomy
B. Paracetamol
C. OCP

Correct Answer: C
Explanation: (According to ACOG)
Management of Primary Dysmenorrhea
- NSAID (first line)
- OCP (second line, if a trial of NSAIDs does not provide adequate relief of dysmenorrhea
symptoms)

A 17-year-old female, complaining of primary dysmenorrhea severe pain affect on life an


school attendance. She was on NSAID and pain become less severe, she able to go school
and do her life activities. What is the most appropriate next step?

A. OCP
B. Counselling and education of self-care

Correct Answer: B
Explanation: (According to UpToDate)
The initial approach of dysmenorrhea includes a discussion of nonpharmacologic interventions that
can be helpful, such as exercise and application of a heat pack to the lower abdomen

164 | P a g e
Obstetrics and Gynecology: SMLE Questions and Answers Chapter 2: Gynecology
Female complaining of breast tenderness, severe depression, inability socalize, anxiety,
crying, and its affecting her and her work. It starts 1 week before menses disappear in 3
day, what is the most appropriate management?

A. Transdermal estrogen patch


B. SSRIs
C. OCP

Correct Answer: B
Explanation: (According to ACOG and UpTodate)
Premenstrual syndrome PMS
- Definition:
• At least one symptom associated with "economic or social dysfunction" that occurs
during the five days before the onset of menses and is present in at least three
consecutive menstrual cycles.
Symptoms may be affective (eg, angry outbursts, depression) or physical (eg, breast
pain and bloating)

Premenstrual Dysphoric Disorder PMDD


- Is a severe type of PMS, If PMS symptoms are severe and cause problems with work or
personal relationships

Management of PMS and PMDD:


- Mild symptoms
• Lifestyle measures such as regular exercise and stress reduction techniques.
- Moderate-severe symptoms
• Women who do not desire contraception
o Selective serotonin reuptake inhibitors SSRI
• Women who desire contraception
o Combined estrogen-progestin contraception
If symptom relief with the COC monotherapy is incomplete, an SSRI can be
added

A 32-year-old came to gynecology clinic for her dysmenorrhea. Which of the following is
history evaluation is importent? Question

A. Medical
B. Surgical
C. Family
D. Menstrual

Correct Answer: D

165 | P a g e
Obstetrics and Gynecology: SMLE Questions and Answers Chapter 2: Gynecology

A 14 year-old female complain of vaginal bleeding in interval of 3 weeks to 2 months, she


has normal development and normal secondary features, every thing normal. What to do?

A. Reassure
B. OCP
C. Order FSH and prolactin

Correct Answer: A
Explanation: (According to UpToDate)
Anovulatory uterine bleeding
- During the first one to two years after menarche, the most common cause of irregular
bleeding in nonpregnant adolescents is anovulatory uterine bleeding due to an immature
hypothalamic-pituitary-ovarian axis
- Laboratory evaluation is not necessary
- If pathologic causes are ruled out, and the patient is not bothered by irregular menses,
anovulatory bleeding may be managed expectantly for the first few years after menarche.

A 14-year-old female came with intermenstrual bleeding. What is the problem?

A. Metabolic
B. Endocrine
C. Genetics

Correct Answer: B
Explanation: (According to UpToDate)
Anovulatory uterine bleeding
- During the first one to two years after menarche, the most common cause of irregular
bleeding in nonpregnant adolescents is anovulatory uterine bleeding due to an immature
hypothalamic-pituitary-ovarian axis

A 36-year-old lady with secondary amenorrhea (elevated FSH & LH) which of risk or
complication she might probably develop in the future?

A. Risk of endometrial cancer


B. Risk of ovarian cancer
C. Risk of osteoporosis

Correct Answer: C
Explanation: (According to UpToDate)

166 | P a g e
Obstetrics and Gynecology: SMLE Questions and Answers Chapter 2: Gynecology
Primary ovarian insufficiency: The diagnosis of primary ovarian insufficiency (POI) can be
made definitively in women younger than 40 years with irregular menses in association with
follicle-stimulating hormone (FSH) concentrations in the postmenopausal range (high)

- High FSH levels with amenorrhea indicates spontaneous primary ovarian failure à which
is a risk for osteoporosis.

A 27-year-old complaining of 9 months amenorrhoea, investigation showed high LH and


FSH. What is the cause of her presntation?

A. PCOS
B. Premature ovarian failue
C. Ovarian cyst

Correct Answer: B
Explanation: (According to UpToDate)
Primary ovarian insufficiency: The diagnosis of primary ovarian insufficiency (POI) can be
made definitively in women younger than 40 years with irregular menses in association with
follicle-stimulating hormone (FSH) concentrations in the postmenopausal range (high)

Which of the following indicate bleeding from a uterine source?

A. Blood color
B. Scanty
C. Heavy bleeding

Correct Answer: C
Explanation: (According to ACOG)
The overarching term AUB is paired with descriptive terms to denote bleeding patterns associated
with AUB, such as heavy menstrual bleeding (instead of menorrhagia) and intermenstrual bleeding
(instead of metrorrhagia).

What defines a uterine bleeding?

A. Postcoital
B. Scanty bleeding
C. Perfuse bleeding.

Correct Answer: C
Explanation: (According to ACOG)
The overarching term AUB is paired with descriptive terms to denote bleeding patterns associated
with AUB, such as heavy menstrual bleeding (instead of menorrhagia) and intermenstrual bleeding
(instead of metrorrhagia).

167 | P a g e
Obstetrics and Gynecology: SMLE Questions and Answers Chapter 2: Gynecology

168 | P a g e
Obstetrics and Gynecology: SMLE Questions and Answers Chapter 2: Gynecology

Vaginal Infections

Patient presenting with yellowish to green vaginal discharge, strawberry cervix, what is the
vaginal infection she has?

A. Bacterial vaginosis
B. Trichomoniasis
C. Candida
D. Gonorrhea

Correct Answer: B
Explanation:
Trichomoniasis
- Clinical presentation:
• Frothy, yellow-green discharge. Foul-smelling, vaginal itchiness, strawberry cervix
- Microscope:
• Flagellated protozoa
- Treatment:
• Metronidazole, Treat Sexual partner(s)

Women came vaginal itchiness test show flagellated cells what's the diagnosis?

A. Bacterial vaginosis
B. Candida infection
C. Trichomanias vaginalis
D. E coli infection

Correct Answer: C
Explanation:
Trichomoniasis
- Clinical presentation:
• Frothy, yellow-green discharge. Foul-smelling, vaginal itchiness, strawberry cervix
- Microscope:
• Flagellated protozoa
- Treatment:
• Metronidazole, Treat Sexual partner(s)

Female, 27 years old complaining of green-yellow discharge that started after her mensuration,
Upon examination: red lesions on cervix, What’s the diagnosis?

169 | P a g e
Obstetrics and Gynecology: SMLE Questions and Answers Chapter 2: Gynecology

A. Atrophic vaginitis
B. Bacterial vaginosis
C. Candidiasis
D. Trichomoniasis

Correct Answer: D
Explanation:
Trichomoniasis
- Clinical presentation:
• Frothy, yellow-green discharge. Foul-smelling, vaginal itchiness, strawberry cervix
- Microscope:
• Flagellated protozoa
- Treatment:
• Metronidazole, Treat Sexual partner(s)

Female had trichomoniasis. What’s prophylactic for husband?

A. Metronidazole
B. No need
C. Doxycycline
D. Azithromycin

Correct Answer: A
Explanation:
Trichomoniasis
- Clinical presentation:
• Frothy, yellow-green discharge. Foul-smelling, vaginal itchiness, strawberry cervix
- Microscope:
• Flagellated protozoa
- Treatment:
• Metronidazole, Treat Sexual partner(s)

Question about trichomoniasis treatment?

A. Metronidazole patient alone


B. Fluconazole
C. Metronidazole patient & partner
D. Ceftriaxone

Correct Answer: C
Explanation:

170 | P a g e
Obstetrics and Gynecology: SMLE Questions and Answers Chapter 2: Gynecology
Trichomoniasis
- Clinical presentation:
• Frothy, yellow-green discharge. Foul-smelling, vaginal itchiness, strawberry cervix
- Microscope:
• Flagellated protozoa
- Treatment:
• Metronidazole, Treat Sexual partner(s)

Patient complaining of yellow green discoloration and husband has urethral discharge, what is the
most appropriate management?

A. Fluconazole
B. Metronidazole patient only
C. Metronidazole patient & partner
D. Ceftriaxone

Correct Answer: C
Explanation:
Trichomoniasis
- Clinical presentation:
• Frothy, yellow-green discharge. Foul-smelling, vaginal itchiness, strawberry cervix
- Microscope:
• Flagellated protozoa
- Treatment:
• Metronidazole, Treat Sexual partner(s)

A 35-year-old female complaining of greenish discharge, cervix: red spots. What is the most
appropriate treatment?

A. Doxycycline
B. Metronidazole
C. Trimethoprim-Sulfamethoxazole
D. Ceftriaxone

Correct Answer: B
Explanation:
Trichomoniasis
- Clinical presentation:
• Frothy, yellow-green discharge. Foul-smelling, vaginal itchiness, strawberry cervix
- Microscope:
• Flagellated protozoa
171 | P a g e
Obstetrics and Gynecology: SMLE Questions and Answers Chapter 2: Gynecology
- Treatment:
• Metronidazole, Treat Sexual partner(s)

White vaginal discharge but no rash, no odor. What is the organism/name of disease?

A. Bacterial vaginosis
B. Trichomoniasis
C. Candidiasis
D. UTI

Correct Answer: C
Explanation:
Vaginal yeast infection
- Clinical presentation:
• Pruritis, white, crumbly, and thick (cottage cheese-like) discharge. Odorless
- Microscope:
• Pseudohyphae on KOH
- Treatment:
• Topical azoles or nystatin or Oral fluconazole

Female elderly complaining of white cheesy vaginal discharge with itchiness, what is the
diagnosis?

A. Diabetes mellitus
B. Hypertension

Correct Answer: A
Explanation:
Vaginal yeast infection
- Risk factors:
• Diabetes mellitus
• Antibiotic use
• Increased estrogen levels
• Immunosuppression
• Genetics
- Clinical presentation:
• Pruritis, white, crumbly, and thick (cottage cheese-like) discharge. Odorless
- Microscope:
• Pseudohyphae on KOH
- Treatment:
• Topical azoles or nystatin or Oral fluconazole

172 | P a g e
Obstetrics and Gynecology: SMLE Questions and Answers Chapter 2: Gynecology

Pregnant with high random blood sugar reading and fasting sugar. She’s at risk of what
vaginal Infection?

A. Candida
B. Bacterial vaginosis

Correct Answer: A
Explanation:
Vaginal yeast infection
- Risk factors:
• Diabetes mellitus
• Antibiotic use
• Increased estrogen levels
• Immunosuppression
• Genetics
- Clinical presentation:
• Pruritis, white, crumbly, and thick (cottage cheese-like) discharge. Odorless
- Microscope:
• Pseudohyphae on KOH
- Treatment:
• Topical azoles or nystatin or Oral fluconazole

A woman presents with vaginal discharge. Microscopic reveals spores. What is the most likely
diagnosis?

A. Bacterial vaginosis
B. Trichomoniasis
C. Candidiasis
D. Bacterial infection

Correct Answer: C
Explanation:
Vaginal yeast infection
- Clinical presentation:
• Pruritis, white, crumbly, and thick (cottage cheese-like) discharge. Odorless

173 | P a g e
Obstetrics and Gynecology: SMLE Questions and Answers Chapter 2: Gynecology
- Microscope:
• Pseudohyphae on KOH
- Treatment:
• Topical azoles or nystatin or Oral fluconazole

A 21-year-old woman patient complained of severe vulvovaginal itching and discharge. There
are no urinary or other symptoms and annual pap test have all been normal. On inspection,
vulvar erythema with swelling. Speculum examination revealed a thick, white, cruddy
discharge adherent to the vaginal walls with no odor. A gross wet mount revealed no evidence
of clue cells or trichomonads and the KOH prepped slide confirmed several pseudo-hyphea (see
lab results)

pH 4 (normal)

Whiff test (negative)

Which of the following is the most likely diagnosis?

A. Trichomoniasis
B. Bacterial vaginosis
C. Physiologic discharge
D. Vulvovaginal candidiasis

Correct Answer: D
Explanation:
Vaginal yeast infection
- Clinical presentation:
• Pruritis, white, crumbly, and thick (cottage cheese-like) discharge. Odorless
- Microscope:
• Pseudohyphae on KOH
- Treatment:
• Topical azoles or nystatin or Oral fluconazole

Patient present with cottage cheese like vaginal discharge, itching and urinary symptoms,
Asking about the treatment?

A. Topical estrogen
B. Topical antibiotic
C. Oral anti-fungal
D. Oral antibiotics

174 | P a g e
Obstetrics and Gynecology: SMLE Questions and Answers Chapter 2: Gynecology
Correct Answer: C
Explanation:
Vaginal yeast infection
- Clinical presentation:
• Pruritis, white, crumbly, and thick (cottage cheese-like) discharge. Odorless
- Microscope:
• Pseudohyphae on KOH
- Treatment:
• Topical azoles or nystatin or Oral fluconazole

Female with cottage cheese like vaginal discharge, What will you give the husband?

A. Ciprofloxacin
B. Metronidazole
C. No need
D. Antifungal

Correct Answer: C
Explanation:
Vaginal yeast infection
- Clinical presentation:
• Pruritis, white, crumbly, and thick (cottage cheese-like) discharge. Odorless
- Microscope:
• Pseudohyphae on KOH
- Treatment:
• Topical azoles or nystatin or Oral fluconazole

Female with vaginal discharge Grey, fishy odor, smear show clue cells, diagnosis?

A. Trichomoniasis
B. Bacterial vaginosis
C. Candida

Correct Answer: B
Explanation:
Bacterial vaginosis
- Clinical presentation:
• Gray/milky discharge. Fishy odor
- Microscope:
• Epithelial clue cells
- Treatment:
• Metronidazole

175 | P a g e
Obstetrics and Gynecology: SMLE Questions and Answers Chapter 2: Gynecology

Patient with vaginal discharge with fishy odor, smear show clue cells, what is the most
appropriate management?

A. Metronidazole patient alone


B. Cefotaxime
C. Fluconazole
D. Metronidazole patient & partner

Correct Answer: A
Explanation:
Bacterial vaginosis
- Clinical presentation:
• Gray/milky discharge. Fishy odor
- Microscope:
• Epithelial clue cells
- Treatment:
• Metronidazole

Female with fishy-smell vaginal discharge, Analysis PH: 5.1, what type of cells will you find?
A. Multinucleated giant cells
B. Overproduction of lactobacillus
C. Single nucleated cells
D. Granulated epithelial cells

Correct Answer: D
Explanation:
Bacterial vaginosis
- Clinical presentation:
• Gray/milky discharge. Fishy odor
- Microscope:
• Epithelial clue cells
- Treatment:
• Metronidazole

Female came to ER with lower abdominal pain and had adhesions in pelvis and hepatic area,
what is the organism?

A. Chlamydia
B. Bacterial vaginosis

176 | P a g e
Obstetrics and Gynecology: SMLE Questions and Answers Chapter 2: Gynecology
Correct Answer: A
Explanation: (According to Uptodate)
Perihepatitis — (Fitz-Hugh Curtis Syndrome) occurs in the setting of PID when there is
inflammation of the liver capsule and peritoneal surfaces of the anterior right upper quadrant.
- The syndrome Is associated with gonococcal salpingitis and Chlamydia trachomatis

Woman her husband has gonorrhea what’s the most diagnostic for her?

A. Anogenital swap
B. High vaginal swap
C. Endocervical swap
D. Urine culture

Correct Answer: C
Explanation: (According to Williams Obstetrics and Uptodate)
- Screening for gonorrhea in women is by culture or nucleic acid ampliication tests (NAATs).
- NAAT swab (endocervical or vulvovaginal)
- Collection from the vagina, endocervix, or urine. Of these, vaginal or cervical samples are
preferred.
- The preferred specimens are a first-catch urine for males and a vaginal swab for females.
An endocervical swab is appropriate if the patient was already undergoing speculum exam
for other reasons.

Post-menopausal women present with itchiness and odorless discharge with vaginal excoriation
what is the diagnosis?

A. Candidiasis
B. Triachomniasis
C. Bacterial Vaginosis
D. Atrophied vaginitis

Correct Answer: D
Explanation: (According to Uptodate)
Genitourinary syndrome of menopause (vulvovaginal atrophy):
- Symptoms of GSM include:
• Vulvovaginal dryness
• Decreased vaginal lubrication during sexual activity
• Dyspareunia, including vulvar or vaginal pain (at the introitus or within the vagina)
• Vulvar or vaginal bleeding (eg, postcoital bleeding, labial fissures)
• Decreased arousal, orgasm, or sexual desire
• Vulvovaginal burning, irritation, or itching
• Vaginal discharge (leukorrhea or yellow and malodorous)

177 | P a g e
Obstetrics and Gynecology: SMLE Questions and Answers Chapter 2: Gynecology
• Levator spasm
• Urinary tract symptoms (eg, urinary frequency, urinary urgency, dysuria, urethral
discomfort, hematuria, recurrent urinary tract infections)
• Urethral prolapse/caruncle
- Treatment include:
• First-line therapy: nonhormonal vaginal moisturizers and lubricants.
• Second-line therapy: Vaginal estrogen for those who does not adequately respond to
moisturizers and lubricants

Woman with dyspareunia and spotting fresh blood after intercourse. She had a history of warts
on vulva with cryotherapy done 2 years back, what is the site of the bleeding?

A. Vulva
B. Vagina
C. Cervix
D. Body of uterus

Correct Answer: C

Newly married woman has been on her honeymoon for 1 week and is now complaining of
dysuria and frequency. How will you manage?

A. Ampicillin
B. Nitrofurantoin
C. Ciprofloxacin
D. Ceftriaxone

Correct Answer: B
Explanation: (According to Uptodate)
The preferred agents for empiric therapy of acute simple cystitis are nitrofurantoin

178 | P a g e
Obstetrics and Gynecology: SMLE Questions and Answers Chapter 2: Gynecology

Pelvic Inflammatory Disease (PID)

Intrauterine device implantation patient has brown discoloration and abdominal pain.
What’s the most likely diagnosis?

A. Uterine rupture
B. Pelvic inflammatory disease
C. Bacterial vaginosis

Correct Answer: B
Explanation: (According to Williams obstetrics and Uptodate)
Pelvic inflammatory disease
- Risk Factors:
• Multiple sexual partners, unprotected sex
• History of prior STI and/or adnexitis
• Intrauterine device
• Vaginal dysbiosis

Which bacterial infection is associated with intrauterine device in PID?

A. Chlamydia
B. Actinomyces isrelii
C. Gonorrhea

Correct Answer: B
Explanation: (According to Williams Gynecology)
Some have found Actinomyces species more frequently in the vaginal flora of IUD users, and rates
of colonization rise with duration of lUD use

A 25-year-old women had severe pelvic inflammatory disease 2 years ago that was treated by
intravenous antibiotics. The isolated organism was chlamydia trachomatis. Since then, the
patient was trying for pregnancy but failed to do so. She had normal regular period and
normal analysis of her husband. Which of the following investigations is recommended?

A. Ultrasound monitoring of her ovulation


B. Magnetic resonance image of the pelvis
C. Computed tomography of the pelvis
D. Hysterosalpingogram

Correct Answer: D
Explanation: (According to Williams Gynecology and Uptodate)

179 | P a g e
Obstetrics and Gynecology: SMLE Questions and Answers Chapter 2: Gynecology
Complications of PID: infertility, due to permanent injury to the fallopian tube. Among the
microorganisms that cause PID, Chlamydia trachomatis appears to carry the greatest risk of
infertility

A young female patient presents with abdominal pain, spasm of the lower abdominal muscles
and copious amounts of vaginal discharge. There is positive cervical motion tenderness. Also,
she is febrile. What is the most likely diagnosis?

A. Acute cervicitis
B. Acute salpingitis
C. Appendicitis
D. Cholecystitis

Correct Answer: B
Explanation: (According to Williams Gynecology and Uptodate)
Pelvic inflammatory disease

- Symptoms and physical findings


• Lower abdominal pain (generally bilateral) or pelvic pain
• Uterine tenderness, adnexal tenderness, or cervical motion tenderness
• Nausea, vomiting, diarrhea
• Fever
• Dysuria, urinary urgency
• Menorrhagia, metrorrhagia
• Dyspareunia, dysmenorrhea
• Abnormal vaginal discharge (yellow/green color)
• One or more of the following enhances diagnostic specificity:
(1) Oral temperature >38.3°C (lOI.6°F)
(2) Mucopurulent cervical discharge or cervical friability
(3) Abundant WBCs on saline microscopy of cervical secretions
(4) Elevated erythrocyte sedimentation rate (ESR) or C-reactive protein (CRP)
(5) Presence of cervical N gonorrhea or C trachomatis
- Differential diagnosis:
• Acute cervicitis
Cervicitis alone generally does not cause pelvic or abdominal pain. However,
patients with associated endometritis or pelvic inflammatory disease (PID) often
complain of pain.

A young woman presents with fever and diffuse abdominal pain. On exam, the cervix and
vaginal wall were normal with positive cervical motion tenderness. US shows no adnexal
masses. Beta-HCG was negative. What is the most likely diagnosis?

A. PID

180 | P a g e
Obstetrics and Gynecology: SMLE Questions and Answers Chapter 2: Gynecology
B. Ectopic pregnancy
C. Acute cervicitis
D. Ovarian torsion

Correct Answer: A
Explanation: (According to Williams Gynecology and Uptodate)
Pelvic inflammatory disease

- Symptoms and physical findings


• Lower abdominal pain (generally bilateral) or pelvic pain
• Uterine tenderness, adnexal tenderness, or cervical motion tenderness
• Nausea, vomiting, diarrhea
• Fever
• Dysuria, urinary urgency
• Menorrhagia, metrorrhagia
• Dyspareunia, dysmenorrhea
• Abnormal vaginal discharge (yellow/green color)
• One or more of the following enhances diagnostic specificity:
(1) Oral temperature >38.3°C (lOI.6°F)
(2) Mucopurulent cervical discharge or cervical friability
(3) Abundant WBCs on saline microscopy of cervical secretions
(4) Elevated erythrocyte sedimentation rate (ESR) or C-reactive protein (CRP)
(5) Presence of cervical N gonorrhea or C trachomatis
- Differential diagnosis:
• Acute cervicitis
Cervicitis alone generally does not cause pelvic or abdominal pain. However,
patients with associated endometritis or pelvic inflammatory disease (PID) often
complain of pain.

Female with suprapubic tenderness and non-purulent vaginal discharge. What is the type of
infection?

A. Vulvar
B. Vaginal
C. Cervicitis
D. Uterocervicitis

Correct Answer: D
Explanation: (According to Williams Gynecology ad Uptodate)
Pelvic inflammatory disease

- Symptoms and physical findings


• Lower abdominal pain (generally bilateral) or pelvic pain

181 | P a g e
Obstetrics and Gynecology: SMLE Questions and Answers Chapter 2: Gynecology
• Uterine tenderness, adnexal tenderness, or cervical motion tenderness
• Nausea, vomiting, diarrhea
• Fever
• Dysuria, urinary urgency
• Menorrhagia, metrorrhagia
• Dyspareunia, dysmenorrhea
• Abnormal vaginal discharge (yellow/green color)
• One or more of the following enhances diagnostic specificity:
(1) Oral temperature >38.3°C (lOI.6°F)
(2) Mucopurulent cervical discharge or cervical friability
(3) Abundant WBCs on saline microscopy of cervical secretions
(4) Elevated erythrocyte sedimentation rate (ESR) or C-reactive protein (CRP)
(5) Presence of cervical N gonorrhea or C trachomatis
- Differential diagnosis:
• Acute cervicitis
Cervicitis alone generally does not cause pelvic or abdominal pain. However,
patients with associated endometritis or pelvic inflammatory disease (PID) often
complain of pain.

A patient with acute salpingitis, she was treated with IV ceftriaxone but did not improve. What
is the most likely causative organism?

A. HSV
B. Neisseria gonorrhoeae
C. Chlamydia trachomatis
D. Adenovirus

Correct Answer: C
Explanation: (According to Williams Gynecology and Uptodate)
Pelvic inflammatory disease
- Treatment
• Outpatient regimen
o One single dose of IM ceftriaxone 250mg IM once PLUS oral therapy
with doxycycline (alternative azithromycin)100mg orally twice daily for 14
days
o With or without Metronidazole 500mg orally twice daily for 14 days (for
coverage of anaerobic organisms)
o Note: Ceftriaxone it is the recommended for coverage of gonorrhea.
Doxycycline or azithromycin is the preferred agent to treat C.
trachomatis infection

182 | P a g e
Obstetrics and Gynecology: SMLE Questions and Answers Chapter 2: Gynecology

PID treated by ceftriaxone, what is the most likely organism?


A. Adenovirus
B. HSV
C. Neisseria Gonorrhea
D. Chlamydia trachomatis

Correct Answer: C
Explanation: (According to Williams Gynecology and Uptodate)
Pelvic inflammatory disease
- Treatment
• Outpatient regimen
o One single dose of IM ceftriaxone 250mg IM once PLUS oral therapy
with doxycycline (alternative azithromycin)100mg orally twice daily for 14
days
o With or without Metronidazole 500mg orally twice daily for 14 days (for
coverage of anaerobic organisms)
o Note: Ceftriaxone it is the recommended for coverage of gonorrhea.
Doxycycline or azithromycin is the preferred agent to treat C.
trachomatis infection

Neisseria Gonorrhea à Ceftriaxone


Chlamydia trachomatis à Doxycycline or azithromycin

PID treated by doxycycline, what is the most likely organism?


A. Adenovirus
B. HSV
C. Neisseria Gonorrhea
D. Chlamydia trachomatis

Correct Answer: D
Explanation: (According to Williams gynecology and Uptodate)
Pelvic inflammatory disease
- Treatment
• Outpatient regimen
o One single dose of IM ceftriaxone 250mg IM once PLUS oral therapy
with doxycycline (alternative azithromycin)100mg orally twice daily for 14
days
o With or without Metronidazole 500mg orally twice daily for 14 days (for
coverage of anaerobic organisms)
o Note: Ceftriaxone it is the recommended for coverage of gonorrhea.
Doxycycline or azithromycin is the preferred agent to treat C.
trachomatis infection

183 | P a g e
Obstetrics and Gynecology: SMLE Questions and Answers Chapter 2: Gynecology
Neisseria Gonorrhea à Ceftriaxone
Chlamydia trachomatis à Doxycycline or azithromycin

A 18-year-old female complains of vaginal pruritus & discharge 5 days ago, no fevers or
abdominal pain. On physical examination: normal genitalia. speculum exam: thick yellow
cervical discharge with easy bleed on touch.

Cervical swabs done & reveal no organism on gram stain. What is the most appropriate
treatment?

A. Azithromycin and ceftriaxone


B. Ceftriaxone only
C. Azithromycin only
D. Metronidazole only

Correct Answer: D
Explanation: (According to Uptodate)
Acute cervicitis
- Etiology
• Infectious:
o Most common: Chlamydia trachomatis and Neisseria gonorrhoeae
o Other infectious etiologies include herpes simplex virus (HSV), Mycoplasma
genitalium, Trichomonas vaginalis
• Non-infectious:
o Caused by mechanical or chemical irritation.
- Clinical presentation:
• Characteristic symptoms include either or both:
o Purulent or mucopurulent (yellow) endocervical exudate that is visible either
in the endocervical canal or on an endocervical swab
o Endocervical bleeding that is easily caused by gently passing an
endocervical swab through the cervical os
• Additional symptoms or signs can include:
o Dysuria, urinary frequency
o Dyspareunia
o Vulvovaginal irritation
o Edema of the cervical ectropion
o Postcoital bleeding

- Diagnosis:
• The diagnosis of acute cervicitis is clinical and, based upon the presence of purulent
or mucopurulent cervical exudate or sustained endocervical bleeding (friability),
easily induced by gently touching the area with a swab
184 | P a g e
Obstetrics and Gynecology: SMLE Questions and Answers Chapter 2: Gynecology
- Treatment
• Empiric therapy — We suggest that patients with cervicitis receive empiric
antibiotic therapy at the time of initial evaluation, without waiting for results of
laboratory tests. Empiric therapy that covers both gonorrhea and chlamydia
infections because these are the most common infectious etiologies of cervicitis

• Targeted treatment of specific infections — In this approach, treatment decisions


are deferred until the laboratory and microscopy results are available.
o Chlamydia – Doxycycline or azithromycin
o Gonorrhea – Ceftriaxone
o Trichomoniasis – Metronidazole
o Herpes simplex virus – Acyclovir

Note: for this question, gram stain might exclude chlamydia and gonorrhea, so its trichomoniasis?

Female with bilateral tubo-ovarian abscess came with very severe abdominal pain, what is
appropriate management?

A. IV Antibiotic
B. CT guided draining
C. Laparotomy
D. Oral antibiotics

Correct Answer: A
Explanation: (According to Williams Gynecology)
In the case of Tubo-ovarian abscess:
- Parenteral antimicrobial therapy is continued until the patient has been afebrile for at least
24 hours, preferably 48 to 72 hours

185 | P a g e
Obstetrics and Gynecology: SMLE Questions and Answers Chapter 2: Gynecology

Contraception and Hormonal Replacement Therapy (HRT)

Patient with hypertension, with heavy bleeding and anemia, what contraceptive she can
use?

A. Tubal ligation
B. IUCD Mirena
C. Condom
D. OCP

Correct Answer: B
Explanation: (According to ACOG)
Intrauterine device IUD
- Copper IUD:
• FDA approved for up to10 continuous years
• The most common adverse effects reported are heavy menstrual bleeding and pain
- Levonorgestrel-Releasing Intrauterine Devices:
• FDA approved for up to 5 years of use
• The most common adverse effects reported are diminished menstrual bleeding and
amenorrhea (Used to treat heavy bleedings)
• Approved by health Canada in the treatment of idiopathic menorrhagia.

Combined oral contraceptives OCP


⁃ Women with blood pressure (BP) below 140/90 mm Hg à may use any hormonal
contraceptive method.
⁃ Women with hypertension of systolic 140–159 mm Hg or diastolic 90–99 mm Hg àOCP
should not be used unless no other method is appropriate for or acceptable to the patient
(USMEC category 3).
⁃ Women with hypertension of systolic 160 mm Hg or greater or diastolic 100 mg Hg or
greater or with vascular disease à should not use combined hormonal contraceptives
(USMEC category 4)

Regarding injectable progesterone, which one of the following is correct?

A. Can cause skin problems


B. Associated with irregular bleeding and weight gain.
C. Decrease in bone mineral density (reversible).
D. Delayed fertility after discontinuation (one shot can last 10 months

Correct Answer: C
Explanation: (According to ACOG)
Progestin-Only Hormone Injection “Medroxyprogesterone acetate (DMPA)”:
Side effects:

186 | P a g e
Obstetrics and Gynecology: SMLE Questions and Answers Chapter 2: Gynecology
⁃Irregular bleeding (usually decreases with each injection.)
⁃Bone Loss (When the injections are stopped, at least some and sometimes all of the bone
that is lost is gained back.)
***DOES NOT CAUSE WEIGHT GAIN

Nullipara with menorrhagia and dysmenorrhea, what is the type of contraceptive


appropriate for this patient?

A. POP (progestogen-only pill)


B. OCP
C. Depo Provera Injection
D. NSAID

Correct Answer: B
Explanation: (According to UpToDate)
Estrogen-progestin oral contraceptives (OCs) are first-line management for AUB. Also, its used for
dysmenorrhea

Patient with cardiovascular disease, she wants contraceptive, what type of contraceptive
appropriate for this patient?

A. IUD
B. OCP
C. POP (progestogen-only pill)
D. Condom

Correct Answer: A
Explanation: (According to Comprehensive Gynecology Text book)
The American Congress of Obstetricians and Gynecologists (ACOG) recommends that LARC
methods be offered as first-line contraception to most women (ACOG, 2009).

Patient underwent total hysterectomy with bilateral oophorectomy what to give her?

A. Estrogen patch
B. lUD
C. OCP (Progesterone and estrogen)
D. Progesterone only pills

Correct Answer: A
Explanation: (According to UpToDate)
Surgical menopause — In women who have undergone a hysterectomy and who are candidates
for MHT, unopposed estrogen is given. Progestins are only given to women with an intact uterus
to prevent endometrial hyperplasia and cancer

187 | P a g e
Obstetrics and Gynecology: SMLE Questions and Answers Chapter 2: Gynecology

A 30-year-old woman asking for effective and reversible method of contraceptive, which if
the following is the most effective?

A. Vaginal ring containing hormones


B. Female condom
C. Male condom
D. Spermicide

Correct Answer: A
Explanation:

A healthy young lady just delivered a baby 4 months ago. However, she tells you that she
does not want to get pregnant for the next 2 years. What method of contraception will you
recommend?

A. Depo provera injections


B. POP
C. OCP

Correct Answer: A

Another recall

A healthy young lady just delivered a baby 4 months ago. However, she tells you that she
does not want to get pregnant for the next 2 years. What method of contraception will you
recommend?

A. Combined OCP
B. Depo Provera injection
C. Vaginal ring
D. Transdermal patch

188 | P a g e
Obstetrics and Gynecology: SMLE Questions and Answers Chapter 2: Gynecology
Correct Answer: B
Explanation:

According to The US MEC and SPR


• Injections are more effective than the pills
According to ACOG:
• Concerns regarding the effect of DMPA on BMD and potential fracture risk should
not prevent practitioners from prescribing DMPA or continuing use beyond 2 years

A 28-year-old female known to have an ovarian cyst, came to your clinic asking about
contraception, which of the following methods is most appropriate for her?

A. OCP
B. Oral progestins only pills
C. Vaginal ring
D. Male condom

Correct Answer: A
Explanation:
- OCP’s are not contraindicated in Ovarian cyst. (Refer to the to US Medical Eligibility
Criteria US MEC for Contraceptive Use, 2016)

- According to UTD
Progestin-only pills (POPs) are one option for women who cannot or prefer not to use
estrogen-containing contraception

Patient has Ovarian cyst want contraception method, what you will give her?

A. OCP
B. Progesterone only pills
C. IUD

Correct Answer: C

189 | P a g e
Obstetrics and Gynecology: SMLE Questions and Answers Chapter 2: Gynecology
Explanation: (According to Comprehensive Gynecology Text book)
The American Congress of Obstetricians and Gynecologists (ACOG) recommends that LARC
methods be offered as first-line contraception to most women (ACOG, 2009).

What is the mechanism of action MOA of an emergency contraceptives?

A. Prevent implantation
B. Prevent fertilization
C. Prevent ovulation
D. Delays fertilisation

Correct Answer: C
Explanation: (According to UpToDate)
Emergency contraceptive pills include levonorgestrel, ulipristal acetate and combined oral
contraceptive pills.
- Oral medication EC methods – Rigorous, meticulously executed studies have
demonstrated that oral ulipristal acetate (UPA) and levonorgestrel (LNG) EC regimens
prevent ovulation
- IUDs – Contrary to the clearly defined mechanisms of action for both oral EC methods,
there is no clear description that fully explains the efficacy of the IUDs for EC. While data
clearly support inhibition of fertilization as the primary mode of action for copper
and levonorgestrel IUDs when they are used continuously, the mechanism for postcoital use
is not clear

A patient with pulmonary embolism that wants contraception. WHAT is the most
appropriate contraceptive method?

A. Condom
B. IUD
C. OCP
D. POP

Correct Answer: B
Explanation: (According to Comprehensive Gynecology Text book)
The American Congress of Obstetricians and Gynecologists (ACOG) recommends that LARC
methods be offered as first-line contraception to most women (ACOG, 2009).

Female patient with Cardiomyopathy and using heparin (or anticoagulant). What
contraception is most appropriate for her?

A. POP
B. Depoprovira
C. IUCD

190 | P a g e
Obstetrics and Gynecology: SMLE Questions and Answers Chapter 2: Gynecology
D. OCP

Correct Answer: C
Explanation: (According to Comprehensive Gynecology Text book)
The American Congress of Obstetricians and Gynecologists (ACOG) recommends that LARC
methods be offered as first-line contraception to most women (ACOG, 2009).

Patient diagnosed with Endometriosis. She is complaining of dysmenorrhea. Which of the


following is the best first line medical treatment?

A. Depo provera injection


B. Progestrone only pills
C. Combined oral contraceptive

Correct Answer: C
Explanation: (According to Williams Gynecology)
Combined Hormonal Contraceptives:
Estrogen plus progestin-containing agents are a mainstay for the initial treatment of endometriosis-
related pain

A women delivered and breastfeeding, what is the most appropriate contraceptive method?

A. POP
B. IUD
C. OCP

Correct Answer: B
Explanation: (According to Comprehensive Gynecology Text book)
The American Congress of Obstetricians and Gynecologists (ACOG) recommends that LARC
methods be offered as first-line contraception to most women (ACOG, 2009).

Patient Para 4, history of c-section and then she developed endometritis which has been
treated, she has a regular cycle but increased bleeding recently. This patient was using IUD
before but she got pregnant, she came now for contraception. Which one of the following is
a contraindication for IUD?

A. Previous C-section
B. Genital bleeding
C. History of Endometritis
D. History of contraception failure

Correct Answer: B
Explanation: (According to Williams Gynecology)

191 | P a g e
Obstetrics and Gynecology: SMLE Questions and Answers Chapter 2: Gynecology
Contraindications to IUD use:

- Copper-containing
• Pregnancy or suspicion of pregnancy
• Uterine abnormality with distorted uterine cavity
• Acute PID, or current behavior suggesting a high risk for PID
• Postpartum endometritis or postabortal endometritis in the past 3 months
• Known or suspected uterine or cervical malignancy
• Genital bleeding of unknown etiology
• Mucopurulent cervicitis
• Wilson disease
• Allergy to any component of ParaGard
• A previously placed IUD that has not been removed

- Levonorgestrel-releasing
• Pregnancy or suspicion of pregnancy
• Uterine abnormality with distorted uterine cavity
• Acute PID or history of unless there has been a subsequent intrauterine pregnancy
• Postpartum endometritis or infected abortion in the past 3 months
• Known or suspected uterine or cervical neoplasia
• Uterine bleeding of unknown etiology
• Untreated acute cervicitis or vaginitis or other lower genital tract infections
• Acute liver disease or liver tumor (benign or malignant)
• Increased susceptibility to pelvic infection
• A previously placed IUD that has not been removed
• Hypersensitivity to any component of Mirena
• Known or suspected breast cancer or other progestin-sensitive cancer

A 23 year old lady was prescribed with azithromycin 1 gm for her chlamydial pelvic
infection. She has got a new boyfriend for the last 2 months. She has recently started
contraception to avoid conception. Which of the following contraceptive method will be
affected by azithromycin?

A. Barrier
B. IUCD
C. POP
D. COCP

Correct Answer: D
Explanation: (According to American Academy of Family Physicians)
Azithromycin is more likely to reduce OCP effectiveness

192 | P a g e
Obstetrics and Gynecology: SMLE Questions and Answers Chapter 2: Gynecology

Abortion, Pregnancy Loss, and Intrauterine Fetal Demise (IUFD)

Pregnant in first trimester develop vaginal bleeding and lower quadrant pain, she denied
any passage of tissue. U/S shows: No sac either intrauterine or extrauterine, what is the
most likely diagnosis ?

A. Ovarian ectopic
B. Complete abortion
C. Missed abortion
D. Pregnancy of unknown location

Correct Answer: D
Explanation:
Let’s Exclude
- Ovarian ectopic à adnexal mass on US
- Complete abortion à Passage or expulsion of tissue
- Missed abortion à a nonviable pregnancy in absence of symptoms
- Pregnancy of unknown location à if the gestational sac cannot be seen at all on US (When
a pregnant patient with pain and/or bleeding has an US that has no findings of pregnancy)

Pregnant at 7 weeks with vaginal bleeding, no intrauterine or extrauterine sac on US, what
is the most appropriate management?

A. US after 2 days
B. Reassurance
C. Discharge home

Correct Answer: A
Explanation: (According to UpToDate)
Pregnancy of Unknown Location
- Absence of findings on US:
o When a pregnant patient with pain and/or bleeding has an US that has no findings of
pregnancy (ie, no intrauterine pregnancy [IUP], adnexal mass, or echogenic fluid in
the pelvis)
o The differential diagnosis is normal early IUP, nonviable IUP, or ectopic pregnancy.
Approximately 15 to 26 percent of patients with ectopic pregnancy will have a
"normal" initial US

- In a hemodynamically stable patient with a desired pregnancy:


o An US with no findings should be repeated when the human chorionic gonadotropin
(hCG) reaches the discriminatory zone for endometrial findings

193 | P a g e
Obstetrics and Gynecology: SMLE Questions and Answers Chapter 2: Gynecology
o Or US with no findings should be repeated in three to four days since the gestational
sac of an IUP grows approximately 1 mm per day and is visible on US when it reaches
3 mm or greater.

A 25-Year-old lady pregnant at 7 weeks gestational age, presenting with vaginal spotting, On
Transvaginal US there is no intrauterine nor extrauterine sac, what to do next?

A. Laparoscopy
B. Repeat US and labs
C. Methotrexate

Correct Answer: B
Explanation: (According to UpToDate)
Pregnancy of Unknown Location
- Absence of findings on US:
o When a pregnant patient with pain and/or bleeding has an US that has no findings of
pregnancy (ie, no intrauterine pregnancy [IUP], adnexal mass, or echogenic fluid in
the pelvis)
o The differential diagnosis is normal early IUP, nonviable IUP, or ectopic
pregnancy. Approximately 15 to 26 percent of patients with ectopic pregnancy will
have a "normal" initial US

- In a hemodynamically stable patient with a desired pregnancy:


o An US with no findings should be repeated when the human chorionic gonadotropin
(hCG) reaches the discriminatory zone for endometrial findings
o Or US with no findings should be repeated in three to four days since the gestational
sac of an IUP grows approximately 1 mm per day and is visible on US when it
reaches 3 mm or greater.

A 45-Year-old lady with miscarriage, she asked if her age had anything to do with her
miscarriage?

A. Risk of miscarriage is 3% at this age.


B. From 10 to 50%
C. 80%
D. No risk

Correct Answer: C
Explanation: (According to ACOG)
The frequency of clinically recognized early pregnancy loss for women aged 20–30 years is 9–
17%, and this rate increases sharply from 20% at age 35 years to 40% at age 40 years and 80% at
age 45 years.

194 | P a g e
Obstetrics and Gynecology: SMLE Questions and Answers Chapter 2: Gynecology
Pregnant women with previous 1 spontaneous abortion asking about the percentage of
abortion happening in next pregnancy?

A. 5%
B. 10%
C. 25%
D. 50%

Correct Answer: B
Explanation: (According to comprehensive Gynecology)

A 22-Years-Old female, primigravida with spontaneous abortion. Asking you to address


her concern about her next pregnancy, what to do?

A. Tell her spontaneous abortions can occur 10-15% of the time


B. Tell her never to get pregnant 1
C. Do a full workup for reccurent pregnancy loss

Correct Answer: A
Explanation: (According to ACOG)
Early pregnancy loss is common, occurring in 10% of all clinically recognized pregnancies

A 25-year-old lady lost her pregnancy at 7 weeks gestational age , she is heavy smoker 20 s
per day , she did not take any supplements nor folic or iron for her last pregnancy, What is
the couse of her abortion?

A. Family history
B. Smoking a 20 s per day
C. No risk factors.
D. Folate acid deficiency
E. Iron deficiency

195 | P a g e
Obstetrics and Gynecology: SMLE Questions and Answers Chapter 2: Gynecology

Correct Answer: B
Explanation: (According to Uptodate)
Pregnancy loss (miscarriage) — Cigarette smoking during pregnancy may have a modest effect
on pregnancy loss

Pregnant woman at 15 weeks of gestation, came with severe bleeding and component seen
in cervix. What is the most appropriate next management?

A. IV fluid and misoprostol


B. IV fluid and expectant management
C. IV fluid and D&C
D. IV fluid and uterine message

Correct Answer: C
Explanation: (According to ACOG and Uptodate)
Inventible, incomplete, and missed abortions management
- Expectant management (if ≤13 weeks GA)
o Up tp 8 weeks, if the pregnancy has not passed in a reasonable time or if bleeding,
infection, or other complication develop à Managed by medical or surgical.
- Medical (up to 20 weeks GA)
o By two different medications mifepristone followed by misoprostol
o Considered in women without infection, hemorrhage, severe anemia, or bleeding
disorders.

- D&C (up to 20 weeks GA)


o Considered in women who present with hemorrhage, hemodynamic instability, or
signs of infection should be treated urgently with surgical uterine evacuation.

A 19-year-old pregnant and did a home pregnancy test 8 weeks back. She came to the clinic
to do an examination and they found out that the fetus was dead at 5 weeks of gestation,
and she had black blood clot, what is the management?

A. Observation
B. Suction
C. Misoprostol
D. Expectant management

Correct Answer: D
Explanation: (According to ACOG and UpTodate)
Inventible, incomplete, and missed abortions management
- Expectant management (if ≤13 weeks GA)
o Up tp 8 weeks, if the pregnancy has not passed in a reasonable time or if bleeding,
infection, or other complication develop à Managed by medical or surgical.

196 | P a g e
Obstetrics and Gynecology: SMLE Questions and Answers Chapter 2: Gynecology
- Medical (up to 20 weeks GA)
o By two different medications mifepristone followed by misoprostol
o Considered in women without infection, hemorrhage, severe anemia, or bleeding
disorders.

- D&C (up to 20 weeks GA)


o Considered in women who present with hemorrhage, hemodynamic instability, or
signs of infection should be treated urgently with surgical uterine evacuation.

Woman at 7 weeks of gestation, came with PV spotting female denies passage of part of
product after doctor examination show tissue product in cervix, what is management?

A. Expectant management
B. D&C
C. Progesterone
D. For surgery

Correct Answer: A
Explanation: (According to ACOG and UpTodate)
Inventible, incomplete, and missed abortions management
- Expectant management (if ≤13 weeks GA)
o Up tp 8 weeks, if the pregnancy has not passed in a reasonable time or if bleeding,
infection, or other complication develop à Managed by medical or surgical.
- Medical (up to 20 weeks GA)
o By two different medications mifepristone followed by misoprostol
o Considered in women without infection, hemorrhage, severe anemia, or bleeding
disorders.

- D&C (up to 20 weeks GA)


o Considered in women who present with hemorrhage, hemodynamic instability, or
signs of infection should be treated urgently with surgical uterine evacuation.

Pregnant at 8 weeks with inevitable abortion, they let her decide what she want, after 15
min severe gush of blood coming out, what to do?

A. Observe
B. Mifepristone
C. D&C
D. Expectant management

Correct Answer: C
Explanation: (According to ACOG and UpTodate)
Inventible, incomplete, and missed abortions management
- Expectant management (if ≤13 weeks GA)

197 | P a g e
Obstetrics and Gynecology: SMLE Questions and Answers Chapter 2: Gynecology
o Up tp 8 weeks, if the pregnancy has not passed in a reasonable time or if bleeding,
infection, or other complication develop à Managed by medical or surgical.
- Medical (up to 20 weeks GA)
o By two different medications mifepristone followed by misoprostol
o Considered in women without infection, hemorrhage, severe anemia, or bleeding
disorders.

- D&C (up to 20 weeks GA)


o Considered in women who present with hemorrhage, hemodynamic instability, or
signs of infection should be treated urgently with surgical uterine evacuation.

Woman at 14 weeks of gestation came with abortion female denies passage of product,
upon examination tissue product felt in cervix, what is management?

A. Expectant management
B. D&C
C. Progesterone
D. For surgery

Correct Answer: B
Explanation: (According to ACOG and UpTodate)
Inventible, incomplete, and missed abortions management
- Expectant management (if ≤13 weeks GA)
o Up tp 8 weeks, if the pregnancy has not passed in a reasonable time or if bleeding,
infection, or other complication develop à Managed by medical or surgical.
- Medical (up to 20 weeks GA)
o By two different medications mifepristone followed by misoprostol
o Considered in women without infection, hemorrhage, severe anemia, or bleeding
disorders.

- D&C (up to 20 weeks GA)


o Considered in women who present with hemorrhage, hemodynamic instability, or
signs of infection should be treated urgently with surgical uterine evacuation.
Note:
- If she is 13 weeks or less à expectant management
- If she is 14-20 weeks à medical or D&C

Patient presenting with severe bleeding at 9th week of pregnancy, upon examination OS
was open and saw some tissue on the cervix. What is your management?

A. Expectant management
B. D&C
C. Oxytocin
D. For Surgery

198 | P a g e
Obstetrics and Gynecology: SMLE Questions and Answers Chapter 2: Gynecology

Correct Answer: B
Explanation: (According to ACOG and UpToDate)
Inventible, incomplete, and missed abortions management
- Expectant management (if ≤13 weeks GA)
o Up tp 8 weeks, if the pregnancy has not passed in a reasonable time or if bleeding,
infection, or other complication develop à Managed by medical or surgical.
- Medical (up to 20 weeks GA)
o By two different medications mifepristone followed by misoprostol
o Considered in women without infection, hemorrhage, severe anemia, or bleeding
disorders.

- D&C (up to 20 weeks GA)


o Considered in women who present with hemorrhage, hemodynamic instability, or
signs of infection should be treated urgently with surgical uterine evacuation.

Female with missed abortion, what to do?

A. Misoprostol
B. Hypertonic saline
C. Myotomy
D. Oxytocin

Correct Answer: A
Explanation: (According to ACOG and UpToDate)
Inventible, incomplete, and missed abortions management
- Expectant management (if ≤13 weeks GA)
o Up tp 8 weeks, if the pregnancy has not passed in a reasonable time or if bleeding,
infection, or other complication develop à Managed by medical or surgical.
- Medical (up to 20 weeks GA)
o By two different medications mifepristone followed by misoprostol
o Considered in women without infection, hemorrhage, severe anemia, or bleeding
disorders.

- D&C (up to 20 weeks GA)


o Considered in women who present with hemorrhage, hemodynamic instability, or
signs of infection should be treated urgently with surgical uterine evacuation.

Pregnant at 7 weeks of gestationcame to ER complaining of passing tissue at home and on


examination she has moderate bleeding, what is the appropriate management?

A. Expectant management
B. D&C

199 | P a g e
Obstetrics and Gynecology: SMLE Questions and Answers Chapter 2: Gynecology
Correct Answer: B
Explanation: (According to ACOG and UpToDate)
Inventible, incomplete, and missed abortions management
- Expectant management (if ≤13 weeks GA)
o Up tp 8 weeks, if the pregnancy has not passed in a reasonable time or if bleeding,
infection, or other complication develop à Managed by medical or surgical.
- Medical (up to 20 weeks GA)
o By two different medications mifepristone followed by misoprostol
o Considered in women without infection, hemorrhage, severe anemia, or bleeding
disorders.

- D&C (up to 20 weeks GA)


o Considered in women who present with hemorrhage, hemodynamic instability, or
signs of infection should be treated urgently with surgical uterine evacuation.

Pregnant at 10 weeks presented with history of cramps and vaginal bleeding at home with
passage of tissue, upon examination the os is closed, US showed empty uterus, what’s the
most appropriate management?

A. Reassure
B. Intravenous fluid
C. D&C
D. Prostaglandin

Correct Answer: A
Explanation:
Complete abortion management
- No treatment is required and follow up in OPD

Pregnant at 8 weeks of gestation with vaginal spotting and everything else is normal, upon
examination cervix is closed. On US normal gestational sac seen, what is the most
appropriate management?

A. Reassurance
B. Evacuate
C. Misoprostol

Correct Answer: A
Explanation:
Threatened abortion management
- Expectant management and resume physical activity

200 | P a g e
Obstetrics and Gynecology: SMLE Questions and Answers Chapter 2: Gynecology

23 years old female pregnant in her 28th week, pregnancy test was done at home. She
presented with mild bleeding loss of fetal movement. She denied any passage of tissue or
abdominal pain. Transvaginal US was done and showed pregnancy of 18 weeks and no
heart beat. (No other information was provided about the cervix). What is your diagnosis?

A. Fetal Demise
B. Missed Abortion
C. Incomplete abortion

Correct Answer: B
Explanation:

Patient with urine pregnancy test positive at home, now presents with lower abdominal
pain. Pain worsens on cervical touching. What you will do initially?

A. Transvaginal assessment
B. Deep palpitation of pelvic to rule out masses

Correct Answer: A
Explanation: (According to ACOG)
Treatment of an early pregnancy loss before confirmed diagnosis can have detrimental
consequences, including interruption of a normal pregnancy, pregnancy complications, or birth
defects. Therefore, a thorough evaluation is needed to make a definitive diagnosis. In combination
with a thorough medical history and physical examination, ultrasonography and serum β-hCG
testing can be helpful in making a highly certain diagnosis.

Patient at 8 weeks gestation, presenting to the ED with vaginal bleeding and abdominal
pain, her cervical OS is open and tissue can be see within the cervical os. What is your
diagnosis?

A. Threatened abortion
B. Incomplete abortion
C. Inevitable abortion
D. Complete abortion

Correct Answer: B
Explanation:
Inevitable abortion
- Clinical presentation:

201 | P a g e
Obstetrics and Gynecology: SMLE Questions and Answers Chapter 2: Gynecology
o Cervix dilated
o Bleeding
o No passage of products of conception. POC are visible within the cervical canal or in
uterus
- Ultrasound findings:
o Products of conception presen
o Fetal activity may be present

Primigravida come to clinic with no symptoms, upon examination cervical Os open, US showed
viable fetus, what the type of abortion?

A. Thretened abortion
B. Inevitable abortion
C. Incomplete abortion
D. Anembryonic pregnancy

Correct Answer: B
Explanation:
Inevitable abortion
- Clinical presentation:
o Cervix dilated
o Bleeding
o No passage of products of conception. POC are visible within the cervical canal or
in uterus
- Ultrasound findings:
o Products of conception presen
o Fetal activity may be present

Woman with history of miscarriage, now she is at 7 weeks of gestation with vaginal
spotting, no history of passing tissue , on examination os is closed and no bleeding, what is
the type of abortion?

A. Threatened abortion.
B. Normal pregnancy
C. Incomplete abortion
D. Inevitable Abortion

Correct Answer: A
Explanation:
Threatened abortion
- Clinical presentation:
o Cervix closed
o Bleeding
o No passage of products of conception
202 | P a g e
Obstetrics and Gynecology: SMLE Questions and Answers Chapter 2: Gynecology
- Ultrasound findings:
o Products of conception present
o Fetal activity present

Pregnant at 8 weeks, presenting with vaginal spotting and everything else is normal, upon
examination cervix is closed. On US normal gestational sac seen, what is the type of abortion?

A. Threatened abortion
B. Normal pregnancy
C. Incomplete abortion
D. Inventible abortion

Correct Answer: A
Explanation:
Threatened abortion
- Clinical presentation:
o Cervix closed
o Bleeding
o No passage of products of conception
- Ultrasound findings:
o Products of conception present
o Fetal activity present

Pregnant patient at 8 weeks of gestation, presenting with minimal bleeding and she denied
passage of anything, upon examination cervical os is closed, her ultrasound (as shown
above), what is the most likely diagnosis?

A. Complete abortion

203 | P a g e
Obstetrics and Gynecology: SMLE Questions and Answers Chapter 2: Gynecology
B. Threatened abortion
C. Missed abortion
D. Anembryonic sac

Correct Answer:
Explanation: (According to UpToDate)
Anembryonic pregnancy
- Refers to a nonviable pregnancy with a gestational sac that does not contain a yolk sac or
embryo.
- Anembryonic pregnancy contrasts with "embryonic or fetal demise" in which an embryo or
fetus is visualized but cardiac activity is not present.

Note:
Ultrasound picture is showing an empty gestational sac

Pregnant at 37 weeks with IUFD, feeling guilty because she smoke 5 cigarettes /day, what
to tell her?

A. Smoking not related to IUFD of IVF


B. Smoking increase rate of IUFD
C. Should stop smoking for future pregnancy
D. Cause not known and need more investigation

Correct Answer: D
Explanation: (According to UpToDate)
The work-up of the patient with a stillbirth is guided by several factors. Approach should be guided
by clinical data, timing of the death, the mother's medical history, whether fetal growth restriction
was present, and sonographic and histopathologic findings.

Pregnant in antenatal visit has history of 4 first trimester abortions what is the highest
diagnostic value?

A. Karyotyping
B. Hysteroscopy
C. Hysterosalpingography
D. Pelvic US

Correct Answer: A
Explanation: (According to RCOG)
Recurrent Pregnancy Loss: Two or more failed clinical pregnancies as documented by
ultrasonography or histopathologic examination
- Risk factors for recurrent miscarriage
o Epidemiological factors

204 | P a g e
Obstetrics and Gynecology: SMLE Questions and Answers Chapter 2: Gynecology
Maternal age and number of previous miscarriages are two independent risk factors
for a further miscarriage.
o Antiphospholipid syndrome
Is the most important treatable cause of recurrent miscarriage.
o Genetic factors
Parental chromosomal rearrangements
§ In approximately 2–5% of couples with recurrent miscarriage, one of the
partners carries a balanced structural chromosomal anomaly: most commonly a
balanced reciprocal or Robertsonian translocation.
Embryonic chromosomal abnormalities
§ In couples with recurrent miscarriage, chromosomal abnormalities of the
embryo account for 30–57% of further miscarriages.
o Anatomical factors
Congenital uterine malformations
The exact contribution that congenital uterine anomalies make to recurrent
miscarriage remains unclear since the prevalence and reproductive implications of
uterine anomalies in the general population are unknown.
The reported prevalence of uterine anomalies in recurrent miscarriage populations
ranges between 1.8% and 37.6%.The prevalence of uterine malformations appears
to be higher in women with second-trimester miscarriages compared with women
who suffer first- trimester miscarriages, but this may be related to the cervical
weakness that is frequently associated with uterine malformation. It has been
reported that women with arcuate uteri tend to miscarry more in the second
trimester while women with septate uteri are more likely to miscarry in the first
trimester.

- What are the recommended investigations of couples with recurrent first-trimester


miscarriage?
o Karyotyping
Cytogenetic analysis should be performed on products of conception of the third
and subsequent consecutive miscarriage(s).
Parental peripheral blood karyotyping of both partners should be performed in
couples with recurrent miscarriage where testing of products of conception reports
an unbalanced structural chromosomal abnormality.
o Anatomical factors
All women with recurrent first-trimester miscarriage and all women with one
or more second-trimester miscarriages should have a pelvic ultrasound to
assess uterine anatomy.
Suspected uterine anomalies may require further investigations to confirm the
diagnosis, using hysteroscopy, laparoscopy or three-dimensional pelvic
ultrasound.
A review75 of studies comparing the diagnostic accuracy of various imaging
modalities has reported that two-dimensional ultrasound scanning and/or
hysterosalpingography can be used as an initial screening test. Combined
hysteroscopy and laparoscopy and possibly three- dimensional ultrasound scanning

205 | P a g e
Obstetrics and Gynecology: SMLE Questions and Answers Chapter 2: Gynecology
should be used for definitive diagnosis.The value of magnetic resonance imaging
scanning remains undetermined.
Hysteroscopy – Hysteroscopy is considered the standard for diagnosis of
intrauterine abnormalities(UTD)

Note:
Both answers (karyotyping and hysteroscopy are correct!!), but chromosomal anomalies are more
common than uterine anomalies to I would go with karyotyping.

206 | P a g e
Obstetrics and Gynecology: SMLE Questions and Answers Chapter 2: Gynecology

Ectopic Pregnancy

A 25-Year-Old female, presented to the ED with vaginal bleeding, nausea, and right lower
abdominal pain and tenderness. History of open appendectomy due to perforated
appendix. Her BP: 90/50 HR: 120 RR: 25. What is diagnosis?

A. Ruptured ectopic pregnancy


B. Ovarian torsion
C. Hemorrhagic cyst
D. Abscess

Correct Answer: A

Another Recall

A 25-Year-Old female, presented to the ED with vaginal bleeding, nausea, and right lower
abdominal pain and tenderness. History of open appendectomy due to perforated
appendix. Her BP: 90/50 HR: 120 RR: 25. What is the possible site of bleeding?

A. Cervix
B. Uterus
C. Fallopian tube

Correct Answer: C
Explanation: (According to Williams Gynecology)
Ectopic pregnancy
- Peritubal adhesions subsequent to salpingitis, appendicitis, or endometriosis are a risk
factor for Ectopic Pregnancy

Patient with tubal Rupture will present with:


- Acute course with sudden and severe lower abdominal and pelvic pain is usually severe and
frequently described as sharp, stabbing, or tearing.
- Signs of hemorrhagic shock (e.g., tachycardia, hypotension, syncope)

Women diagnosed as ectopic pregnancy asks what is the most common predisposing
factor?

A. Previous tubal pregnancy


B. Pelvic Inflammatroy Disease PID

Correct Answer: A
Explanation: (According to UpToDate)

207 | P a g e
Obstetrics and Gynecology: SMLE Questions and Answers Chapter 2: Gynecology
Ectopic pregnancy
- Risk factors:

Case about ectopic pregnancy (they didn’t mention the diagnosis), what is the next step?

A. BHCG quantitative
B. BHCG qualitative
C. Transvaginal Ultrasound

Correct Answer: A
Explanation: (According to UpToDate)
Human chorionic gonadotropin — Serum, rather than urine, hCG is the preferred test for a
pregnant patient with pain and/or bleeding

**Serum = quantitative

A-22-years old female pregnant, with pregnancy test positive at home, came with sever
abdominal pain, in examination cervical is closed, there is fluid collection 15*13. Thin
endometrium and empty uterus. what is the diagnosis?

A. Intact ectopic pregnancy


B. Ruptured ectopic pregnancy
C. Luteal phase

Correct Answer: B
Explanation:
Severe abdominal pain and fluid collection indicates ruptured ectopic.

A 23-Year-Old female, had a history of IVF pregnancy and embryo was transferred 4 weeks
ago, presenting to the ER with sudden severe abdominal pain for 12 hours and she was
hypotensive, what is the most likely diagnosis?

A. Ovarian cyst

208 | P a g e
Obstetrics and Gynecology: SMLE Questions and Answers Chapter 2: Gynecology
B. Ectopic pregnancy

Correct Answer: B
Explanation: (According to William gynecology)
Ectopic pregnancy
Tubal Rupture clinical presentation:
- Acute course with sudden and severe lower abdominal and pelvic pain is usually severe and
frequently described as sharp, stabbing, or tearing.
- Signs of hemorrhagic shock (e.g., tachycardia, hypotension, syncope)

Patient had previous ectopic pregnancy asked about the percentage in the next pregnancy
to be ectopic?

A. 5%
B. 10%
C. 30%
D. 50 %

Correct Answer: B
Explanation: (According to ACOG)
- Risk of ectopic pregnancy (women with a history of one previous ectopic) à 10%
- Risk of ectopic pregnancy (women with a history of two or more previous ectopic) à more than
25%

A 40-years-old female, P4 with a history of tubal ligation 4 years ago after her last
Cesarean section. Her last Menstrual period was 6 weeks ago. Presented to ER with vaginal
spotting, no abdominal pain. O/E: OS is closed. Tenderness and fullness in her abdomen in
Right lower quadrant. What is the best Initial investigation?

A. Pregnancy test
B. US
C. Abdomen

Correct Answer: A
Explanation: (According to Williams Gynecology and Comprehensive Gynecology)
TUBAL STERILIZATION
- LONG-TERM COMPLICATIONS:
o Contraceptive failure

209 | P a g e
Obstetrics and Gynecology: SMLE Questions and Answers Chapter 2: Gynecology
o Approximately 30 percent of pregnancies that follow a failed tubal sterilization
procedure are ectopic. This rate is 20 percent for those following a postpartum
procedure. Thus, any symptoms of pregnancy in a woman after tubal sterilization
must be investigated, and an ectopic pregnancy excluded.
o Because about one third of pregnancies that occur after all tubal sterilizations are
ectopic, women should be counseled that if they do not experience the expected
menses at any time following tubal sterilization before menopause, a test to detect
human chorionic gonadotropin (HCG) should be performed rapidly, and if they are
pregnant, a diagnostic evaluation to exclude the presence of ectopic pregnancy is
necessary.

A 27-Year-Old female presents with acute lower abdominal pain radiating to the left
shoulder, what is the highest diagnostic investigation?

A. Pelvic CT
B. Abdominal MRI
C. Pregnancy test

Correct Answer: C
Explanation: (According to Williams Gynecology)
Ectopic pregnancy
- Diagnosis:
• Serum B-hCG level
Rapid and accurate determination of pregnancy is essential to identify an ectopic
pregnancy.

Patient diagnosed with ectopic pregnancy received methotrexate her bHCG were:

- On day 1 (3000)

- On day 4 (3100)

- On day 7 (2900) what to do next?

A. Nothing
B. Surgery
C. Multi dose methotrexate regimen
D. Consider second dose of methotrexate

210 | P a g e
Obstetrics and Gynecology: SMLE Questions and Answers Chapter 2: Gynecology
Correct Answer: D

Another recall

Patient diagnosed with ectopic pregnancy, She was given methotrexate.

- Day 1 Bhcg was 3100

- Day 4 Bhcg was 3000

- Day 7 Bhcg was 2900.

What to do?

A. Give another dose of methotrexate


B. Don’t give
C. Observation

Correct Answer: A
Explanation: (According to ACOG)
Between 4 and 7 days after methotrexate is a adminstered, the HCG levels should fall at least 15%.
If this amount of decrease does not occur, an additional dose of methotrexate should be given

Patient diagnosed with ectopic pregnancy, started methotrexate 1 weak ago. Follow up
result:

- D1 : BHCG 1000

- D4: BHCG 1200

- D7: BHCG 2200

What is the most appropriate next step?

A. Immediately laparotomy
B. Second dose methotrexate
C. Multiple dose methotrexate

Correct Answer: B
Explanation: (According to ACOG)
Between 4 and 7 days after methotrexate is a adminstered, the HCG levels should fall at least 15%.
If this amount of decrease does not occur, an additional dose of methotrexate should be given

211 | P a g e
Obstetrics and Gynecology: SMLE Questions and Answers Chapter 2: Gynecology

Patient with ectopic pregnancy, her husband is in a military mission, she lives 80Km away
fron hospital, brought by her neighbor, US showed unviable fetus, abscent heart rate, 4cm
fetus, BHCG is 5000, what is the indication for surgical intervension?

A. BHCG
B. Fetal heart
C. Fetal size
D. Far away from hospital or distance

Correct Answer: D
Explanation: (According to ACOG and Williams Gynecology)
Ectopic pregnancy
- Absolute Contraindications to Methotrexate MTX therapy (ACOG)
• Intrauterine pregnancy
• Evidence of immunodeficiency
• Moderate to severe anemia, leukopenia, or thrombocytopenia
• Sensitivity to methotrexate
• Active pulmonary disease
• Active peptic ulcer disease
• Clinically important hepatic dysfunction
• Clinically important renal dysfunction
• Breastfeeding
• Ruptured ectopic pregnancy
• Hemodynamically unstable patient
• Inability to participate in follow-up

Patient with ectopic pregnancy, her husband is in a military mission, she lives 80Km away
fron hospital, brought by her neighbor, US showed unviable fetus, abscent heart rate, 4cm
fetus, BHCG is 5000,

Her vitals: BP 90/50, HR 110, T 35. What is the absolute contraindication for medical
therapy in her condition?

A. HCG
B. Vital Signs
C. Fetal size
D. Far away from hospital or distance

Correct Answer: B
Explanation: (According to ACOG and Williams Gynecology)

212 | P a g e
Obstetrics and Gynecology: SMLE Questions and Answers Chapter 2: Gynecology
Ectopic pregnancy
- Absolute Contraindications to Methotrexate MTX therapy (ACOG)
• Intrauterine pregnancy
• Evidence of immunodeficiency
• Moderate to severe anemia, leukopenia, or thrombocytopenia
• Sensitivity to methotrexate
• Active pulmonary disease
• Active peptic ulcer disease
• Clinically important hepatic dysfunction
• Clinically important renal dysfunction
• Breastfeeding
• Ruptured ectopic pregnancy
• Hemodynamically unstable patient
• Inability to participate in follow-up

Note:
She is hemodynamically unstable which indicates ruptured ectopic. Which is an indication for
LAPAROTOMY

Patient diagnosed with ectopic pregnancy, BhCG 2500, 3cm, what is the contraindications
for methotrexate treatment?

A. BhCg and ultrasound findings


B. Far away from hospital
C. Her vital signs

Correct Answer: B
Explanation: (According to ACOG and Williams Gynecology)
Ectopic pregnancy
- Absolute Contraindications to Methotrexate MTX therapy (ACOG)
• Intrauterine pregnancy
• Evidence of immunodeficiency
• Moderate to severe anemia, leukopenia, or thrombocytopenia
• Sensitivity to methotrexate
• Active pulmonary disease
• Active peptic ulcer disease
• Clinically important hepatic dysfunction
• Clinically important renal dysfunction
• Breastfeeding
• Ruptured ectopic pregnancy
• Hemodynamically unstable patient
• Inability to participate in follow-up
- Relative Contraindications to Methotrexate MTX therapy (ACOG)

213 | P a g e
Obstetrics and Gynecology: SMLE Questions and Answers Chapter 2: Gynecology
• Embryonic cardiac activity detected by transvaginal ultrasonography
• High initial hCG concentration >5000 milli-international units/mL.
• Ectopic pregnancy greater than 4 cm in size as imaged by transvaginal
ultrasonography
• Refusal to accept blood transfusion

A female patient diagnosed as ectopic pregnancy, where she lives far away from the
hospital. Her B-HCG level is 6000 with an absent fetal heartbeat. What is the best
management for her?

A. Methotrexate
B. Surgical intervention
C. Expectant management

Correct Answer: B
Explanation: (According to ACOG and Williams Gynecology)
Ectopic pregnancy
- Absolute Contraindications to Methotrexate MTX therapy (ACOG)
• Intrauterine pregnancy
• Evidence of immunodeficiency
• Moderate to severe anemia, leukopenia, or thrombocytopenia
• Sensitivity to methotrexate
• Active pulmonary disease
• Active peptic ulcer disease
• Clinically important hepatic dysfunction
• Clinically important renal dysfunction
• Breastfeeding
• Ruptured ectopic pregnancy
• Hemodynamically unstable patient
• Inability to participate in follow-up
- Relative Contraindications to Methotrexate MTX therapy (ACOG)
• Embryonic cardiac activity detected by transvaginal ultrasonography
• High initial hCG concentration >5000 milli-international units/mL.
• Ectopic pregnancy greater than 4 cm in size as imaged by transvaginal
ultrasonography
• Refusal to accept blood transfusion

Female patient present in the emergency with lower abdominal pain and bleeding,
ultrasound done and a 3cm ectopic pregnancy found in the ovary. The B-HCG was 15000.
from the above history what will make the medical treatment contraindicated?

A. The size
B. Abdominal pain

214 | P a g e
Obstetrics and Gynecology: SMLE Questions and Answers Chapter 2: Gynecology
C. BHCG level

Correct Answer: C
Explanation: (According to ACOG and Williams Gynecology)
Ectopic pregnancy
- Absolute Contraindications to Methotrexate MTX therapy (ACOG)
• Intrauterine pregnancy
• Evidence of immunodeficiency
• Moderate to severe anemia, leukopenia, or thrombocytopenia
• Sensitivity to methotrexate
• Active pulmonary disease
• Active peptic ulcer disease
• Clinically important hepatic dysfunction
• Clinically important renal dysfunction
• Breastfeeding
• Ruptured ectopic pregnancy
• Hemodynamically unstable patient
• Inability to participate in follow-up
- Relative Contraindications to Methotrexate MTX therapy (ACOG)
• Embryonic cardiac activity detected by transvaginal ultrasonography
• High initial hCG concentration >5000 milli-international units/mL.
• Ectopic pregnancy greater than 4 cm in size as imaged by transvaginal
ultrasonography
• Refusal to accept blood transfusion

Ecopic pregnancy at 7 or 9 week, 4 cm mass. (Bhcg not mentioned). What is the most
appropriate management?

A. Methotrexate
B. Surgical management
C. Observation

Correct Answer: A
Explanation: (According to ACOG)
Ectopic pregnancy
- Methotrexate MTX is the preferred treatment option when all of the following
characteristics are present:
• Hemodynamic stability.
• Serum beta-human chorionic gonadotropin (hCG) concentration ≤5000 milli-
international units/mL.
• No fetal cardiac activity detected on transvaginal ultrasound (TVUS).
• Ectopic mass size less than 4 cm

215 | P a g e
Obstetrics and Gynecology: SMLE Questions and Answers Chapter 2: Gynecology
• Patients are willing and able to comply with post-treatment follow-up and have
access to emergency medical services within a reasonable time frame in case of a
ruptured fallopian tube.

Patient diagnosed with ectopic pregnancy, the ectopic size is less than 3.5 cm, BhCG is 2500
how will you manage?

A. Medical
B. Surgical

Correct Answer: A
Explanation: (According to ACOG)
Ectopic pregnancy
- Methotrexate MTX is the preferred treatment option when all of the following
characteristics are present:
• Hemodynamic stability.
• Serum beta-human chorionic gonadotropin (hCG) concentration ≤5000 milli-
international units/mL.
• No fetal cardiac activity detected on transvaginal ultrasound (TVUS).
• Ectopic mass size less than 4 cm
• Patients are willing and able to comply with post-treatment follow-up and have
access to emergency medical services within a reasonable time frame in case of a
ruptured fallopian tube.

Patient diagnosed with ectopic pregnancy Bhcg 3400, 3 cm. What is the most appropriate
management?

A. Medical treatment
B. Surgical management

Correct Answer: A
Explanation: (According to ACOG)
Ectopic pregnancy
- Methotrexate MTX is the preferred treatment option when all of the following
characteristics are present:
• Hemodynamic stability.
• Serum beta-human chorionic gonadotropin (hCG) concentration ≤5000 milli-
international units/mL.
• No fetal cardiac activity detected on transvaginal ultrasound (TVUS).
• Ectopic mass size less than 4 cm

216 | P a g e
Obstetrics and Gynecology: SMLE Questions and Answers Chapter 2: Gynecology
• Patients are willing and able to comply with post-treatment follow-up and have
access to emergency medical services within a reasonable time frame in case of a
ruptured fallopian tube.

Patient diagnosed with ectopic pregnancy; B-HCG was 5000. What is the most appropriate
management?

A. Methotrexate
B. Surgical management

Correct Answer: A
Explanation: (According to ACOG)
Ectopic pregnancy
- Methotrexate MTX is the preferred treatment option when all of the following
characteristics are present:
• Hemodynamic stability.
• Serum beta-human chorionic gonadotropin (hCG) concentration ≤5000 milli-
international units/mL.
• No fetal cardiac activity detected on transvaginal ultrasound (TVUS).
• Ectopic mass size less than 4 cm
• Patients are willing and able to comply with post-treatment follow-up and have
access to emergency medical services within a reasonable time frame in case of a
ruptured fallopian tube.

A female patient her LMP was 6 weeks ago, presented with mild abdominal pain. Vitally
stable, Closed OS. US shows no intrauterine pregnancy, but a 3 cm sac in the fornix area
with no cardiac activity, BhCG was 3000. What is you management?

A. Medical management
B. Surgical management
C. Medical and surgical management
D. Medical management given that she has access to the hospital

Correct Answer: D
Explanation: (According to ACOG)
Ectopic pregnancy
- Methotrexate MTX is the preferred treatment option when all of the following
characteristics are present:
• Hemodynamic stability.
• Serum beta-human chorionic gonadotropin (hCG) concentration ≤5000 milli-
international units/mL.
• No fetal cardiac activity detected on transvaginal ultrasound (TVUS).
• Ectopic mass size less than 4 cm
217 | P a g e
Obstetrics and Gynecology: SMLE Questions and Answers Chapter 2: Gynecology
• Patients are willing and able to comply with post-treatment follow-up and have
access to emergency medical services within a reasonable time frame in case of a
ruptured fallopian tube.

Note:
A and D are both correct. But D is more accurate.

Patient diagnosed with ectopic pregnancy , her mother refusing OR. her B-hcg 3500 size of
sac 3 nonviable no signs of rupture. what will u do?

A. Explain that failure is high


B. Treat medically but sign consent
C. Laparoscopy
D. Laparotomy

Correct Answer: B
Explanation: (According to ACOG)
Ectopic pregnancy
- Methotrexate MTX is the preferred treatment option when all of the following
characteristics are present:
• Hemodynamic stability.
• Serum beta-human chorionic gonadotropin (hCG) concentration ≤5000 milli-
international units/mL.
• No fetal cardiac activity detected on transvaginal ultrasound (TVUS).
• Ectopic mass size less than 4 cm
• Patients are willing and able to comply with post-treatment follow-up and have
access to emergency medical services within a reasonable time frame in case of a
ruptured fallopian tube.

A case of ectopic pregnancy, plateau of B-HCG for 3 weeks. What to do?

A. Consider methotrexate
B. Reassure and follow up
C. Salpingostomy or Surgical management

Correct Answer: A
Explanation: (According to ACOG)
If hCG levels plateau or increase during follow-up à consider administering methotrexate for
treatment of a persistent ectopic pregnancy

Case of ectopic pregnancy treated by Salpingostomy. On regular follow up her BHCG was
decreasing until the last three visits the BHCG results plateaued. BHCG was 3200 on the
last visit, How would you manage the case?

218 | P a g e
Obstetrics and Gynecology: SMLE Questions and Answers Chapter 2: Gynecology

A. Consider giving Methotrexate


B. Start OCP
C. Surgical intervention
D. Reassure

Correct Answer: A
Explanation: (According to Williams Gynecology)
Persistent Trophoblast
- After surgery, B-hCG levels usually fall quickly and approximate 10 percent of
preoperative values by day 12.
- Persistent trophoblast is rare following salpingectomy but complicates 5 to 15 percent of
salpingostomies.
- Bleeding caused by retained trophoblast is the most serious complication.
- Incomplete removal of trophoblast can be identified by stable or rising B-hCG levels.
- Monitoring approach: measure serum B-hCG levels weekly levels.
- With stable or increasing B-hCG levels, additional surgical or medical therapy is
necessary.
o Without evidence for tubal ruptureà standard therapy for this is single-dose
MTX, 50 mg/m 2 X body surface area (BSA).
o With evidence of rupture and bleeding à require surgical intervention.

Patient diagnosed with ectopic pregnancy, her initial Bchg 108,000 Treated and under
observation

- 1st week 700

- 2 week 300

- 3 week 180

- 4 week 70

- 5 week 300

How to manage it?

A. Give methotrexate.
B. Continue observation.
C. Treat surgically

Correct Answer: A

219 | P a g e
Obstetrics and Gynecology: SMLE Questions and Answers Chapter 2: Gynecology
Explanation: (According to Williams Gynecology)
Persistent Trophoblast
- After surgery, B-hCG levels usually fall quickly and approximate 10 percent of
preoperative values by day 12.
- Persistent trophoblast is rare following salpingectomy but complicates 5 to 15 percent of
salpingostomies.
- Bleeding caused by retained trophoblast is the most serious complication.
- Incomplete removal of trophoblast can be identified by stable or rising B-hCG levels.
- Monitoring approach: measure serum B-hCG levels weekly levels.
- With stable or increasing B-hCG levels, additional surgical or medical therapy is
necessary.
o Without evidence for tubal ruptureà standard therapy for this is single-dose
MTX, 50 mg/m 2 X body surface area (BSA).
o With evidence of rupture and bleeding à require surgical intervention.

Patient had a salpingostomy, she is following up with the BHCG every week, they noticed
the BHCG plateaued for 3 weeks on 3442, what’s next?

A. Laparotomy
B. OCP
C. Methotrexate
D. Reassurance

Correct Answer: C
Explanation: (According to Williams Gynecology)
Persistent Trophoblast
- After surgery, B-hCG levels usually fall quickly and approximate 10 percent of
preoperative values by day 12.
- Persistent trophoblast is rare following salpingectomy but complicates 5 to 15 percent of
salpingostomies.
- Bleeding caused by retained trophoblast is the most serious complication.
- Incomplete removal of trophoblast can be identified by stable or rising B-hCG levels.
- Monitoring approach: measure serum B-hCG levels weekly levels.
- With stable or increasing B-hCG levels, additional surgical or medical therapy is
necessary.
o Without evidence for tubal ruptureà standard therapy for this is single-dose
MTX, 50 mg/m 2 X body surface area (BSA).
o With evidence of rupture and bleeding à require surgical intervention.

A 34-year-old lady pregnant, complaining of amenorrhea, bleeding, and abdominal pain. β-


HCG done showed levels of 1600, she was given methotrexate. One week later she still has
severe abdominal pain despite analgesia. β-HCG done showed 6000 units. What is the best
management?

220 | P a g e
Obstetrics and Gynecology: SMLE Questions and Answers Chapter 2: Gynecology

A. Salpingostomy
B. Salpingectomy
C. Continue methotrexate
D. Exploratory laparotomy

Correct Answer: B
Explanation:
Ectopic pregnancy
Surgical management:
- Laparoscopy is the preferred surgical treatment for ectopic pregnancy unless a woman is
hemodynamically unstable
- Ruptured tubal pregnancies with hemoperitoneum-can safely be managed laparoscopically.
• Salpingectomy
o Standard procedure if the condition of the tube with the ectopic gestation is
damaged (ruptured or otherwise disrupted), bleeding is uncontrolled, or the
gestation appears too large to remove with salpingostomy.

Note:
The severe abdominal pain indicates rupture of ectopic pregnancy à which is an indication of
laparoscopic salpingectomy (unless hemodynamically unstable à Laparotomy)

Ectopic pregnancy bhcg 3500, she is hypotensive and tachypneic. What is the most
appropriate management?

A. Methotrexate
B. Laparoscopy salpingectomy
C. Laparotomy salpingectomy

Correct Answer: C
Explanation: (According to ACOG and Williams Gynecology)
Ectopic pregnancy
Surgical management:
- Laparoscopy is the preferred surgical treatment for ectopic pregnancy unless a woman is
hemodynamically unstable
- Ruptured tubal pregnancies with hemoperitoneum-can safely be managed laparoscopically.
• Salpingectomy
o Standard procedure if the condition of the tube with the ectopic gestation is
damaged (ruptured or otherwise disrupted), bleeding is uncontrolled, or the
gestation appears too large to remove with salpingostomy.

221 | P a g e
Obstetrics and Gynecology: SMLE Questions and Answers Chapter 2: Gynecology
A 25-Year-old female, presented to the ED with vague abdominal pain and amenorrhea for
2 months. She has a history of open appendectomy due to perforated appendix 14 year ago.
Her B-hCG 1800. What’s the most appropriate management?

A. Surgical intervention after stabilizing


B. GS review for acute abdominal
C. Strong analgesic
D. Methotrexate

Correct Answer: D
Explanation:
Ectopic pregnancy
- Peritubal adhesions subsequent to salpingitis, appendicitis, or endometriosis are a risk
factor for Ectopic Pregnancy

- Methotrexate MTX is the preferred treatment option when all of the following
characteristics are present:
• Hemodynamic stability.
• Serum beta-human chorionic gonadotropin (hCG) concentration ≤5000 milli-
international units/mL.
• No fetal cardiac activity detected on transvaginal ultrasound (TVUS).
• Ectopic mass size less than 4 cm
• Patients are willing and able to comply with post-treatment follow-up and have
access to emergency medical services within a reasonable time frame in case of a
ruptured fallopian tube.

Patient diagnosed with ectopic pregnancy on methotrexate and bhcg elevated in day 4 and
7 what to do?

A. Recheck bhcg after 48 hours


B. Recheck bhcg after one week
C. Salpingectomy
D. Discharge

Correct Answer: C
Explanation: (According to Williams Gynecology and ACOG)
Ectopic pregnancy
- Failure of Medical Treatment:
• When the B-hCG level plateaus or rises, fail to decrease adequately by 15% from
days 4 to 7 postinjection or the tube ruptures.
• Managed by:
o A repeat single dose of methotrexate (as indicated)

222 | P a g e
Obstetrics and Gynecology: SMLE Questions and Answers Chapter 2: Gynecology
o Surgical management (if maximum doses of methotrexate have reached or
rupture of tubes)

Ectopic pregnancy, her Initial BHCG 2900, given methotrexate, one week later BHCG
6000, What to do?

A. Repeated methotrexate
B. Diagnostic laparoscopy

Correct Answer: A
Explanation: (According to Williams Gynecology and ACOG)
Ectopic pregnancy
- Failure of Medical Treatment:
• When the B-hCG level plateaus or rises, fail to decrease adequately by 15% from
days 4 to 7 postinjection or the tube ruptures.
• Managed by:
o A repeat single dose of methotrexate (as indicated)
o Surgical management (if maximum doses of methotrexate have reached or
rupture of tubes)

Female had a previous ectopic pregnancy want to conceive, what to do?

A. Close follow up for early detection of fetus location


B. Folic acid therapy
C. Should do IVF

Correct Answer: A
Explanation: (According to Comprehensive Gynecology)
Ectopic pregnancy
- Women with an ectopic pregnancy who become pregnant again should be monitored by
ultrasound early in pregnancy.
- Only about one of three nulliparous women who have had an ectopic pregnancy ever
conceives again (35%), and about one third of these conceptions are an ectopic pregnancy,

After salpingectomy how to follow?

A. One bhcg to confirm decline


B. Hysterosalpingogram

223 | P a g e
Obstetrics and Gynecology: SMLE Questions and Answers Chapter 2: Gynecology
C. Weekly bhcg
D. Pelvic ultrasound after 6 days

Correct Answer: A
Explanation:
Ectopic pregnancy
Follow Up after treatment:
- Serial beta-hCG measured weekly after treatment until the level is undetectable.
- **Exception (According to UTD):
o For patients who undergo salpingectomy-> if the pathology evaluation confirms a tubal
gestation, many surgeons do not check a postoperative hCG, and others check a single
postoperative hCG to confirm a large decline in the level.

Ectopic pregnancy managed with salpingostomy. BHCG postoperative was 3500. how to
follow up the BHCG?

A. No need follow up
B. Pelvic ultrasound
C. Weekly measurements of B-HCG until undetectable

Correct Answer: C
Explanation:
Ectopic pregnancy
Follow Up after treatment:
- Serial beta-hCG measured weekly after treatment until the level is undetectable.
- **Exception (According to UTD):
o For patients who undergo salpingectomy-> if the pathology evaluation confirms a tubal
gestation, many surgeons do not check a postoperative hCG, and others check a single
postoperative hCG to confirm a large decline in the level.

A pregnant lady at 7 weeks of gestation, she came to emergency room complaining of left
iliac fossa pain and brownish vaginal discharge, what is your provisional diagnosis?

A. Ectopic pregnancy
B. Appendicitis
C. Irritable bowel syndrome
D. Threatened miscarriage

Correct Answer: A
Explanation:

224 | P a g e
Obstetrics and Gynecology: SMLE Questions and Answers Chapter 2: Gynecology
She has a localized pain à indicates ectopic more than threateend abortion

A 30-year-old, pregnant women by in vitro fertilization (IVF) and diagnosed with ectopic
pregnancy and scheduled of laproscopic removal of the prgnency. On US imaging: 4 cm
tubal pregnancy on the right, and hydrosalpinx on the left. What is the management?

Question

A. Salpingotomy on the right only


B. Salpingectomy on the right only
C. Removal of both tubes (Bilateral salpingectomy)
D. Hysterosalpingography with contrast

Correct Answer: C
Explanation:
According to Comprehensive Gynecology Textbook
- If the hydrosalpinx is large and clearly visible on ultrasound, it is preferable to perform
laparoscopic salpingectomy prior to IVF-ET because the pregnancy rate with IVF-ET may
be decreased by as much as 40%
- A significantly dilated tube (hydrosalpinx) on ultrasound is an indication for salpingectomy
before carrying out IVF.

According to Dewhurst’s Textbook of Obstetrics and Gynecology:


- In women undergoing IVF, the presence of hydrosalpinx is associated with early pregnancy
loss and poor implantation and pregnancy rates.
- Hydrosalpinges large enough to be visible on ultrasound are associated with the poorest
outcome, including increased miscarriage rates. Women with hydrosalpinges should
therefore be offered treatment before IVF because this improves the chance of a live birth.
- Various surgical treatments including salpingectomy, salpingostomy, proximal tubal
ligation or clipping, and transvaginal aspiration have all been used to improve IVF
outcome.

Patient with a history of perforated ectopic and previous management, came now with
lower abdominal pain. Pregnancy test Lab’s show: 18000 bhcg Hb low. How to mange?

A. Consult surgery for possible acute abdomen


B. Misoprostol
C. Surgical management after stabilization

Correct Answer: C
Explanation: (According to ACOG and Williams gynecology)

225 | P a g e
Obstetrics and Gynecology: SMLE Questions and Answers Chapter 2: Gynecology
Ectopic pregnancy
- Absolute Contraindications to Methotrexate MTX therapy (ACOG)
• Intrauterine pregnancy
• Evidence of immunodeficiency
• Moderate to severe anemia, leukopenia, or thrombocytopenia
• Sensitivity to methotrexate
• Active pulmonary disease
• Active peptic ulcer disease
• Clinically important hepatic dysfunction
• Clinically important renal dysfunction
• Breastfeeding
• Ruptured ectopic pregnancy
• Hemodynamically unstable patient
• Inability to participate in follow-up
- Relative Contraindications to Methotrexate MTX therapy (ACOG)
• Embryonic cardiac activity detected by transvaginal ultrasonography
• High initial hCG concentration >5000 milli-international units/mL.
• Ectopic pregnancy greater than 4 cm in size as imaged by transvaginal
ultrasonography
• Refusal to accept blood transfusion

Female with amenorrhea for 6 weeks, pregnancy test positive, presented with abdominal
pain,, US showed no intrauterine pregnancy with minimal fluid in cul-de-sac and mass of
1.2 cm in tube. She was vitally stable. What's your action?

A. Methotrexate
B. Salpingostomy
C. Salpingectomy

Correct Answer: A
Explanation:
According to Williams Gynecology
- A small amount of peritoneal fluid is physiologically normal.

According to UTD
- A small amount of clear free fluid in the pelvis is a normal sonographic finding. There is no
established threshold for the volume of fluid that is physiologic and the volume is difficult
to measure sonographically. However, fluid that is anechoic and isolated to the pelvic cul-
de-sac and adjacent recesses is likely physiologic
- The presence or absence of peritoneal free fluid is not a reliable indicator of whether an
ectopic pregnancy has ruptured

According to ACOG and Williams gynecology

226 | P a g e
Obstetrics and Gynecology: SMLE Questions and Answers Chapter 2: Gynecology
- Methotrexate MTX is the preferred treatment option when all of the following
characteristics are present:
• Hemodynamic stability.
• Serum beta-human chorionic gonadotropin (hCG) concentration ≤5000 milli-
international units/mL.
• No fetal cardiac activity detected on transvaginal ultrasound (TVUS).
• Ectopic mass size less than 4 cm
• Patients are willing and able to comply with post-treatment follow-up and have
access to emergency medical services within a reasonable time frame in case of a
ruptured fallopian tube.

Note:
If large amount of fluid (indicates hemoperitoneum=ruptured ectopic) à salpingectomy

Ectopic pregnancy received MTX and gradually result in bHCG decreased, what is the
next step?

A. Laparoscopy
B. Give 2nd dose MTX
C. Observe

Correct Answer: C
Explanation:
Answered by exclusion

Ectopic pregnancy is a defect in which process?

A. Implantation
B. Fertilization

Correct Answer: A
Explanation:

A-22-years old female pregnant, with pregnancy test positive at home, came with sever
abdominal pain, in examination cervical is closed, there is fluid collection 15*13. Thin
endometrium and empty uterus. what is the most appropriate management?

227 | P a g e
Obstetrics and Gynecology: SMLE Questions and Answers Chapter 2: Gynecology
A. Endoscopy
B. Conservative
C. Exploratory laparoscopy
D. Methotrexate

Correct Answer: C
Explanation: (According to Williams Gynecology and ACOG)
Ectopic pregnancy
Surgical management:
- Laparoscopy is the preferred surgical treatment for ectopic pregnancy unless a woman is
hemodynamically unstable
- Ruptured tubal pregnancies with hemoperitoneum-can safely be managed laparoscopically.
• Salpingectomy
o Standard procedure if the condition of the tube with the ectopic gestation is
damaged (ruptured or otherwise disrupted), bleeding is uncontrolled, or the
gestation appears too large to remove with salpingostomy.

Patient primigravida at 6 weeks of gestation, presents with moderate vaginal bleeding and
severe lower abdominal pain and radiating to the shoulder. Upon examination: Abdominal
guarding and rebound tenderness. US showed no Intrauterine pregnancy. What is the most
appropriate next step?

A. Laparoscopy
B. Methotrexate
C. Prostaglandin

Correct Answer: A
Explanation: (According to Williams gynecology and ACOG)
Ectopic pregnancy
Surgical management:
- Laparoscopy is the preferred surgical treatment for ectopic pregnancy unless a woman is
hemodynamically unstable
- Ruptured tubal pregnancies with hemoperitoneum-can safely be managed laparoscopically.
• Salpingectomy
o Standard procedure if the condition of the tube with the ectopic gestation is
damaged (ruptured or otherwise disrupted), bleeding is uncontrolled, or the
gestation appears too large to remove with salpingostomy.

228 | P a g e
Obstetrics and Gynecology: SMLE Questions and Answers Chapter 2: Gynecology

Gestational Trophoblastic Disease (Molar pregnancy and Choriocarcinoma)

Patient her LMP 12 weeks ago while physical exam is at 19 weeks. US reveals molar. What
is the management?

A. D&C
B. Methotrexate
C. Suction & Evacuation

Correct Answer: C
Explanation: (According to Williams Gynecology
Molar pregnancy
- Management
o Molar evacuation by suction curettage is usually the preferred treatment.

Molar pregnancy case treated by dilatation and suction. What is the most common early
complication?

A. Perforation
B. Pneumonia
C. Amniotic fluid embolism

Correct Answer: A
Explanation: (According to UpToDate)
Uterine perforation — is the most common immediate complication of D&C.

Common complication after uterine evacuation?

A. Perforation
B. Infection
C. Bleeding
D. Abscess

Correct Answer: B
Explanation: (According to UpToDate)
- Uterine perforation is the most common immediate complication of D&C.
- After the procedure à Infection (rare), intrauterine adhesions

Patient diagnosed with molar pregnancy did evacuation and curettage, bHcg monitoring
after 1 month did not change (didnt increase or decrease), What is the most appropriate
management?
229 | P a g e
Obstetrics and Gynecology: SMLE Questions and Answers Chapter 2: Gynecology

A. Repeat evacuation.
B. Refer to oncology to start methotrexate.
C. Observe as long it’s not elevated.

Correct Answer: B
Explanation: (According to Williams Gynecology and UpToDate)
Molar pregnancy
- Posttreatment surveillance
Serum hCG levels:
o Close biochemical surveillance for persistent gestational neoplasia follows each
hydatidiform mole evacuation.
o This monitoring is by serial measurement of serum B-hCG to detect persistent or
renewed trophoblastic proliferation.
§ The initial B-hCG level is obtained within 48 hours after evacuation. (baseline)
§ Then weekly until undetectable
§ Once B-hCG is undetectable (for three consecutive weeks), this is confirmed
with monthly for another 6 months

**Importantly, during B-hCG level surveillance, either increasing or persistently plateaued levels
mandate evaluation for trophoblastic neoplasia. If the woman has not become pregnant, then these
levels signify increasing trophoblastic proliferation that is most likely malignant.

**Women with GTN are best managed by oncologists

Molar pregnancy follow up of HCG?

A. Weekly until 3 negatives


B. Weekly then monthly for 6 months
C. Monthly then weekly
D. Monthly until negative

Correct Answer: B
Explanation: (According to Williams Gynecology and UpToDate)
Molar pregnancy
- Posttreatment surveillance
Serum hCG levels:
o Close biochemical surveillance for persistent gestational neoplasia follows each
hydatidiform mole evacuation.
o This monitoring is by serial measurement of serum B-hCG to detect persistent or
renewed trophoblastic proliferation.
§ The initial B-hCG level is obtained within 48 hours after evacuation. (baseline)
§ Then weekly until undetectable

230 | P a g e
Obstetrics and Gynecology: SMLE Questions and Answers Chapter 2: Gynecology
§ Once B-hCG is undetectable (for three consecutive weeks), this is confirmed
with monthly for another 6 months

**Importantly, during B-hCG level surveillance, either increasing or persistently plateaued levels
mandate evaluation for trophoblastic neoplasia. If the woman has not become pregnant, then these
levels signify increasing trophoblastic proliferation that is most likely malignant.

**Women with GTN are best managed by oncologists

Patient presenting with uterus size more than gestational age and hemoptysis. What is the
most appropriate next step in management?

A. Biopsy the mass


B. Admit for D&E
C. Admit for staging and chemo
D. Admit for hysterectomy

Correct Answer: C
Explanation: (According to Williams Gynecology)
In case of choriocarcinoma
Once the diagnosis is verified, in addition to a baseline serum B-hCG level and hemogram, a
search for local disease and metastases includes:
- Tests of liver and renal function, transvaginal sonography, chest radiograph, and brain and
abdominopelvic CT scan or MR imaging.
- Less commonly, positron-emission tomographic (PET) scanning and cerebrospinal fluid B-
hCG level determination are used to identify metastases.

**GTN is staged clinically using the system of the International Federation of Gynecology and
Obstetrics (FIGO) and the World Health Organization (WHO) Prognostic Scoring System

Pregnant in her 10th week, presented with nausea, vomiting, abdominal pain and
hemoptysis, her fundal hight is large for gestational age, bhcg levels are extremely high,
speculum exam showed irregular aggressive mass protruding from the cervix and liable to
bleeding when touched, what’s your next step in management?

A. Biopsy the mass


B. Dilatation and evacuation
C. Staging and chemotherapy
D. Hysterectomy

Correct Answer: C
Explanation: (According to Williams Gynecology)

231 | P a g e
Obstetrics and Gynecology: SMLE Questions and Answers Chapter 2: Gynecology
In case of choriocarcinoma
Once the diagnosis is verified, in addition to a baseline serum B-hCG level and hemogram, a
search for local disease and metastases includes:
- Tests of liver and renal function, transvaginal sonography, chest radiograph, and brain and
abdominopelvic CT scan or MR imaging.
- Less commonly, positron-emission tomographic (PET) scanning and cerebrospinal fluid B-
hCG level determination are used to identify metastases.

**GTN is staged clinically using the system of the International Federation of Gynecology and
Obstetrics (FIGO) and the World Health Organization (WHO) Prognostic Scoring System

**If there are findings on examination or imaging that suggest metastatic disease, this supports the
diagnosis. Biopsies should not be performed because GTN lesions are highly vascular and may
cause vigorous bleeding.

Patient with very high bhcg with hemoptysis, what is the first thing to do?

A. Evacuation
B. Radiotherapy
C. Chemotherapy
D. Chest Xray

Correct Answer: D
Explanation: According to Williams Gynecology)
In case of choriocarcinoma
Once the diagnosis is verified, in addition to a baseline serum B-hCG level and hemogram, a
search for local disease and metastases includes:
- Tests of liver and renal function, transvaginal sonography, chest radiograph, and brain and
abdominopelvic CT scan or MR imaging.
- Less commonly, positron-emission tomographic (PET) scanning and cerebrospinal fluid B-
hCG level determination are used to identify metastases.

**GTN is staged clinically using the system of the International Federation of Gynecology and
Obstetrics (FIGO) and the World Health Organization (WHO) Prognostic Scoring System

**If there are findings on examination or imaging that suggest metastatic disease, this supports the
diagnosis. Biopsies should not be performed because GTN lesions are highly vascular and may
cause vigorous bleeding.

Case of molar pregnancy with very high B-Hcg, treated and following up with her every
week. B-hcg level given weekly, dropping until week 5 it was 1. What will you do?

A. Next week B-hCG


B. Next month B-hCG

232 | P a g e
Obstetrics and Gynecology: SMLE Questions and Answers Chapter 2: Gynecology
C. Discharge.
D. Give methotrexate.

Correct Answer: A
Explanation: (According to UpToDate)
Molar pregnancy
- Posttreatment surveillance
Interpretation and management of hCG levels during monitoring
o Decreasing and undetectable hCG levels — is defined as a level that
progressively decreases >10 percent across four values during a three week period
(eg, on days 1, 7, 14, and 21).
o Increasing hCG levels — is defined as a level that progressively increases >10
percent across three values during at least a two week period (eg, on days 1, 7, and
14)
o Plateaued hCG levels — is defined as four measurements that remain within ±10
percent over at least a three week period (eg, days 1, 7, 14, and 21)

A 42-Year-Old lady with a history of molar pregnancy 2 years ago, she wants to conceive.
What to do regarding her history?

A. Early follow up in pregnancy


B. Contraception as she shouldn’t get pregnant

Correct Answer: A
Explanation: (According to Williams Gynecology)
- Women with prior hydatidiform mole sonographic evaluation is recommended in early
pregnancy
- For women with molar pregnancy after these 6 months of surveillance and undetectable B-
hCG level, monitoring is discontinued, and pregnancy allowed.

Patient at 12 weeks of gestation with fundal hight at 19 weeks and bhcg 270000(very high),
what is the most likely diagnosis?

A. Partial mole
B. Complete mole
C. Ectopic pregnancy

Correct Answer: B
Explanation: (According to Williams Gynecology and UpToDate)
Hydatidiform mole
- Clinical presentation
o Common features

233 | P a g e
Obstetrics and Gynecology: SMLE Questions and Answers Chapter 2: Gynecology
§ Vaginal bleeding
§ Pelvic pressure or pain
§ Enlarged uterus (greater than normal)
§ Hyperemesis gravidarum
o Less common or late features
§ Hyperthyroidism — Due to elevation of hCG >100,000 mIU/mL for several
weeks. These patients may present with tachycardia, warm skin, and
tremor. Laboratory evidence of hyperthyroidism is commonly detected in
asymptomatic patients with HM
§ Ovarian theca lutein cysts — Are a form of ovarian
hyperstimulation resulting from high circulating levels of hCG and prolactin
§ Preeclampsia <20 weeks of gestation
§ Passage of hydropic vesicles from the vagina

**Partial mole has less severe symptoms than in complete mole (due to lower levels of B-hCG)

Pregnant at 8 weeks came with very high HCG, severe vomiting and nausea, abdominal
pain, and heavy bleeding, passed some vesicle, on vaginal examination you find some part
of tissue and cervical os opening, uterine examination was bulk, abdominal was tender and
more than 8 weeks, What is the diagnosis ?

A. Complete abortion
B. Incomplete abortion
C. Threatened abortion
D. Complete hydatidiform mole

Correct Answer: D
Explanation: (According to Williams Gynecology and UpToDate)
Hydatidiform mole
- Clinical presentation
o Common features
§ Vaginal bleeding
§ Pelvic pressure or pain
§ Enlarged uterus (greater than normal)
§ Hyperemesis gravidarum
o Less common or late features
§ Hyperthyroidism — Due to elevation of hCG >100,000 mIU/mL for several
weeks. These patients may present with tachycardia, warm skin, and
tremor. Laboratory evidence of hyperthyroidism is commonly detected in
asymptomatic patients with HM
§ Ovarian theca lutein cysts — Are a form of ovarian
hyperstimulation resulting from high circulating levels of hCG and prolactin
§ Preeclampsia <20 weeks of gestation
§ Passage of hydropic vesicles from the vagina

234 | P a g e
Obstetrics and Gynecology: SMLE Questions and Answers Chapter 2: Gynecology

**Partial mole has less severe symptoms than in complete mole (due to lower levels of B-hCG)

Patient with snowstorm appearance of uterus on ultrasound, you will counsel the patient
on?

A. This condition is highly malignant.


B. Risk of infertility must be addressed

Correct Answer: A
Explanation: (According to Williams Gynecology)
- Women with prior hydatidiform mole generally do not have impaired fertility, and their
pregnancy outcomes are usually normal

- Hydatidiform mole is a premalignant disease.

Pregnant woman at 12th weeks of gestation, beta HCG is very high, what is the diagnosis?

A. Down syndrome
B. Molar pregnancy

Correct Answer: B
Explanation: (According to UpToDate)
In case of Down syndrome
- Second-trimester quadruple test: measures the level of the biochemical markers AFP,
unconjugated estriol (uE3), human chorionic gonadotropin (hCG), and dimeric inhibin A
(DIA) in maternal serum
- Ideally performed at 15+0 to 18+6 weeks of gestation

In case of Molar pregnancy


- If the initial hCG level is high (>100,000 mIU/mL), a transvaginal ultrasound should be
performed and will likely demonstrate molar disease if present.
- If the hCG level is high and the ultrasound shows an apparently normal singleton gestation,
the ultrasound and hCG should be repeated in one week to exclude the possible presence of
a twin conception with normal fetus and coexistent molar pregnancy.

Post partum three months, came with history of something that protruded from the cervix
bleeding on touch, what is the management?

A. Immediate D&C
B. Measure hcg after 1 week

235 | P a g e
Obstetrics and Gynecology: SMLE Questions and Answers Chapter 2: Gynecology
C. Biopsy
D. Tests for metastasis

Correct Answer: D
Explanation: (According to UpToDate)
This is a case of Choriocarcinoma following a normal gestation

DIAGNOSIS AND STAGING


GTN is a clinical diagnosis made based upon elevation of serum human chorionic gonadotropin
(hCG), after a nonmolar pregnancy and after other etiologies of an elevated hCG have been
excluded.
Imaging findings of uterine enlargement or pathology consistent with GTN, bilateral ovarian theca
lutein cysts, or metastatic disease support the diagnosis
Unlike other solid tumors, a tissue diagnosis is not required prior to treatment, biopsy is not
required and may cause significant bleeding.

Laboratory evaluation
hCG — An elevated human chorionic gonadotropin (hCG) is often the first evidence of possible
GTN. A serum quantitative hCG should be drawn in all patients with suspected GTN.
- For women with a prior molar pregnancy, serial measurement of hCG is part of
posttreatment surveillance, and an elevation, plateau, or persistence of hCG suggests the
development of GTN.
- For women with no prior history of a molar pregnancy, an elevated hCG may be initially
presumed to be a normal pregnancy. GTN may be suspected if pelvic ultrasound does not
confirm a nonmolar pregnancy (viable intrauterine pregnancy, spontaneous abortion, or
ectopic pregnancy), or in some cases, if the patient is certain that she has not conceived
recently.

Following a nonmolar pregnancy — Women who develop GTN after a nonmolar pregnancy
typically undergo evaluation with serum hCG and ultrasound only after they become symptomatic.
- The diagnosis is made based upon an elevated hCG, with the exclusion of any other
explanation than GTN. This is sufficient for diagnosis even if there is no uterine
enlargement and no evidence of metastatic disease. It is critical to exclude a normal viable
pregnancy and abnormal pregnancies (eg, spontaneous abortion, ectopic pregnancy),
ectopic hCG production by a nontrophoblastic tumor, or other causes of persistent low-
level hCG.
- If there are findings on examination or imaging that suggest metastatic disease, this
supports the diagnosis. Biopsies should not be performed because GTN lesions are highly
vascular and may cause vigorous bleeding. Thus, hemorrhage is common, particularly from
the uterus due to repeated trauma by dilation and curettage. Life-threatening hemorrhage
may necessitate embolization or resection of the affected organ. Unlike other tumors,
histologic confirmation is not necessary for diagnosis, although on rare occasions, a biopsy
may be needed if there is significant question about the diagnosis of GTN.
- Another exception to obtaining a histologic diagnosis is in patients who present with
postpartum or postabortal bleeding, uterine enlargement, or evidence of uterine disease on

236 | P a g e
Obstetrics and Gynecology: SMLE Questions and Answers Chapter 2: Gynecology
imaging. In these patients, a uterine curettage may be performed and the diagnosis can be
confirmed based upon the pathology evaluation of the curettage specimen.

237 | P a g e
Obstetrics and Gynecology: SMLE Questions and Answers Chapter 2: Gynecology

Adnexal masses

How to defrinitaite between benign vs malignant ovarian tumor?

A. Hypoechoic
B. Bilateral
C. Cyst with septate

Correct Answer: A
Explanation: (According to ACOG)
What ultrasound findings suggest malignancy of adnexal mass?
- Cyst size greater than 10 cm
- Papillary or solid components
- Irregularity
- Presence of ascites
- High color Doppler flow.

**Hypoechoic mass on US means a more solid tissue.

238 | P a g e
Obstetrics and Gynecology: SMLE Questions and Answers Chapter 2: Gynecology

Leiomyoma (Uterine Fibroids) and Leiomyosarcoma

A 33-year-old women presenting with heavy menstruation. During examination you noticed
firm fundal mass, what is the next step in management?

A. CT
B. US
C. MRI
D. Biopsy

Correct Answer: B
Explanation: (According to Uptodate Williams Gynecology, Comprehensive Gynecology)
Leiomyoma
- Investigations
• Initial:
o Pelvic ultrasound
§ First-line study used to evaluate for uterine fibroids.
Fibroids are seen on ultrasound usually as hypoechoic, well-
circumscribed round masses, frequently with shadowing
o

A 30-Year-old and has fibroid 10cm, presented with heavy vaginal bleeding and doctor
want to do hysterectomy but she refused, what is the most appropriate treatment?

A. Uterine artery embolization


B. OCP
C. Observation

Correct Answer: A
Explanation: (According to Uptodate Williams Gynecology, Comprehensive Gynecology)
Leiomyoma
- Management
• Medical Management:
o Uterine Artery Embolization UAE
§ For women who might otherwise be considered candidates for
hysterectomy or myomectomy
§ Women choosing a conservative approach to preserve fertility should
have a surgical myomectomy rather than UAE.
§ Candidates:
• Heavy menstrual bleeding or dysmenorrhea caused by
intramural fibroids
• Premenopausal
• No desire for future pregnancy

239 | P a g e
Obstetrics and Gynecology: SMLE Questions and Answers Chapter 2: Gynecology
A45 year-old female, on pelvic US she was found to have uterine fibroid at the fundus
measuring 5x6cm, and totally asymptomatic, what is the best management?

A. Follow up with pelvic US yearly.


B. Follow up pelvic US and CBC every 3 months
C. Myomectomy
D. Hysterectomy

Correct Answer: A
Explanation: (According to Uptodate Williams Gynecology, Comprehensive Gynecology)
Leiomyoma
- Management
• Observation
o Regardless of their size, asymptomatic leiomyomas usually can be observed
and surveilled with an annual pelvic examination

A 39 Year-old female, has asymptomatic fibroid 5cm, most appropriate management?

A. Myomectomy
B. Biopsy
C. Hysterectomy
D. Clinical surveillance

Correct Answer: D
Explanation: (According to Uptodate Williams Gynecology, Comprehensive Gynecology)
Leiomyoma
- Management
• Observation
o Regardless of their size, asymptomatic leiomyomas usually can be observed
and surveilled with an annual pelvic examination

A 37-year-old woman comes to you with infertility. Semen analysis of the husband is
normal, and the woman has normal, regular cycles and normal FSH and LH levels.
However, US revealed many fibroids disrupting the uterine cavity. What is the best way to
restore her fertility?

A. Surgical removal of the fibroid


B. Reassurance
C. GnRH to help shrink the fibroid

Correct Answer: A
Explanation: (According to Uptodate Williams Gynecology, Comprehensive Gynecology)
Leiomyoma
- Management

240 | P a g e
Obstetrics and Gynecology: SMLE Questions and Answers Chapter 2: Gynecology
• Surgical management

• Hysteroscopic myomectomy
- For patients desiring fertility who present with heavy
menstrual bleeding (HMB) and a submucosal fibroid
or fibroids (FIGO type 0, type 1)

Patient with submucosal fibroid, she wants to preserve her fertility, what is the most
appropriate treatment?

A. Hysteroscopic myomectomy
B. Laparoscopic myomectomy
C. Laparotomy myomectomy

Correct Answer: A

Another recall

A woman with submucosal fibroid that is causing her infertility what is the most
appropriate treatment?

A. Hysteroscopic myomectomy
B. Laparoscopic myomectomy
C. Laparotomy myomectomy
D. Laparoscopic hysterectomy

Correct Answer: A
Explanation: (According to Uptodate Williams Gynecology, Comprehensive Gynecology)
Leiomyoma
- Management
• Surgical management

• Hysteroscopic myomectomy
- For patients desiring fertility who present with heavy
menstrual bleeding (HMB) and a submucosal fibroid
or fibroids (FIGO type 0, type 1)

Female with history of previous abortion, she is having multiple uterine fibroids. Which of
the following will help her the most to get pregnant?

A. Myomectomy
B. Uterine artery embolization
C. Medical therapy

241 | P a g e
Obstetrics and Gynecology: SMLE Questions and Answers Chapter 2: Gynecology
D. GNRH to decrease the size of fibroid

Correct Answer: A
Explanation: (According to Uptodate Williams Gynecology, Comprehensive Gynecology)
Leiomyoma
- Management
• Surgical management

• Hysteroscopic myomectomy
- For patients desiring fertility who present with heavy
menstrual bleeding (HMB) and a submucosal fibroid
or fibroids (FIGO type 0, type 1)

A 32 Year-old female, presented with menorrhagia and on US fibroid measuring 4x4cm,


she wants to preserve fertility, what is the most appropriate management?

A. NSAID
B. Myomectomy
C. Hysterectomy
D. Uterine artery ligation

Correct Answer: B
Explanation: (According to Uptodate Williams Gynecology, Comprehensive Gynecology)
Leiomyoma
- Management
• Surgical management

• Hysteroscopic myomectomy
- For patients desiring fertility who present with heavy
menstrual bleeding (HMB) and a submucosal fibroid
or fibroids (FIGO type 0, type 1)

A 50-year-old female, complaining of chronic lower abdominal pain and prevaginal


bleeding Imaging shows: fibroid. What’s the most appropriate management?

A. Hysterectomy
B. Combined OCP
C. Myomectomy
D. Dilation and curettage

Correct Answer: A
Explanation: (According to Uptodate Williams Gynecology, Comprehensive Gynecology)

242 | P a g e
Obstetrics and Gynecology: SMLE Questions and Answers Chapter 2: Gynecology
Leiomyoma
- Management
• Surgical Management:
o Hysterectomy
§ Definitive treatment
§ Indication:
• AUB and bulk symptoms in patients who do not desire future
childbearing or do not wish to retain their uterus
• Persistent symptoms despite conservative efforts.
• Who have significant concomitant diseases, such as
adenomyosis, cervical dysplasia, or uterine prolapse, for
which hysterectomy is curative for both conditions.

A 42-Year-old female, complaining of heavy menstrual bleeding, on examinations uterus is


normal and ovaries are not palbable, on US there is 3-2 lesions on uterus. What is the most
likely diagnosis?

A. Adenomyosis
B. Submucosal leiomyoma

Correct Answer: B
Explanation: (According to Uptodate Williams Gynecology, Comprehensive Gynecology)
Leiomyoma
- Clinical presentation
• Symptoms:
o Majority are small and asymptomatic.
o Bleeding
§ Most common symptom
§ Heavy and/or prolonged menses is the typical bleeding pattern with
leiomyomas and the most common fibroid symptom
§ Submucosal myomas that protrude into the uterine cavity (eg, types 0
and 1) are most frequently related to significant heavy menstrual
bleeding
o Pressure and pain
§ Large uterus can cause chronic pressure, urinary frequency,
incontinence, or constipation.
o Fertility issues
§ Subfertility is more closely associated with submucous leiomyomas
than with tumors located elsewhere

A 40-year-old female presented with severe menorrhagia, US revealed uterine mass, what
is the most likely diagnosis?

243 | P a g e
Obstetrics and Gynecology: SMLE Questions and Answers Chapter 2: Gynecology

A. Parasitic fibroid
B. Subserosal fibroid
C. Submucosal fibroid

Correct Answer: C
Explanation: (According to comprehensive gynecology)
Although only 5% to 10% of myomas become submucosal, they usually are the most troublesome
clinically. These submucosal tumors may be associated with abnormal vaginal bleeding or
distortion of the uterine cavity that may produce infertility or miscarriage.

Patient with uterine fibroids booking for operation, what drug will be given to reduce the
fibroid size?

A. Steroids
B. GnRH
C. Contraceptive pills
D. Progesterone only pills

Correct Answer: B
Explanation: (According to Uptodate Williams Gynecology, Comprehensive Gynecology)
Leiomyoma
- Management
• Medical Management:
o GnRH receptor agonist
§ Primarily used as either preoperative therapy (typically three to six
months in duration) or as transitional therapy for patients in late
perimenopause as they move to menopause.
§ GnRH agonists shrink leiomyomas by targeting the growth effects of
estrogen and progesterone

Patient with fibroid, what contraceptive to give?

A. Condom
B. IUD
C. OCP
D. POP

Correct Answer: C
Explanation: (According to Uptodate Williams Gynecology, Comprehensive Gynecology)
Leiomyoma
- Management
• Medical Management:
o Estrogen-progesterone contraceptives (COC)

244 | P a g e
Obstetrics and Gynecology: SMLE Questions and Answers Chapter 2: Gynecology
§ Most common medical therapy utilized by patients with HMB and
fibroids, especially those who desire contraception.

Patient presented with heavy menstrual bleeding with hemoglobin 6 and MRI revealed
large fibroid, what is the next step in management?

A. Correct anemia
B. OCP
C. Myomectomy

Correct Answer: A
Explanation: (According to Jeffcoates Principles of Gynecology)
General treatment of leiomyoma:
- Since the patient is usually anaemic it is important to investigate and to correct the anaemia
before and after any operation is undertaken.

What is the commonest type of degenerative of fibroid in pregnancy?

A. Red
B. Cystic
C. Fatty
D. Hyaline

Correct Answer: A
Explanation: (According to Jeffcoates Principles of Gynecology)
Red degeneration is mostly seen during pregnancy

Most common uterine fibroid location?

A. Submucosal
B. Intramural
C. Subserosal

Correct Answer: B

245 | P a g e
Obstetrics and Gynecology: SMLE Questions and Answers Chapter 2: Gynecology
Postmenopausal women on tamoxifen, with a history of fibroid “certain size” and now
increased, endometrial thickness 5mm. What’s the diagnosis?

A. Leiomyosarcoma
B. Endometrial cancer
C. Adenomyosis

Correct Answer: A
Explanation: (According to uptodate)
- Tamoxifen has been associated with abnormal uterine bleeding as well as an increased risk
of both endometrial cancer and uterine sarcoma.
- For postmenopausal patients, a new or growing uterine mass warrants further evaluation for
uterine sarcoma.

Note:
- If patient is on Tamoxifen and her endometrium increased in thickness with Abnormal
uterine bleeding ONLY à choose endometrial cancer.
- If patient is on Tamoxifen + fibroid increased in size (regardless of the endometrial
thickness) à choose Leiomyosarcoma.

A 59-Year-old female came for checkup she stated that she had her routine gynecological
checkup 12 years ago when she was still pre-menopausal and it showed fibroid it was small
(5cm) and it wasn't treated. Now when you examined her the fibroid increased in the size
and you found endometrial thickening, what is the most likely diagnosis‫؟‬

A. Leiomyoma
B. Leiomyosarcoma
C. Metastasis from ovarian cancer

Correct Answer: B
Explanation: (According to uptodate)
- For postmenopausal patients, a new or growing uterine mass warrants further evaluation for
uterine sarcoma.

A 60-year-old lady, with a current 5 cm fibroid, you checked her previous ultrasonographic
results before 5 years and found that it has the same size of 5 cm. What to do in regards to
management?

A. Myomectomy
B. Hysterectomy
C. Endometrial sampling

246 | P a g e
Obstetrics and Gynecology: SMLE Questions and Answers Chapter 2: Gynecology
Correct Answer: C
Explanation: (According to Uptodate)
Endometrial sampling:
- Since endometrial biopsy is a minimally invasive procedure and is already indicated in
many patients with abnormal uterine bleeding to exclude endometrial neoplasia, we suggest
endometrial sampling in patients in whom sarcoma is suspected

247 | P a g e
Obstetrics and Gynecology: SMLE Questions and Answers Chapter 2: Gynecology

Endometriosis

Doctor while taking a history, He found history of severe dysmenorrhea What would be
likely the cause?
A. Ovarian thecoma
B. Ovarian fibroma
C. Endometriosis
D. Leiomyoma

Correct Answer: C
Explanation: (According to Uptodate)
Common presenting symptoms of endometriosis include chronic abdominal/pelvic pain and/or
pressure, severe dysmenorrhea, dyspareunia, heavy menstrual bleeding, and infertility

Female with dyschazia, pain before menstruation and infertility and laparoscopy was
performed (finding in the picture), what is the diagnosis?
A. Adenomyosis
B. Pelvic inflammatory disease
C. Endometriosis

Correct Answer: C
Explanation: (According to Uptodate)
- Diagnosis of endometriosis:
• Laparoscopy is the gold standard. Visual appearance of lesions: areas of peritoneal
endometriosis appear as raised flame-like patches, whitish opacifications, yellow-brown
discolorations, translucent blebs, or reddish or reddish-blue irregularly-shaped islands
- Presenting symptoms:
• Common symptoms –include chronic abdominal/pelvic pain and/or pressure, severe
dysmenorrhea, dyspareunia, heavy menstrual bleeding, and infertility
• Women with bowel endometriosis can present with diarrhea, constipation, dyschezia,
and bowel cramping

Patient has heavy bleeding and usually changes tampon 15-30 times/day, recently she has
been engaged in intercourse with multiple partners to conceive, on examination:

248 | P a g e
Obstetrics and Gynecology: SMLE Questions and Answers Chapter 2: Gynecology
retroverted uterus, decreased uterine mobility, and tender uterosacral ligament with
nodularity. What’s the diagnosis?

A. PID
B. Endometriosis
C. Leiomyoma
D. Adenomyosis

Correct Answer: B
Explanation: (According to Canadian Guidelines, Williams Gynecology, Comprehensive Gynecology)
Endometriosis
- Clinical presentation
• Symptoms:
o Painful menstruation (dysmenorrhea). Unlike primary dysmenorrhea, the
pain may last for many days, including several days before and after the
menstrual flow.
o Painful intercourse (dyspareunia)
o Painful micturition (dysuria)
o Painful defecation (dyschezia)
o Lower back or abdominal discomfort. It may be unilateral or bilateral and
may radiate to the lower back, legs, and groin.
o Chronic pelvic pain (non-cyclic abdominal and pelvic pain of at least 6
months’ duration)
o Abnormal uterine bleeding
o Infertility
o Atypical presentations suggesting more significant disease involvement
§ Cyclic leg pain or sciatica (nerve involvement)
§ Cyclic rectal bleeding or hematuria (bowel or bladder invasion)
§ Cyclic dyspnea secondary to catamenial pneumothorax.
o Many women are asymptomatic; the lesions may be an incidental finding at
surgery.

• Physical examination:
o Retroverted uterus
o Fixed uterus
o Nodularity of the uterosacral ligaments and cul-de-sac may be palpated on
rectovaginal examination.
o The ovaries may be enlarged and tender and are often fixed to the broad
ligament or lateral pelvic sidewall

A 24-Year-old female complaining of severe abdominal cramps during menses not relieved
by NSAIDs and history of infertility for 6 years, What is the most likely diagnosis?

A. Adenomyosis
B. Endometriosis

249 | P a g e
Obstetrics and Gynecology: SMLE Questions and Answers Chapter 2: Gynecology
C. Uterine leiomyoma

Correct Answer: B
Explanation: (According to Canadian Guidelines, Williams Gynecology, Comprehensive Gynecology)
Endometriosis
- Clinical presentation
• Symptoms:
o Painful menstruation (dysmenorrhea). Unlike primary dysmenorrhea, the
pain may last for many days, including several days before and after the
menstrual flow.
o Painful intercourse (dyspareunia)
o Painful micturition (dysuria)
o Painful defecation (dyschezia)
o Lower back or abdominal discomfort. It may be unilateral or bilateral and
may radiate to the lower back, legs, and groin.
o Chronic pelvic pain (non-cyclic abdominal and pelvic pain of at least 6
months’ duration)
o Abnormal uterine bleeding
o Infertility
o Atypical presentations suggesting more significant disease involvement
§ Cyclic leg pain or sciatica (nerve involvement)
§ Cyclic rectal bleeding or hematuria (bowel or bladder invasion)
§ Cyclic dyspnea secondary to catamenial pneumothorax.
o Many women are asymptomatic; the lesions may be an incidental finding at
surgery.

A 28-year-old female complaining of dysmenorrhea and dyspareunia, pelvic ultrasound


done and shows a hypoechoic mass (ground glass) what statement is true?

A. Highly malignant
B. Partner needs to be treated.
C. Antibiotic should be given.
D. Future fertility is a concern.

Correct Answer: D
Explanation: (According to Uptodate)
Endometriosis has been estimated to affect 10 to 15 percent of women of reproductive age [1]. Of
women with endometriosis, 30 to 50 percent are estimated to have infertility

250 | P a g e
Obstetrics and Gynecology: SMLE Questions and Answers Chapter 2: Gynecology

Female patient with dysmenorrhea and you suspect endometriosis, what is the most
appropriate step?
A. US
B. Diagnostic laparoscope
C. CT abdomen

Correct Answer: B
Explanation: (According to Uptodate)
- Diagnosis of endometriosis:
• Laparoscopy is the gold standard.

Most appropriate test to diagnose endometriosis?

A. Ultrasonography
B. MRI
C. Diagnostic laparoscopy
D. Endometrial biopsy

Correct Answer: C
Explanation: (According to Uptodate)
- Diagnosis of endometriosis:
• Laparoscopy is the gold standard.

What is the risk of malignancy in endometriosis?

A. No risk
B. Risk of epithelial ovarian tumor

Correct Answer: B
Explanation: (According to Uptodate and Canadian guidelines)
The risk of malignant transformation of endometriosis has been estimated at 1 percent for
premenopausal females and 1 to 2.5 percent for postmenopausal females.
Endometriosis appears to be associated with some epithelial ovarian cancers

251 | P a g e
Obstetrics and Gynecology: SMLE Questions and Answers Chapter 2: Gynecology
Patient with dysmenorrhea and she use NSAID with no improvement, laparoscopy was
performed (finding in the picture), which one most appropriate drug for her?

A. OCP
B. IUCD
C. Laparoscopy
D. Hysterectomy

Correct Answer: A
Explanation: (According to Canadian Guidelines, Williams Gynecology, Comprehensive Gynecology)
Endometriosis
- Management
Medical (Laparoscopy is no required before medical management)
• First line
o Combined estrogen and progestin therapy

• Second line
o Intrauterine progestin releasing system IUD
o GnRH agonists with HT add-back

Surgical
• Conservative
o Laparoscopic ablation, lysis or excision of the lesions
§ Indications:
• Women of reproductive age who wish to conceive in the
future
• To avoid induction of menopause at an early age
• Women who do not respond to medical therapy or who have
recurrent pain symptoms.

• Definitive:
o Bilateral oophorectomy
§ Indications:
• Women who have significant pain and symptoms despite
conservative treatment
• Do not desire future pregnancies and have severe disease.
• Undergoing hysterectomy because of other pelvic conditions,
such as fibroids or menorrhagia

Patient with endometriosis not responding to OCP, what is the next step in management?

A. Laparoscopy
B. Hysterectomy

252 | P a g e
Obstetrics and Gynecology: SMLE Questions and Answers Chapter 2: Gynecology

Correct Answer: A
Explanation: (According to Canadian Guidelines, Williams Gynecology, Comprehensive Gynecology)
Endometriosis
- Management
Medical (Laparoscopy is no required before medical management)
• First line
o Combined estrogen and progestin therapy

• Second line
o Intrauterine progestin releasing system IUD
o GnRH agonists with HT add-back

Surgical
• Conservative
o Laparoscopic ablation, lysis or excision of the lesions
§ Indications:
• Women of reproductive age who wish to conceive in the
future
• To avoid induction of menopause at an early age
• Women who do not respond to medical therapy or who have
recurrent pain symptoms.

• Definitive:
o Bilateral oophorectomy
§ Indications:
• Women who have significant pain and symptoms despite
conservative treatment
• Do not desire future pregnancies and have severe disease.
• Undergoing hysterectomy because of other pelvic conditions,
such as fibroids or menorrhagia

Best management for endometriosis?

A. OCP
B. IUCD
C. NSAID

Correct Answer: A
Explanation: (According to Canadian Guidelines, Williams Gynecology, Comprehensive Gynecology)
Endometriosis
- Management
Medical (Laparoscopy is no required before medical management)
• First line

253 | P a g e
Obstetrics and Gynecology: SMLE Questions and Answers Chapter 2: Gynecology
o Combined estrogen and progestin therapy

• Second line
o Intrauterine progestin releasing system IUD
o GnRH agonists with HT add-back

Surgical
• Conservative
o Laparoscopic ablation, lysis or excision of the lesions
§ Indications:
• Women of reproductive age who wish to conceive in the
future
• To avoid induction of menopause at an early age
• Women who do not respond to medical therapy or who have
recurrent pain symptoms.

• Definitive:
o Bilateral oophorectomy
§ Indications:
• Women who have significant pain and symptoms despite
conservative treatment
• Do not desire future pregnancies and have severe disease.
• Undergoing hysterectomy because of other pelvic conditions,
such as fibroids or menorrhagia

Note:
Best and definitive treatment is bilateral oophorectomy, but I chose OCP as there is no bilateral
oophorectomy in the choices.

39 years old female who has three children and completed her family had right
oophorectomy for right endometrioma 2 years ago, she presented to the clinic with mild to
moderate dysmenorrhea and dyspareunia during intercourse and chronic lower abdominal
pain. Pelvic ultrasound shows: Left ovary endometrioma cyst 6x7 in size. What is most
appropriate management?

A. Removal of cyst more than 10 in size


B. Aspiration of cyst content under ultrasound guidance
C. Oophorectomy with ablation of other lesions
D. Removal of cyst by laparoscopic ablation of endometrioma spots.

Correct Answer: C
Explanation: (According to Canadian Guidelines, Williams Gynecology, Comprehensive Gynecology)
Endometriosis
- Management

254 | P a g e
Obstetrics and Gynecology: SMLE Questions and Answers Chapter 2: Gynecology
Medical (Laparoscopy is no required before medical management)
• First line
o Combined estrogen and progestin therapy

• Second line
o Intrauterine progestin releasing system IUD
o GnRH agonists with HT add-back

Surgical
• Conservative
o Laparoscopic ablation, lysis or excision of the lesions
§ Indications:
• Women of reproductive age who wish to conceive in the
future
• To avoid induction of menopause at an early age
• Women who do not respond to medical therapy or who have
recurrent pain symptoms.

• Definitive:
o Bilateral oophorectomy
§ Indications:
• Women who have significant pain and symptoms despite
conservative treatment
• Do not desire future pregnancies and have severe disease.
• Undergoing hysterectomy because of other pelvic conditions,
such as fibroids or menorrhagia

Post menopause, presenting with abdominal pain and dyschezia, last menstrual period was
15 months ago what to give?

A. Depo provera injection (Progestogen only)


B. Conjecated estrogen pill
C. OCP

Correct Answer: A
Explanation: (According to Uptodate)
Postmenopausal endometriosis:
- Progestogen only is the first line treatment
- OCP is the alternative first line

255 | P a g e
Obstetrics and Gynecology: SMLE Questions and Answers Chapter 2: Gynecology

Adenomyosis

A 45 year old female, P6+0, presented complaining of chronic abdominal pain, heavy
menstruation and vaginal bleeding. Imaging shows: Thickened myometrium What's the
most definitive management?

A. NSAID’s
B. OCP
C. Hysterectomy
D. Uterine artery ligation

Correct Answer: C
Explanation: (According to Williams Obstetrics, Comprehensive Gynecology, Uptodate)
Adenomyosis
- Management
• Conservative: For symptomatic relief of pain and bleeding
o NSAIDS
o Levonorgestrel (LNG)-releasing intrauterine device (IUD)
o Estrogen-progestin contraceptives pills COC
o Uterine artery embolization
For patients who have completed childbearing, who decline, or have
contraindications to a hysterectomy or have failed hormonal treatment.

• Definitive:
o Hysterectomy

A 41-year-old P5 +3 presented to the clinic complaining of abnormal uterine bleeding her


Menstrual period is regular every 30-days associated with blood clots and pain that is not
relieved by simple analgesic, she had a history of previous myomectomy. She is a known case of
PCOS and BMI is 40. What is the most likely diagnosis?

A. Adenomyosis
B. Endometriosis
C. Uterine fibroids
D. Endometrial hyperplasia

Correct Answer: A
Explanation: (According to Williams Obstetrics, Comprehensive Gynecology, Uptodate)
Adenomyosis
- Risk factors:
• Multiparity
• Age 40-50

256 | P a g e
Obstetrics and Gynecology: SMLE Questions and Answers Chapter 2: Gynecology
• Prior uterine surgery
• Uterine leiomyomas
• Endometriosis

- Clinical presentation
• Symptoms:
o Menorrhagia
o Dysmenorrhea
o Dyspareunia

• Physical examination:
o Uterus is diffusely enlarged “Globular or boggy”
o Tender immediately before and during menstruation
o Some patients have a normal sized uterus.

A 45-Year-old female, P4, complaining of dysmenorrhea. One of her pregnancy was delivered
by CS and all of them was induced. Examination shows normal uterus not enlarged not tender
and no adnexal masses. What is the most likely diagnosis?

A. Endometriosis
B. Adenomyosis
C. Fibroid
D. Pelvic congestion syndrome

Correct Answer: B
Explanation: (According to Williams Obstetrics, Comprehensive Gynecology, Uptodate)
Adenomyosis
- Risk factors:
• Multiparity
• Age 40-50
• Prior uterine surgery
• Uterine leiomyomas
• Endometriosis

- Clinical presentation
• Symptoms:
o Menorrhagia
o Dysmenorrhea
o Dyspareunia

• Physical examination:
o Uterus is diffusely enlarged “Globular or boggy”
o Tender immediately before and during menstruation
o Some patients have a normal sized uterus.
257 | P a g e
Obstetrics and Gynecology: SMLE Questions and Answers Chapter 2: Gynecology

A 50 years old, presented with heavy painful menstrual bleeding in each cycle With previous
history of myomectomy 12 years ago, on examination: bulky uterus, no other signs no US
finding. What is the most likely diagnosis?

A. Leiomyoma
B. Endometriosis
C. Adenomyosis
D. Endometrial cancer

Correct Answer: C
Explanation: (According to Williams Obstetrics, Comprehensive Gynecology, Uptodate)
Adenomyosis
- Risk factors:
• Multiparity
• Age 40-50
• Prior uterine surgery
• Uterine leiomyomas
• Endometriosis

- Clinical presentation
• Symptoms:
o Menorrhagia
o Dysmenorrhea
o Dyspareunia

• Physical examination:
o Uterus is diffusely enlarged “Globular or boggy”
o Tender immediately before and during menstruation
o Some patients have a normal sized uterus.

A 42 years with secondary dysmenorrhea and heavy menses, on pelvic examination you found
symmetrical uterine enlargement. What is the most likely diagnosis?

A. Leiomyoma
B. Endometriosis
C. Adenomyosis
D. Endometrial cancer

Correct Answer: C
Explanation: (According to Williams Obstetrics, Comprehensive Gynecology, Uptodate)

258 | P a g e
Obstetrics and Gynecology: SMLE Questions and Answers Chapter 2: Gynecology
Adenomyosis
- Risk factors:
• Multiparity
• Age 40-50
• Prior uterine surgery
• Uterine leiomyomas
• Endometriosis

- Clinical presentation
• Symptoms:
o Menorrhagia
o Dysmenorrhea
o Dyspareunia

• Physical examination:
o Uterus is diffusely enlarged “Globular or boggy”
o Tender immediately before and during menstruation
o Some patients have a normal sized uterus.

A 42 Years-old female, with heavy menstrual bleeding and dysmenorrhea, examination reveals
bulky tender uterus, how to confirm the diagnosis?

A. Transvaginal US
B. MRI
C. Histopathology

Correct Answer: C
Explanation: (According to Williams Obstetrics, Comprehensive Gynecology, Uptodate)
Adenomyosis
- Investigations
• Initial:
o Transvaginal ultrasonography TVS (first line)
o MRI (second line)

• Definitive:
o Histopathology

A 45 Years-old patient with heavy menstrual bleeding with clots. Had a history of
myomectomy. On physical examination uterus is bulky. What is the next step?

A. CT
B. Transvaginal US
C. MRI
D. Hysterectomy

259 | P a g e
Obstetrics and Gynecology: SMLE Questions and Answers Chapter 2: Gynecology

Correct Answer: B
Explanation: (According to Williams Obstetrics, Comprehensive Gynecology, Uptodate)
Adenomyosis
- Investigations
• Initial:
o Transvaginal ultrasonography TVS (first line)
o MRI (second line)

• Definitive:
o Histopathology

Female P6, presented with secondary dysmenorrhea and abnormal uterine bleeding. On
physical examination enlarged tender uterus. What is the definitive treatment?

A. NSAID
B. Hysterectomy
C. OCP
D. Intrauterine device

Correct Answer: B
Explanation: (According to Williams Obstetrics, Comprehensive Gynecology, Uptodate)
Adenomyosis
- Management
• Conservative: For symptomatic relief of pain and bleeding
o NSAIDS
o Levonorgestrel (LNG)-releasing intrauterine device (IUD)
o Estrogen-progestin contraceptives pills COC
o Uterine artery embolization
For patients who have completed childbearing, who decline, or have
contraindications to a hysterectomy or have failed hormonal treatment.

• Definitive:
o Hysterectomy

260 | P a g e
Obstetrics and Gynecology: SMLE Questions and Answers Chapter 2: Gynecology

Asherman’s Syndrome

A pregnant lady G3p0, A2 now at 5 weeks presented with spotting on examination open os
and no active bleed. History showed 2 abortions at 2nd trimester, last one with D&C
diagnosed as incompetent cervix. Your diagnosis now for the third pregnancy of this
patient?

A. Asherman syndrome
B. Incompetent cervix
C. Chromosomal abnormalities

Correct Answer: C
Explanation: (According to ACOG)
- Approximately 50% of all cases of early pregnancy loss are due to fetal chromosomal
abnormalities
- In rare cases, after a D&C has been performed after a miscarriage, bands of scar tissue,
or adhesions, may form inside the uterus. This is called Asherman syndrome

Pregnant at 5 weeks gestation with heavy bleeding and clots, she has a hx of 5 previous
abortions all her Previous abortions were at 2nd trimester, she had multiple D&C, what
causes her current bleeding?

A. Asherman syndrome
B. Cervical incompetence
C. Chromosomal abnormalities

Correct Answer: A
Explanation:
The risk of asherman syndrome increases with the increase of the D&C procedures the patient had.
This scenario goes more with asherman than chromosomal abnormalities because she had
MULTIPLE D&C

A woman has previous abortion, and she was managed by D&C, and now after 1 year she
is presenting with amenorrhea, what the diagnosis?

A. Asherman syndrome
B. Sheehan syndrome

Correct Answer: A
Explanation:
IUAs, or intrauterine synechiae

261 | P a g e
Obstetrics and Gynecology: SMLE Questions and Answers Chapter 2: Gynecology
- Is a condition in which scar tissue develops within the uterine cavity. IUAs that are
accompanied by symptoms (eg, infertility, amenorrhea) are referred to as Asherman
syndrome
- IUAs appear to result from trauma to the basalis layer of the endometrium

Patient with a history of multiple D&C for recurrent miscarriages , she can't get pregnant,
what is the endometrial layer removed?

A. Stratum compactum
B. Stratum spongiosum
C. Stratum functionalis
D. Stratum basalis

Correct Answer: D
Explanation:
IUAs, or intrauterine synechiae
- Is a condition in which scar tissue develops within the uterine cavity. IUAs that are
accompanied by symptoms (eg, infertility, amenorrhea) are referred to as Asherman
syndrome
- IUAs appear to result from trauma to the basalis layer of the endometrium

262 | P a g e
Obstetrics and Gynecology: SMLE Questions and Answers Chapter 2: Gynecology

Cervical Cancer and Screening

When should married women start to have pap smear screening?

A. 20-24
B. 25-29
C. 30-34
D. 35-39

Correct Answer: A
Explanation:
CERVICAL CANCER SCREENING GUIDELINES
The current guidelines in the United States recommend screening women for cervical cancer
between the ages of 21 and 65

When to start HPV SCREENING for cervical cancer?

A. 20-24
B. 25-29
C. 30-34
D. 35-39

Correct Answer: C
Explanation:
CERVICAL CANCER SCREENING GUIDELINES
The current guidelines in the United States recommend screening women for cervical cancer
between the ages of 21 and 65

263 | P a g e
Obstetrics and Gynecology: SMLE Questions and Answers Chapter 2: Gynecology

A 27-year-old, asymptomatic, her last pap smear was 3 years ago and it showed
unconcerned squamous cells. What is the most appropriate thing to do?

A. Repeat pap with cytology


B. No need and reassure
C. Colposcopy
D. Cervical swab

Correct Answer: A
Explanation:
CERVICAL CANCER SCREENING GUIDELINES
The current guidelines in the United States recommend screening women for cervical cancer
between the ages of 21 and 65

264 | P a g e
Obstetrics and Gynecology: SMLE Questions and Answers Chapter 2: Gynecology
A 20-Year-old girl divorced, came to clinic first visit, When to do pap smear?

A. Now
B. After 1 year at 21 years
C. After 5 years
D. No need

Correct Answer: B
Explanation:
- The current guidelines in the United States recommend screening women for cervical
cancer between the ages of 21 and 65 (Saslow, 2012).
- Cervical cancer screening should not be performed in women younger than 21 years of age,
regardless of age of onset of sexual activity

A 22-years-old, female married never did pap smear before when to do it?

A. Immediately
B. 3 years
C. 5 years
D. No need

Correct Answer: A
Explanation: According to Comprehensive Gynecology Textbook
Cervical cancer screening:
⁃ Cervical cancer screening should not be performed in women younger than 21 years of age,
regardless of age of onset of sexual activity
⁃ 21-29 à pap smear every 3 years
⁃ 30-65 à pap smear every 3 years Or pap smear + HPV testing every 5 years
⁃ >65 à no need for screening if the previous results were negative.

Newly married 22 years old comes to you for a regular follow-up for her gynecological
health. She is medically free and has regular 28 day cycle, complaining of mobile breast
lumps bilaterally that are painful right before meneses. Examination was normal. She
wants to get pregnant, but she is worried about her breast lump. What is the screening test
to be performed?

A. Bilateral mammogram
B. Papanicolaou test
C. Breast ultrasound

Correct Answer: B
Explanation: (According to UpToDate)
- Cyclical pain affects two-thirds of patients with true mastalgia. Cyclical pain is associated
with hormonal fluctuations of the menstrual cycle, usually presenting in the week prior to

265 | P a g e
Obstetrics and Gynecology: SMLE Questions and Answers Chapter 2: Gynecology
onset of menses. It is frequently bilateral and most severe in the upper outer quadrant of the
breasts.
- Women with cyclical or bilateral nonfocal breast pain usually do not require imaging

Note:
There’s no indication for mammogram or US in her case (because her examination is normal).
Pap smear screening is indicated in her case because she’s married.

Newly married young woman came for routine check up gyne, highest diagnostic value?

A. General appearance
B. Vaginal inspection
C. Abdominal exam
D. Pelvic digital exam

Correct Answer: D
Explanation: (According to ACOG and Williams Gynecology)
Well Woman Visit:
- A comprehensive history. This history includes symptoms; medications; allergies; and
medical, surgical, family, social, and gynecologic history, including questions on
reproductive, sexual, and mental health (using screening tools as indicated).
- Physical examination, including: breast and pelvic examination (inspection of external
genitalia, speculum, and bimanual palpation of the adnexa, uterus and bladder, and
sometimes rectovaginal examination) when indicated by medical history or symptoms.

Benefits of the pelvic examination include early detection of treatable gynecologic condition

A 31-year-old female, came for pap smear when to do it?

A. Every 5 years
B. Every 1 year
C. No need
D. If 3 is negative, no need to re-screen again

Correct Answer: A by exclusion


Explanation:
Answered by exclusion as B, C and D are incorrect

Correct answer is:


- Every 3 years pap smear alone
- Every 5 years Pap smear + HPV testing

266 | P a g e
Obstetrics and Gynecology: SMLE Questions and Answers Chapter 2: Gynecology
A 31-year-old female came for pap smear when to do it?

A. Every 2 years
B. Every 3 years
C. Every 5 years
D. If 3 is negative, no need to re-screen again

Correct Answer: B
Explanation:
CERVICAL CANCER SCREENING GUIDELINES
The current guidelines in the United States recommend screening women for cervical cancer
between the ages of 21 and 65

A 40-year-old female with history of wart 7 or 10 years back, last pap smear normal, what
is investigation you will do in this visit?

A. Repeat pap smear


B. Colposcopy

Correct Answer: A
Explanation:
CERVICAL CANCER SCREENING GUIDELINES
The current guidelines in the United States recommend screening women for cervical cancer
between the ages of 21 and 65

267 | P a g e
Obstetrics and Gynecology: SMLE Questions and Answers Chapter 2: Gynecology

Female patient did pap smear 7 years ago, result was atypical hyperplasia, she came now to
the clinic. What will you do?

A. Repeat pap smear


B. Colposcopy
C. CT

Correct Answer: A
Explanation:
CERVICAL CANCER SCREENING GUIDELINES
The current guidelines in the United States recommend screening women for cervical cancer
between the ages of 21 and 65

How to do pap smear?

268 | P a g e
Obstetrics and Gynecology: SMLE Questions and Answers Chapter 2: Gynecology
A. From transformation zone
B. From ectocervix
C. From endocervix

Correct Answer: A
Explanation: (According to Comprehensive Gynecology)
Cervical Cytology Testing (Pap smear)
- Cells are sampled from the transformation zone, which is the area of the cervix where
cervical cancer can develop.
The transformation zone includes the squamocolumnar junction, which is the area where
the squamous epithelium of the ectocervix meets the columnar epithelium of the
endocervix.

A 33-year-old female came to you at your office and her papsmear report was unsatisfactory
for evaluation, the best action is?

A. Consider it normal & D/C the pt.


B. Repeat it immediately
C. Repeat it as soon as possible
D. Repeat it after 6 months if considered low risk
E. Repeat it after 1 year if no risk

Correct Answer: C
Explanation:
Management of unsatisfactory cervical cytology: repeat pap smear after 2-4 months

A 24-year-old female did pap have abnormal results. What you will do?

A. Colposcopy
B. Repeat pap after 3 months
C. Reassurance

Correct Answer: A
Explanation:
Answered by exclusion.
Both B and C are incorrect

269 | P a g e
Obstetrics and Gynecology: SMLE Questions and Answers Chapter 2: Gynecology

A 58-Year-old female did Pap smear and showed (ASC-US), her treating physician prescribe
for her topical vaginal estrogen for 1 month, she came back after that and Pap smear repeated
and it was also (ASC-US ). What you will do for her?

A. Colposcopy
B. Punch biopsy
C. HPV testing
D. No further investigations

Correct Answer: A
Explanation:

A 34-years-old women did pap smear show ASCUS. What to do next?

A. Repeat pap.
B. HPV
C. Colposcopy
D. Conization

Correct Answer: B
Explanation:

A 40-Year-old female patient underwent PAP smear histopathology showed ASCUS, what is
your next step?
270 | P a g e
Obstetrics and Gynecology: SMLE Questions and Answers Chapter 2: Gynecology

A. Do HPV test
B. Colposcopy.
C. Re-evaluate after 6 months
D. Surgery

Correct Answer: A
Explanation:

A case of Atypical squamous cells, cannot exclude HSIL (ASC-H). What to do?

A. Conization
B. Colposcopy
C. HPV
D. Repeat pap

Correct Answer: B
Explanation:

Pap test came with high grade squamous intraepithelial lesion, what is the next step?

A. Colposcopy
B. Repeat pap test
C. Hysterectomy

Correct Answer: A
Explanation:

271 | P a g e
Obstetrics and Gynecology: SMLE Questions and Answers Chapter 2: Gynecology

Female heavy smoker (2packes/week) for the past 7 years. and since then show was doing Pap
smear and all negative, and she had previously infected with benign warts. this time it shows
LSIL, what will you do?

A. Colposcopy
B. HPV DNA
C. Pap smear

Correct Answer: Depends on the age of the patient


Explanation:
LSIL Management
- 21-24 à repeat pap smear after 1 year
- 25-29 à Colposcopy
- >30 à HPV test
o HPV positive à Colposcopy
o HPV negative à repeat pap smear and HPV after 1 year

A 30-year-old female did pap smear shows LSIL, What is the next step?

A. Coloposcoy
B. Repeat pap test after one year
C. HPV
D. Biopsy

Correct Answer: C
Explanation:
LSIL Management
- 21-24 à repeat pap smear after 1 year
- 25-29 à Colposcopy
- >30 à HPV test
o HPV positive à Colposcopy
o HPV negative à repeat pap smear and HPV after 1 year

272 | P a g e
Obstetrics and Gynecology: SMLE Questions and Answers Chapter 2: Gynecology

A 30-Year-old female came for pap screening, all her past results were negative, now results
show low grade squamous epithelial lesion. What's the appropriate next step?

A. Humen papilloma virus


B. Colposcopy
C. Evaluate after 6 months.
D. Surgery

Correct Answer: A
Explanation:
LSIL Management
- 21-24 à repeat pap smear after 1 year
- 25-29 à Colposcopy
- >30 à HPV test
o HPV positive à Colposcopy
o HPV negative à repeat pap smear and HPV after 1 year

A 30-year-old female her pap smear, result showed squamous cell ca (SCC), what to do next?

A. Colposcopy directed biopsy


B. Repeat pap smear
C. Total hysterectomy
D. Neoadjuvant chemotherapy

Correct Answer: A
Explanation:
Pap smear is only a screening test, it’s not diagnostic.
We should do biopsy first after the pap smear, and manage according to the biopsy result

Pregnant at 12 weeks with suspicious cervical lesion, how to confirm the diagnosis or what
is next?

A. Cone biopsy
B. Colposcopy directed biopsy
C. Pap smear
D. Endocervical curettage

Correct Answer: B
Explanation:
According to Berek and Hacker’s Gynecologic Oncology and Uptodate:
- Any visible lesion, tumor growth, or ulceration in the cervix should undergo office punch
biopsy or loop excision for histologic confirmation (regardless of previous benign cervical
cytology results).

273 | P a g e
Obstetrics and Gynecology: SMLE Questions and Answers Chapter 2: Gynecology
- Any cervix that is unusually firm or expanded should also undergo punch biopsy and
endocervical curettage (ECC) (even if the cervical cytology test does not show evidence of
neoplasia)

According to Comprehensive Gynecology Textbook:


- Colposcopy is safe in pregnancy.
- Endocervical curettage (ECC) should never be performed during pregnancy
- If invasive cancer is diagnosed during pregnancy, a conization procedure under anesthesia
can be performed.

A 34-Year-old female at 30 weeks of gestation with painless vaginal bleeding, did vaginal
examination found suspicious mass on cervix.

US Report: the fetus corresponds to the Gestational age.

What is the most appropriate next step?

A. Colposcopy
B. Cone biopsy
C. Pap smear
D. Endocervical curettage

Correct Answer: A
Explanation:
According to Berek and Hacker’s Gynecologic Oncology and Uptodate:
- Any visible lesion, tumor growth, or ulceration in the cervix should undergo office punch
biopsy or loop excision for histologic confirmation (regardless of previous benign cervical
cytology results).
- Any cervix that is unusually firm or expanded should also undergo punch biopsy and
endocervical curettage (ECC) (even if the cervical cytology test does not show evidence of
neoplasia)

According to Comprehensive Gynecology Textbook:


- Colposcopy is safe in pregnancy.
- Endocervical curettage (ECC) should never be performed during pregnancy
- If invasive cancer is diagnosed during pregnancy, a conization procedure under anesthesia
can be performed.

During a cervical examination of a patient, you found her to have lesion, what to do?

A. Pap test
B. RNA test

274 | P a g e
Obstetrics and Gynecology: SMLE Questions and Answers Chapter 2: Gynecology
C. Cone biopsy
D. Colposcopy directed biopsy

Correct Answer: D
Explanation:
According to Berek and Hacker’s Gynecologic Oncology and Uptodate:
- Any visible lesion, tumor growth, or ulceration in the cervix should undergo office punch
biopsy or loop excision for histologic confirmation (regardless of previous benign cervical
cytology results).
- Any cervix that is unusually firm or expanded should also undergo punch biopsy and
endocervical curettage (ECC) (even if the cervical cytology test does not show evidence of
neoplasia)

Female with cervical lesion measuring 11mmx12mm with irregular borders, pap smear test
was done, no results yet, what to do?

A. Excise the lesion


B. Biopsy
C. Reassure until pap results are available
D. HPV test

Correct Answer: B
Explanation:
According to Berek and Hacker’s Gynecologic Oncology and Uptodate:
- Any visible lesion, tumor growth, or ulceration in the cervix should undergo office punch
biopsy or loop excision for histologic confirmation (regardless of previous benign cervical
cytology results).
- Any cervix that is unusually firm or expanded should also undergo punch biopsy and
endocervical curettage (ECC) (even if the cervical cytology test does not show evidence of
neoplasia)

A woman presented with dyspareunia but denies any heaviness or urinary symptoms,
found to have a cervical mass on examination, what is your management?

A. Discharge
B. Follow up
C. Excision at the clinic
D. Pap smear

Correct Answer: C
Explanation:
According to Berek and Hacker’s Gynecologic Oncology and Uptodate:

275 | P a g e
Obstetrics and Gynecology: SMLE Questions and Answers Chapter 2: Gynecology
- Any visible lesion, tumor growth, or ulceration in the cervix should undergo office punch
biopsy or loop excision for histologic confirmation (regardless of previous benign cervical
cytology results).
- Any cervix that is unusually firm or expanded should also undergo punch biopsy and
endocervical curettage (ECC) (even if the cervical cytology test does not show evidence of
neoplasia)

Fungating mass from cervix, what is the highest diagnostic value?

A. PAP smear
B. Colposcopy
C. Biopsy
D. MRI pelvis

Correct Answer: B
Explanation:
According to Berek and Hacker’s Gynecologic Oncology and Uptodate:
- Any visible lesion, tumor growth, or ulceration in the cervix should undergo office punch
biopsy or loop excision for histologic confirmation (regardless of previous benign cervical
cytology results).
- Any cervix that is unusually firm or expanded should also undergo punch biopsy and
endocervical curettage (ECC) (even if the cervical cytology test does not show evidence of
neoplasia)

Young patient did pap smear in screening and showed HISL, the biopsy showed carcinoma
insitue, she wants to conserve her fertility. What is the most appropriate management?

A. Cold knife (conization)


B. Electrosurgical loop (LEEP)
C. Cryotherapy
D. Laser

Correct Answer: B
Explanation: (According to Comprehensive Gynecology)
CERVICAL INTRAEPITHELIAL NEOPLASIA (CIN)
- Management
• CIN 1, or low grade dysplasia
In almost all cases, CIN 1 is a manifestation of a transient HPV infection, and the
regression rates are high. Patients with CIN 1 require follow-up to ensure that the
lesion regresses

• CIN 2/3, or high grade dysplasia


o Most women should be treated with an ablative or excisional procedure.

276 | P a g e
Obstetrics and Gynecology: SMLE Questions and Answers Chapter 2: Gynecology
o Pregnant women with CIN 2/3 and no evidence of invasion may be observed
during the pregnancy, with evaluation delayed until 6 weeks postpartum.
o When treatment is performed, we prefer excision over ablation for all CIN
grades because it provides a diagnostic specimen

§ Ablative methods
• Treat CIN but do not provide further diagnostic information.
• Cryotherapy
Þ Is a commonly used treatment for CIN lesions that is
safe, effective. However, it does not provide a
specimen for pathology review and has been replaced
by LEEP
• Co2 Laser ablation
Þ Has largely been replaced by LEEP.

§ Excisional procedures
• Have the advantage over ablative procedures of providing a
pathologic specimen for further diagnostic information.
• Loop electrosurgical excision procedure (LEEP)
Þ Currently the most common method for the treatment
of CIN 2/3 in the United States. Can be performed
safely in the office.
Þ The removed tissue is examined histologically for
diagnosis and evaluation of margin status.
• Cold knife conization (CKC)
Þ Offers little advantage over LEEP. However, CKC is
advantageous in patients with glandular abnormalities
or suspicion of invasive cancer
• Co2 Laser conization

A 38-years-old female with non invasive carcinoma in cervix of young woman wishing to
preserve fertility, what is the best treatment?

A. Endometrial ablation
B. Hysterectomy
C. Cold knife conization
D. Electrosurgical loop (LEEP)

Correct Answer: D
Explanation: (According to Comprehensive Gynecology)
Non invasive Carcinoma of the cervix = Carcinoma in situ = CIN3
CERVICAL INTRAEPITHELIAL NEOPLASIA (CIN)
- Management

277 | P a g e
Obstetrics and Gynecology: SMLE Questions and Answers Chapter 2: Gynecology
• CIN 1, or low grade dysplasia
In almost all cases, CIN 1 is a manifestation of a transient HPV infection, and the
regression rates are high. Patients with CIN 1 require follow-up to ensure that the
lesion regresses

• CIN 2/3, or high grade dysplasia


o Most women should be treated with an ablative or excisional procedure.
o Pregnant women with CIN 2/3 and no evidence of invasion may be observed
during the pregnancy, with evaluation delayed until 6 weeks postpartum.
o When treatment is performed, we prefer excision over ablation for all CIN
grades because it provides a diagnostic specimen

§ Ablative methods
• Treat CIN but do not provide further diagnostic information.
• Cryotherapy
Þ Is a commonly used treatment for CIN lesions that is
safe, effective. However, it does not provide a
specimen for pathology review and has been replaced
by LEEP
• Co2 Laser ablation
Þ Has largely been replaced by LEEP.

§ Excisional procedures
• Have the advantage over ablative procedures of providing a
pathologic specimen for further diagnostic information.
• Loop electrosurgical excision procedure (LEEP)
Þ Currently the most common method for the treatment
of CIN 2/3 in the United States. Can be performed
safely in the office.
Þ The removed tissue is examined histologically for
diagnosis and evaluation of margin status.
• Cold knife conization (CKC)
Þ Offers little advantage over LEEP. However, CKC is
advantageous in patients with glandular abnormalities
or suspicion of invasive cancer
• Co2 Laser conization

40 years old with post coital bleeding and intermenstrual bleeding She had 3 pap smears
positive and did a colposcopy showed intraepithelial carcinoma ,What is the next step ?

A. MRI abdominal
B. Cone biopsy
C. LEEP
D. CT abdomen chest pelvis

278 | P a g e
Obstetrics and Gynecology: SMLE Questions and Answers Chapter 2: Gynecology
Correct Answer: C
Explanation: (According to Comprehensive Gynecology)
This is a case of CIN3
Non invasive Carcinoma of the cervix = Carcinoma in situ = CIN3
CERVICAL INTRAEPITHELIAL NEOPLASIA (CIN)
- Management
• CIN 1, or low grade dysplasia
In almost all cases, CIN 1 is a manifestation of a transient HPV infection, and the
regression rates are high. Patients with CIN 1 require follow-up to ensure that the
lesion regresses

• CIN 2/3, or high grade dysplasia


o Most women should be treated with an ablative or excisional procedure.
o Pregnant women with CIN 2/3 and no evidence of invasion may be observed
during the pregnancy, with evaluation delayed until 6 weeks postpartum.
o When treatment is performed, we prefer excision over ablation for all CIN
grades because it provides a diagnostic specimen

§ Ablative methods
• Treat CIN but do not provide further diagnostic information.
• Cryotherapy
Þ Is a commonly used treatment for CIN lesions that is
safe, effective. However, it does not provide a
specimen for pathology review and has been replaced
by LEEP
• Co2 Laser ablation
Þ Has largely been replaced by LEEP.

§ Excisional procedures
• Have the advantage over ablative procedures of providing a
pathologic specimen for further diagnostic information.
• Loop electrosurgical excision procedure (LEEP)
Þ Currently the most common method for the treatment
of CIN 2/3 in the United States. Can be performed
safely in the office.
Þ The removed tissue is examined histologically for
diagnosis and evaluation of margin status.
• Cold knife conization (CKC)
Þ Offers little advantage over LEEP. However, CKC is
advantageous in patients with glandular abnormalities
or suspicion of invasive cancer
• Co2 Laser conization

279 | P a g e
Obstetrics and Gynecology: SMLE Questions and Answers Chapter 2: Gynecology
Females with abnormal Pap smear and colposcopy diagnosed with invasive cervical cancer,
what is the most appropriate next step?

A. Clinical staging
B. Hysterectomy and chemotherapy
C. Hysterectomy and radiotherapy

Correct Answer: A
Explanation: (According to Berek and Hacker’s Gynecologic Oncology)
Cervical cancer has always been staged clinically

18 or 25 Year-old female, worraied about cervical cancer, she took her first dose of HPV
vaccine 3 months ago. What is the best thing to do at this visit today?

A. Schedule appointment after 3 months


B. No need to do anything at this visit
C. Give 2nd dose at this visit
D. Repeat 1st dose

Correct Answer: C
Explanation: (According to Centers for Disease Control and Prevention (CDC))
HPV Vaccine Schedule and Dosing
**Who Gets 2 Doses? 9-14 Years old

• A 2-dose schedule is recommended for people who get the first dose before their
15th birthday.
• In a 2-dose series, the second dose should be given 6–12 months after the first dose (0, 6–
12 month schedule).
• The minimum interval is:
- 5 months between the first and second dose.
- If the second dose is administered after a shorter interval, a third dose should be
administered a minimum of 5 months after the first dose and a minimum of 12 weeks
after the second dose.
• If the vaccination schedule is interrupted, vaccine doses do not need to be repeated (no
maximum interval).
• Immunogenicity studies have shown that 2 doses of HPV vaccine given to 9–14 year-olds
at least 6 months apart provided as good or better protection than 3 doses given to older
adolescents or young adults.

**Who Gets 3 Doses? 15-26 Years old

• A 3-dose schedule is recommended for people who get the first dose on or after their
15th birthday, and for people with certain immunocompromising conditions.
• In a 3-dose series, the second dose should be given 1–2 months after the first dose, and
the third dose should be given 6 months after the first dose (0, 1–2, 6 month schedule).

280 | P a g e
Obstetrics and Gynecology: SMLE Questions and Answers Chapter 2: Gynecology
• The minimum intervals are:
- 4 weeks between the first and second dose.
- 12 weeks between the second and third doses, and 5 months between the first and third
doses.
- If a vaccine dose is administered after a shorter interval, it should be re-administered
after another minimum interval has elapsed since the most recent dose.
• If the vaccination schedule is interrupted, vaccine doses do not need to be repeated (no
maximum interval).

281 | P a g e
Obstetrics and Gynecology: SMLE Questions and Answers Chapter 2: Gynecology

Endometrial Polyp, Hyperplasia and Carcinoma

A 47-Year-Old Female complaining of AUB for 6 months. She wants to take OCP. What is
the most important test to do before treatment?

A. FSH
B. LH
C. Pelvic US
D. Endometrial biopsy

Correct Answer: D
Explanation: (According to ACOG)
American College of Obstetricians and Gynccologists (2016a) recommends endomettial
assessment in any woman older than 45 years with AUB.

A 60-Year-Old lady present with lower genital bleeding, she described it as scanty and
barely stain the pad, what is the source of bleeding?

A. Fallopian tube
B. Ovary
C. Uterus
D. Genital tract

Correct Answer: C
Explanation: (According to ACOG)
Vaginal bleeding is the presenting sign in more than 90% of postmenopausal women with
endometrial carcinoma

Note:
Postmenopausal bleeding is endometrial cancer until proven otherwise.

Women presents with postcoital bleeding, her cycles are becoming heavier and irregular,
she has type 2 diabetes and BMI 38. US showed endometrial thickness 18mm. What is the
most appropriate next step in her management?

A. Pelvic CT
B. Hysteroscopy
C. Clinical endometrial sample

Correct Answer: C
Explanation: (According to Uptodate)

282 | P a g e
Obstetrics and Gynecology: SMLE Questions and Answers Chapter 2: Gynecology
2,
Any abnormal uterine bleeding in patients with BMI ≥30 kg/m should undergo evaluation for
endometrial hyperplasia or endometrial carcinoma via endometrial sampling

A 64-Year-old lady with vaginal spotting, what is the most appropriate next step in
management?

A. Endometrial sample
B. Follow up
C. Hysterectomy

Correct Answer: A
Explanation: (According to Uptodate)
Postmenopausal patients with any uterine bleeding, regardless of volume (including spotting or
staining), should undergo evaluation for endometrial hyperplasia or carcinoma via endometrial
sampling, pelvic ultrasound is an alternative.

A 59-Year-old lady, presenting with abnormal uterine bleeding, upon examination, uterus
normal size with no adnexal masses, US showed endometrial thickness of 15mm, what is
the most appropriate next step?

A. Pelvic MRI
B. Endometrial biopsy
C. Exploratory laparotomy

Correct Answer: B
Explanation: (According to Uptodate)
Postmenopausal patients with any uterine bleeding, regardless of volume (including spotting or
staining), should undergo evaluation for endometrial hyperplasia or carcinoma via endometrial
sampling, pelvic ultrasound is an alternative

A post-menopausal patient, known to have DM, hypertensin and morbid obesity,


presenting with abnormal uterine bleeding, what is the most appropriate test to reach
diagnosis?

A. Pelvic US
B. Pelvic MRI
C. Pelvic CT
D. Endometrial biopsy

Correct Answer: D

283 | P a g e
Obstetrics and Gynecology: SMLE Questions and Answers Chapter 2: Gynecology
Explanation: (According to Uptodate)
Postmenopausal patients with any uterine bleeding, regardless of volume (including spotting or
staining), should undergo evaluation for endometrial hyperplasia or carcinoma via endometrial
sampling, pelvic ultrasound is an alternative

A 47-Year-Old female, complaining of polymenorrhagia with BMI 37, What is the most
important step in management?

A. Pelvic MRI
B. US
C. Endometrial biopsy

Correct Answer: C
Explanation: (According to ACOG and Uptodate)
• Any abnormal uterine bleeding in patients with BMI ≥30 kg/m2, should undergo evaluation
for endometrial hyperplasia or endometrial carcinoma via endometrial sampling
• American College of Obstetricians and Gynccologists (2016a) recommends endomettial
assessment in any woman older than 45 years with AUB.

A 55-Year-old lady presented with intermittent vaginal bleeding, no history of weight loss,
unremarkable history, what is the most likely diagnosis?

A. Endometrial cancer
B. Endometrial hyperplasia
C. Endometrial atrophy

Correct Answer: C
Explanation: (According to Uptodate)
In postmenopausal patients with uterine bleeding, the frequency of endometrial pathology
was as follows:
- Polyp (37.7 percent)
- Hypotrophy/atrophy (30.8 percent)
- Proliferative/secretory (14.5 percent)
- Carcinoma (6.6 percent)
- Fibroid (6.2 percent)
- Hyperplasia without atypia (2 percent)
- Hyperplasia with atypia (0.2 percent)

Obese patient with irregular menstrual cycle. Endometrial biopsy showed hyperplasia,
what is the pathophysiology?

A. Adrenal hyperplasia
B. Peripheral conversion of precursors to estrogen

284 | P a g e
Obstetrics and Gynecology: SMLE Questions and Answers Chapter 2: Gynecology
C. Genetic mutation of endometrium
D. Hepatic dysfunction

Correct Answer: B
Explanation: (According to Comprehensive Gynecology)
Women who are obese (body mass index >30) have a two- to threefold increased risk, and this risk
increased with increasing weight. The association is believed to be due in part to increased
circulating estrogen levels that result from conversion of androstenedione to estrone in the adipose
tissue, decreased sex hormone-binding globulin

Endometrial hyperplasia with atypia unfit for surgery what’s your management?

A. Chemotherapy
B. Radiotherapy
C. Continued progesterone

Correct Answer: C
Explanation:
Endometrial hyperplasia
- Management

• Endometrial hyperplasia with atypia


o Preferred: Hysterectomy
- Candidates:
§ For most postmenopausal patients and premenopausal patients who
have completed childbearing
§
o Alternative: Progestin therapy
- Candidates:
§ Premenopausal patients who desire future fertility
§ Patients of any reproductive status who decline hysterectomy
§ Patients at high risk of surgical complications

A 32-Year-Old lady, obese, complaining of irregular mensuration and chronic anovulation,


endometrial sample showed: atypical complex hyperplasia. What’s the most definitive
treatment?

A. Tamoxifen
B. Letrozole
C. Progesterone
D. Spironolactone

Correct Answer: C
Explanation: (According to Uptodate, Williams Gynecology)

285 | P a g e
Obstetrics and Gynecology: SMLE Questions and Answers Chapter 2: Gynecology
Endometrial hyperplasia

• Endometrial hyperplasia with atypia


o Preferred: Hysterectomy
- Candidates:
§ For most postmenopausal patients and premenopausal patients who
have completed childbearing
§
o Alternative: Progestin therapy
- Candidates:
§ Premenopausal patients who desire future fertility
§ Patients of any reproductive status who decline hysterectomy
§ Patients at high risk of surgical complications

Highest risk factor for endometrial cancer?

A. Age
B. PCOS
C. HTN
D. Multiparity

Correct Answer: B
Explanation: (According to William Gynecology, ACOG, Comprehensive Gynecology)
Endometrial hyperplasia and carcinoma
- Risk factors:
• Increase the Risk
o Older age (1-2x)
o Unopposed estrogen stimulation (10x)
o Polycystic ovary syndrome (3x)
o Unopposed menopausal estrogen (4-8×) replacement therapy
o Menopause after 52 years (2.4×)
o Early age at menarche
o Obesity (2-5×)
o Nulliparity (2-3×)
o Infertility
o Menstrual irregularities
o Diabetes (2.8×)
o Hypertension or gallbladder disease
o Insulin resistance
o Estrogen secreting ovarian tumors
o Tamoxifen therapy for breast cancer
o Lynch syndrome
o Higher level of education or income

286 | P a g e
Obstetrics and Gynecology: SMLE Questions and Answers Chapter 2: Gynecology
o North America or northern Europe residence

• Diminishes the Risk


o Ovulation
o Progestin therapy
o Combination oral contraceptives
o Menopause before 49 years
o Multiparity
o Cigarette smoking

Which of the following associated with increase risk of endometrial cancer?

A. Nulliparity
B. Underweight
C. Oral progesterone
D. Family history of thyroid cancer

Correct Answer: A
Explanation: (According to William Gynecology, ACOG, Comprehensive Gynecology)
Endometrial hyperplasia and carcinoma
- Risk factors:
• Increase the Risk
o Older age (1-2x)
o Unopposed estrogen stimulation (10x)
o Polycystic ovary syndrome (3x)
o Unopposed menopausal estrogen (4-8×) replacement therapy
o Menopause after 52 years (2.4×)
o Early age at menarche
o Obesity (2-5×)
o Nulliparity (2-3×)
o Infertility
o Menstrual irregularities
o Diabetes (2.8×)
o Hypertension or gallbladder disease
o Insulin resistance
o Estrogen secreting ovarian tumors
o Tamoxifen therapy for breast cancer
o Lynch syndrome
o Higher level of education or income
o North America or northern Europe residence

• Diminishes the Risk


o Ovulation
o Progestin therapy

287 | P a g e
Obstetrics and Gynecology: SMLE Questions and Answers Chapter 2: Gynecology
o Combination oral contraceptives
o Menopause before 49 years
o Multiparity
o Cigarette smoking

Patient, P4, known case of PCOS and a smoker, which of the following increases her
chance of endometrial cancer?

A. PCOS
B. Age
C. Smoking
D. Parity

Correct Answer: A
Explanation: (According to William Gynecology, ACOG, Comprehensive Gynecology)
Endometrial hyperplasia and carcinoma
- Risk factors:
• Increase the Risk
o Older age (1-2x)
o Unopposed estrogen stimulation (10x)
o Polycystic ovary syndrome (3x)
o Unopposed menopausal estrogen (4-8×) replacement therapy
o Menopause after 52 years (2.4×)
o Early age at menarche
o Obesity (2-5×)
o Nulliparity (2-3×)
o Infertility
o Menstrual irregularities
o Diabetes (2.8×)
o Hypertension or gallbladder disease
o Insulin resistance
o Estrogen secreting ovarian tumors
o Tamoxifen therapy for breast cancer
o Lynch syndrome
o Higher level of education or income
o North America or northern Europe residence

• Diminishes the Risk


o Ovulation
o Progestin therapy
o Combination oral contraceptives
o Menopause before 49 years
o Multiparity
o Cigarette smoking

288 | P a g e
Obstetrics and Gynecology: SMLE Questions and Answers Chapter 2: Gynecology
Highest risk factor for endometrial cancer?

A. Late menarche
B. Early menopause
C. DM
D. Progesterone secreting tumor

Correct Answer: C
Explanation: (According to William Gynecology, ACOG, Comprehensive Gynecology)
Endometrial hyperplasia and carcinoma
- Risk factors:
• Increase the Risk
o Older age (1-2x)
o Unopposed estrogen stimulation (10x)
o Polycystic ovary syndrome (3x)
o Unopposed menopausal estrogen (4-8×) replacement therapy
o Menopause after 52 years (2.4×)
o Early age at menarche
o Obesity (2-5×)
o Nulliparity (2-3×)
o Infertility
o Menstrual irregularities
o Diabetes (2.8×)
o Hypertension or gallbladder disease
o Insulin resistance
o Estrogen secreting ovarian tumors
o Tamoxifen therapy for breast cancer
o Lynch syndrome
o Higher level of education or income
o North America or northern Europe residence

• Diminishes the Risk


o Ovulation
o Progestin therapy
o Combination oral contraceptives
o Menopause before 49 years
o Multiparity
o Cigarette smoking

Patient on Tamoxifen, leads to what?

A. Endometrial cancer
B. Breast cancer

Correct Answer: A

289 | P a g e
Obstetrics and Gynecology: SMLE Questions and Answers Chapter 2: Gynecology
Explanation: (According to William Gynecology, ACOG, Comprehensive Gynecology)
Endometrial hyperplasia and carcinoma
- Risk factors:
• Increase the Risk
o Older age (1-2x)
o Unopposed estrogen stimulation (10x)
o Polycystic ovary syndrome (3x)
o Unopposed menopausal estrogen (4-8×) replacement therapy
o Menopause after 52 years (2.4×)
o Early age at menarche
o Obesity (2-5×)
o Nulliparity (2-3×)
o Infertility
o Menstrual irregularities
o Diabetes (2.8×)
o Hypertension or gallbladder disease
o Insulin resistance
o Estrogen secreting ovarian tumors
o Tamoxifen therapy for breast cancer
o Lynch syndrome
o Higher level of education or income
o North America or northern Europe residence

• Diminishes the Risk


o Ovulation
o Progestin therapy
o Combination oral contraceptives
o Menopause before 49 years
o Multiparity
o Cigarette smoking

Which of the following occur with OCP?

A. Decrease the risk of ovarian cancer


B. Increase the risk of breast cancer
C. Decrease the risk of endometrial cancer.
D. Increase risk of ectopic pregnancy

Correct Answer: C
Explanation: (According to William Gynecology, ACOG, Comprehensive Gynecology)
Endometrial hyperplasia and carcinoma
- Risk factors:
• Increase the Risk

290 | P a g e
Obstetrics and Gynecology: SMLE Questions and Answers Chapter 2: Gynecology
o Older age (1-2x)
o Unopposed estrogen stimulation (10x)
o Polycystic ovary syndrome (3x)
o Unopposed menopausal estrogen (4-8×) replacement therapy
o Menopause after 52 years (2.4×)
o Early age at menarche
o Obesity (2-5×)
o Nulliparity (2-3×)
o Infertility
o Menstrual irregularities
o Diabetes (2.8×)
o Hypertension or gallbladder disease
o Insulin resistance
o Estrogen secreting ovarian tumors
o Tamoxifen therapy for breast cancer
o Lynch syndrome
o Higher level of education or income
o North America or northern Europe residence

• Diminishes the Risk


o Ovulation
o Progestin therapy
o Combination oral contraceptives
o Menopause before 49 years
o Multiparity
o Cigarette smoking

A 55-Year-Old female, presented with abnormal uterine bleeding, she had history of
myomectomy, x2 D&C, upon examination abdomen bulky with no tenderness, no adnexal
masses. US showed endometrial thickness of 20mm, what is the most likely diagnosis?

A. Endometrial cancer
B. Leiomyoma
C. Adenomyosis
D. Endometriosis

Correct Answer: A
Explanation: (According to Williams Gynecology)
Postmenopausal bleeding is particularly worrisome and carries a 5- to 10-percent likelihood of
diagnosing endometrial carcinoma

291 | P a g e
Obstetrics and Gynecology: SMLE Questions and Answers Chapter 2: Gynecology
A 60-Year-Old female, last menses 12 years back. Presented with abnormal uterine
bleeding, suspected to be due to intrauterine polyps. US thickness 13mm. what is the most
appropriate management?

A. Endometrial ablation
B. Laparoscopic hysterectomy
C. Vaginal hysterectomy
D. Polypectomy

Correct Answer: B
Explanation: (According to Uptodate and Williams gynecology)
Endometrial carcinoma
- Staging
• Surgically
o Procedure:
§ Total extrafascial hysterectomy with bilateral salpingo-
oophorectomy
§ Cytoreduction often is performed when metastases are evident.
o Route:
§ Robotic-assisted laparoscopy or conventional laparoscopy
§ Previously, laparotomy had been the standard approach
o Alternative route:
§ Vaginal hysterectomy with or without BSO is another option for
women who cannot undergo systematic surgical staging due to
comorbidities.
o Lymph node:
§ Options including pelvic-aortic lymph node dissection and sentinel
lymph node biopsy,

Old lady with uterine fundal mass. Diagnosed with endometrial cancer, underwent surgery.
What lymph nodes to resect?

A. External iliac lymph node


B. Internal iliac lymph node
C. Deep inguinal lymph node
D. Para-aortic lymph node

Correct Answer: D
Explanation: (According to Uptodate and Williams gynecology)
Endometrial carcinoma
- Staging
• Surgically
o Procedure:
§ Total extrafascial hysterectomy with bilateral salpingo-
oophorectomy
§ Cytoreduction often is performed when metastases are evident.
292 | P a g e
Obstetrics and Gynecology: SMLE Questions and Answers Chapter 2: Gynecology
o Route:
§ Robotic-assisted laparoscopy or conventional laparoscopy
§ Previously, laparotomy had been the standard approach
o Alternative route:
§ Vaginal hysterectomy with or without BSO is another option for
women who cannot undergo systematic surgical staging due to
comorbidities.
o Lymph node:
§ Options including pelvic-aortic lymph node dissection and sentinel
lymph node biopsy,

293 | P a g e
Obstetrics and Gynecology: SMLE Questions and Answers Chapter 2: Gynecology

Patient with a history of minimal vaginal bleeding, upon speculum examination (as seen in
the picture), What is the most appropriate management?

A. Excision in the clinic


B. Follow up after few months
C. Admission and more investigations

Correct Answer: A
Explanation: (According to Uptodate)
Cervical Polyps
- Should be removed if they are symptomatic (eg, bleeding, excessive discharge), large (≥3
cm), or appear atypical.
- Polypectomy can usually be accomplished by grasping the base of the polyp with forceps
and twisting it off. If visible, the base can be cauterized to prevent bleeding and reduce the
chance of recurrence.

Patient with abnormal uterine bleeding, hysteroscopy was done (as seen in the picture,
what is the most likely diagnosis?

A. Subserous leiomyoma
B. Endometrial polyp

294 | P a g e
Obstetrics and Gynecology: SMLE Questions and Answers Chapter 2: Gynecology
Correct Answer: B
Explanation:

A 43-Year-old, multiparous, complaining of inter-menstrual bleed, US showed 13 mm


echongenic structure projecting from the endometrium, most appropriate management?

A. Endometrial sample
B. Hysteroscopy with removal of structure
C. US after 6 months
D. Serial progestin in 3 months

Correct Answer: B
Explanation: (According to Uptodate)
Endometrial Polyp
- Management
• Asymptomatic
o Managed expectantly.

• Patients with risk factors for cancer, large or multiple polyps, or infertility
o Removal of the endometrial polyp via hysteroscopic polypectomy

• Patient with abnormal uterine bleeding (AUB)


o Removal of the endometrial polyp via hysteroscopic polypectomy

A 57-Year-Old female complaining of abnormal uterine bleeding she has an endometrial


polyp, on US endometrial lining was 19mm, what will you offer to this patient at this stage?

A. Open hysterectomy
B. Laparoscopic hysterectomy
C. Hysteroscopy polypectomy

Correct Answer: C
Explanation: (According to Uptodate)
Endometrial Polyp
- Management
• Asymptomatic
o Managed expectantly.

• Patients with risk factors for cancer, large or multiple polyps, or infertility
o Removal of the endometrial polyp via hysteroscopic polypectomy

• Patient with abnormal uterine bleeding (AUB)


o Removal of the endometrial polyp via hysteroscopic polypectomy
295 | P a g e
Obstetrics and Gynecology: SMLE Questions and Answers Chapter 2: Gynecology

296 | P a g e
Obstetrics and Gynecology: SMLE Questions and Answers Chapter 2: Gynecology

Urogynecology

A 45-year-old lady, complaining of urinary leakage especially when running. What test to
do?

A. Q tip test
B. Bonney’s test
C. Cough stress test
D. Uroflowmetry
E. Cystoscopy

Correct Answer: C
Explanation:
According to UROGYNECOLOGY WALTERS&KARRAM
- Q-Tip Test is no longer considered useful in helping with diagnosis or treatment of
incontinence.

According to UpToDate
- Bladder stress test – In patients with suspected SUI, we perform the bladder stress test to
confirm the diagnosis

A 50-year-old female, medically and surgically free, she presented with complaints of urine
incontinence with coughing and sneezing, most appropriate management?

A. Pelvic floor exercise


B. Urethral sling
C. Colporrhaphy
D. Burch procedure

Correct Answer: B
Explanation: (According to UROGYNECOLOGY WALTERS&KARRAM)
Urinary incontinence
- Midurethral slings: Gold standard for SUI
Þ Tension-free Vaginal Tape (TVT) - most commonly used
Þ Transobturator Tape (TOT)

A 50-year-old female diagnosed with stress incontinence, what is the definitive


management?

A. Passery

297 | P a g e
Obstetrics and Gynecology: SMLE Questions and Answers Chapter 2: Gynecology
B. Tension free Vaginal Tape
C. Kelly plicatiom
D. Burch procedure

Correct Answer: B
Explanation: (According to UROGYNECOLOGY WALTERS&KARRAM)
Urinary incontinence
- Midurethral slings: Gold standard for SUI
Þ Tension-free Vaginal Tape (TVT) - most commonly used
Þ Transobturator Tape (TOT)

A 60-year-old this is her 2nd UTI in the last 6 months, feels a bulge in the vagina and has
difficulty emptying her bladder, what is the most appropriate next step?

A. Uroflowmetry
B. Urodynamic test
C. Video Urodynamic
D. Post voiding Residual volume

Correct Answer: D
Explanation: (According to UROGYNECOLOGY WALTERS&KARRAM)
- A postvoid residual (PVR) urine volume is one method used to evaluate for urinary
retention in patients with Pelvic organ prolapse.

- Options A, B and C are the same.

A patient G5P5 with urine leak when she coughs or sneeze she denies any urgency
frequency or dysuria. What you suspect to find during examination?

A. Paravaginal defect
B. Rectocele
C. Enterocele
D. Hypermobile urethra

Correct Answer: D
Explanation: (According to UpToDate)
Mechanisms of SUI include:
- Urethral hypermobility – may be related to loss of connective tissue and/or muscular
strength due to chronic pressure (ie, high-impact activity, chronic cough, or obesity) or
trauma due to childbirth, particularly vaginal deliveries.

298 | P a g e
Obstetrics and Gynecology: SMLE Questions and Answers Chapter 2: Gynecology
- Intrinsic sphincteric deficiency

A female patient presented complaining of a sudden urge to void, frequent voidings during
the day and getting up to void more than once per night. She was diagnosed as urge
urinary incontinence. Which of the following the most appropriate management?

A. Kegel exercises
B. Vaginal cones
C. Oxybutynin
D. Oxybutynin and Kegel exercises

Correct Answer: D
Explanation: (According to UROGYNECOLOGY WALTERS&KARRAM and UpToDate)
TREATMENT OF URGANCY URINARY INCONTINENCE
- Initial treatment: Trial of pelvic floor exercises, lifestyle therapy, and behavioral changes
- Addition of medication — We offer adjunctive medication if initial treatments do not
provide sufficient symptoms relief. There are six antimuscarinic medications; darifenacin,
fesoterodine, oxybutynin, solifenacin(Vesicare), tolterodine, and trospium.

A 54-year-old female presented to gynecology clinic complaining of dysuria and urinary


incontinence, she stated that the leak is not related to specific activities and occasionally
associated with cough.

Upon investigation urinalysis and cultures were insignificant, However, urodynamic study
showed: contracting bladder even with small amount of dripping

Which of the following is the most appropriate management?

A. Kegel exercise
B. Anterior Colporrhaphy
C. Anticholinergic agent
D. Bladder Suspension

Correct Answer: C
Explanation:
- According to the urodynamic study, the patient has Urgency urinary incontinence.
Therefore the definitive treatment is Anticholinergic medications.

A 56-year-old multiparous patient on oral estrogen, complains of painless urine


incontinence when coughing, sneezing or laughing for 1 year. On examination, there was
mild vaginal atrophy and constrictor muscle laxity. Urinary incontinence is demonstrated
in the lithotomy position with Valsalva maneuver .Urinalysis: normal

299 | P a g e
Obstetrics and Gynecology: SMLE Questions and Answers Chapter 2: Gynecology
Which of the following would be the recommended management?

A. Beta blocker
B. Kegel exercises
C. Periurethral bulking
D. Post-coital antibiotics

Correct Answer: B
Explanation: (According to Williams Gynecology)
TREATMENT OF STRESS URINARY INCONTINENCE

CONSERVATIVE
- Modifying contributory factors — Before starting any treatment, contributory factors
such as medical conditions and medications should be addressed, particularly in older
patients
- Lifestyle modification
Þ Weight loss
Þ Dietary changes
Þ Constipation
Þ Smoking cessation
- Pelvic floor muscle exercises (PFME)
- Bladder training or scheduled voiding

SURGERY (Considered when conservative measures have failed)


- Midurethral slings
Are indicated for SUI with urethral hypermobility. Gold standard for SUI
Þ Tension-free Vaginal Tape (TVT)
o Most commonly used
Þ Transobturator Tape (TOT)

A-70-year-old female with prosthetic heart valve on warfarin, has grade 3 pelvic organ
prolapse. How to manage?

A. Pessary
B. Sacrocolpopexy
C. Le Fort Technique
D. Hysterectomy
Correct Answer: A
Explanation: (According to Williams Gynecology and UpToDate)
TREATMENT OF Pelvic organ prolapse.
- Conservative management — is the first line option for all women with POP, since
surgical treatment incurs the risk of complications and recurrence
- Surgical treatment — Surgical candidates include women with symptomatic prolapse who
have failed or declined conservative management of their prolapse.

300 | P a g e
Obstetrics and Gynecology: SMLE Questions and Answers Chapter 2: Gynecology
A 69-year-old female asymptomatic. On examination, there's grade 3 posterior vaginal wall
prolapse. She denies any symptoms. Which of the following is the most appropriate
managgement?

A. Posterior colporrhaphy
B. Conservative mx and observation
C. Surgical repair

Correct Answer: B
Explanation: (According to Williams Gynecology and UpToDate)
MANAGEMENT OF PELVIC ORGAN PROLAPSE
• EXPECTANT
- Is reasonable approach for asymptomatic or mildly symptomatic prolapse as well as in
patients without bothersome concomitant LUTS, obstructive voiding, or defecatory
dysfunction.
- Providers can offer reassurance that treatment is available if and when prolapse
becomes bothersome.

• CONSERVATIVE
- Prevent the prolapse becoming worse, to help decrease the frequency or severity of
symptoms
- To Avert or delay the need for surgery.
- Useful in patients with a mild degree of prolapse who desire future childbearing, have
frail health, or are unwilling to undergo surgery.
o Lifestyle modification
o Vaginal pessary (mainstay of nonsurgical treatment for POP)
o Pelvic floor muscle exercise PFMT
o Estrogen therapy

A 70-year-old lady had a hesterectomty 15 years ago now she has a very large vaginal vault
prolapse reach the premium what is the appropriate treatment?

A. Vaginal pessary
B. Colpoclesis
C. Anterior posterior repair
D. Vaginal surgery

Correct Answer: A
Explanation: (According to Williams Gynecology and UpToDate)
MANAGEMENT OF PELVIC ORGAN PROLAPSE
• EXPECTANT
- Is reasonable approach for asymptomatic or mildly symptomatic prolapse as well as in
patients without bothersome concomitant LUTS, obstructive voiding, or defecatory
dysfunction.

301 | P a g e
Obstetrics and Gynecology: SMLE Questions and Answers Chapter 2: Gynecology
- Providers can offer reassurance that treatment is available if and when prolapse
becomes bothersome.

• CONSERVATIVE
- Prevent the prolapse becoming worse, to help decrease the frequency or severity of
symptoms
- To Avert or delay the need for surgery.
- Useful in patients with a mild degree of prolapse who desire future childbearing, have
frail health, or are unwilling to undergo surgery.
o Lifestyle modification
o Vaginal pessary (mainstay of nonsurgical treatment for POP)
o Pelvic floor muscle exercise PFMT
o Estrogen therapy

A 70 years old female with prosthetic heart valve on warfarin, has grade 3 pelvic organ
prolapse. How to manage?

A. Pessary
B. Sacrocolpopexy
C. Le Fort Technique
D. Hystrectomy

Correct Answer: A
Explanation: (According to Williams Gynecology and UpToDate)
MANAGEMENT OF PELVIC ORGAN PROLAPSE
• EXPECTANT
- Is reasonable approach for asymptomatic or mildly symptomatic prolapse as well as in
patients without bothersome concomitant LUTS, obstructive voiding, or defecatory
dysfunction.
- Providers can offer reassurance that treatment is available if and when prolapse
becomes bothersome.

• CONSERVATIVE
- Prevent the prolapse becoming worse, to help decrease the frequency or severity of
symptoms
- To Avert or delay the need for surgery.
- Useful in patients with a mild degree of prolapse who desire future childbearing, have
frail health, or are unwilling to undergo surgery.
o Lifestyle modification
o Vaginal pessary (mainstay of nonsurgical treatment for POP)
o Pelvic floor muscle exercise PFMT
o Estrogen therapy

302 | P a g e
Obstetrics and Gynecology: SMLE Questions and Answers Chapter 2: Gynecology
Woman did some procedure (pelvic procedure) and then came complaining of urine from
vagina during urination(micturition). What is the diagnosis?

A. Vesicovaginal fistula
B. Ureterovaginal fistula
C. Uretherovaginal fisula
D. Rectovaginal fistula

Correct Answer: C
Explanation:

Woman had 7 kids, now she is complaining of a vaginal mass protruding more with
coughing, what is the next step?

A. Vaginal speculum exam


B. Retrograde Cystourethrogram

Correct Answer: A
Explanation: (According to UpToDate )
POP is diagnosed using pelvic examination.

A multiparous woman presents with pelvic fullness and the feeling of something coming out
of her vagina. How do you establish a diagnosis?

A. Speculum exam
B. Ultrasound
C. Biopsy
D. Computed tomography

Correct Answer: A
Explanation: (According to UpToDate)
POP is diagnosed using pelvic examination.

Old lady medically free with difficulty defecation and constipation, during defecation she
introduce her finger in the vagina to intiate movement, What is the most appropriate
management?

A. Posterior colporrhaphy
B. Anterior colporrhaphy
C. Enterocele resection
D. Hystrectomy

303 | P a g e
Obstetrics and Gynecology: SMLE Questions and Answers Chapter 2: Gynecology
Correct Answer: A
Explanation: (According to Williams Gynecology)
- Digital decompression by the woman of her posterior vaginal wall, perineal body, or distal
rectum to evacuate the rectum is the most common defecatory symptom associated with
posterior vaginal wall prolapse
- If the posterior vaginal wall descends, attempts are made to determine if a posterior vaginal
wall defect or an enterocele is present. An enterocele can be definitively diagnosed by
observing small-bowel peristalsis behind the vaginal wall. In general, bulges at the apical
segment of the posterior vaginal wall implicate enteroceles.

A 50-Year-Old female with vaginal wall prolapse grade 3, on medical treatment for 6
months with no improvement, what to do?

A. Conservative
B. Surgical repair
C. Follow-up with 3 months

Correct Answer: B
Explanation: (According to UpToDate)
Pelvic Organ Prolapse POP
- First line management of POP is conservative therapy. The mainstay of nonsurgical
treatment for POP is the vaginal pessary.
- Candidates for surgical repair of POP are women with symptomatic prolapse who have
unsuccessful or who have declined conservative management.

A 50-year-old female, presented complaining of a sensation of pelvic pressure but no stress


incontinence. Her work involves lifting heavy objects. On examination you find mucosa
bulging through the vagina. What’s the most appropriate management:

A. Kegel exercises
B. Pessary
C. Bladder neck fixation
D. Colporrhaphy

Correct Answer: B
Explanation: (According to UpToDate)
Pelvic Organ Prolapse POP
- First line management of POP is conservative therapy. The mainstay of nonsurgical
treatment for POP is the vaginal pessary.
- Candidates for surgical repair of POP are women with symptomatic prolapse who have
unsuccessful or who have declined conservative management.

304 | P a g e
Obstetrics and Gynecology: SMLE Questions and Answers Chapter 2: Gynecology
Patient para 4 Complaining of urinary frequency with no leak, work as loading heavy
boxes to her shop, on exam there's a large bulge on the anterior vaginal wall, What is the
management?

A. Kegel exercise
B. Vaginal pessaries
C. Anterior colporrhaphy

Correct Answer: B
Explanation: (According to UpToDate)
Pelvic Organ Prolapse POP
- First line management of POP is conservative therapy. The mainstay of nonsurgical
treatment for POP is the vaginal pessary.
- Candidates for surgical repair of POP are women with symptomatic prolapse who have
unsuccessful or who have declined conservative management.

A 70-Year-Old woman, sexually active, previous history of hysterectomy before, now


presenting complaining of prolapse and enlarged vaginal opening, what is the appropriate
procedure?

A. Sacrospinous fixation
B. Ant and post colpoperineorrhaphy
C. Manchester repair

Correct Answer: A
Explanation: (According to UpToDate)
- The most common risk factors associated with apical prolapse are vaginal parity (number
of vaginal deliveries) and previous hysterectomy
- Sacrospinous ligament suspension: Transvaginal procedure for treating vaginal vault
prolapse.

Female, para 6, complaining of heaviness and discomfort on her pelvic region, upon
examination it showed mass coming from introitus, what is the diagnosis?

A. Rectocle
B. Cystocele
C. Uterine prolapse
D. Enterocoele

Correct Answer: C
Explanation: (According to UpToDate)

305 | P a g e
Obstetrics and Gynecology: SMLE Questions and Answers Chapter 2: Gynecology
The most common risk factors associated with apical prolapse are vaginal parity (number of
vaginal deliveries) and previous hysterectomy

A 30-year-old women P6, came with second degree pelvic organ prolapse cyctocele and
rectocele, what is most appropriate management ?

A. Anterior and posterior colporraphy


B. Anterior colporraphy
C. Posterior colporraphy
D. Manchester repair

Correct Answer: A

Pregnant in third trimester has vaginal prolapse grade2. CTG normal, Vitals are normal,
what is the most appropriate management?

A. C-section
B. Reassurance
C. Induction of labor

Correct Answer: B
Explanation: (According to Uptodate)
Women may present with new symptoms or an exacerbation of POP during pregnancy. These
women are managed conservatively.

Pregnant at 24 week, everything is normal, she’s just complaining of protruding mass from
the vagina, diagnosed as posterior vaginal wall Prolapse, what to do ?

A. Reassure
B. Emergency delivery

Correct Answer: A
Explanation: (According to UpToDate)
Women may present with new symptoms or an exacerbation of POP during pregnancy. These
women are managed conservatively.

306 | P a g e

You might also like